Tải bản đầy đủ (.pdf) (96 trang)

Bo de thi cac truong THPT co dap an

Bạn đang xem bản rút gọn của tài liệu. Xem và tải ngay bản đầy đủ của tài liệu tại đây (5.81 MB, 96 trang )

<span class='text_page_counter'>(1)</span>m o .c. 7 4 2 nh. T. i s n e y u.

<span class='text_page_counter'>(2)</span> m o .c. 7 4 2 nh. T. i s n e y u.

<span class='text_page_counter'>(3)</span> m o .c. 7 4 2 nh. T. i s n e y u.

<span class='text_page_counter'>(4)</span> m o .c. 7 4 2 nh. T. i s n e y u.

<span class='text_page_counter'>(5)</span> Sở GD - ĐT Bắc Ninh Trường THPT Hàn Thuyên Câu 1 2 3 4 5 6 7 8 9 10 11 12 13 14 15 16 17 18 19 20 21 22 23 24 25 26 27 28 29 30 31 32 33 34 35 36 37 38 39 40 41 42 43 44 45 46 47 48 49 50. ĐÁP ÁN ĐỀ THI THPT QUỐC GIA LẦN 1 NĂM HỌC 2016 - 2017 MÔN: TIẾNG ANH Mã đề. 061. 104. 132. 209. 238. 357. 485. 570. 628. 743. 896. 914. D D B C C D A A D C B A D B E A C D A A B D D C A C C C A D B C D C A A A B A D B D B C B C B B B D. A B C B C C E A B D B C D C D A D B A D D C A D A B B C A D A C C C D D B C A C B B A D D B A B A B. A A B C A B D B A B B B C E D D C C D D B C D A D C C A C D D C A C B B D C B A A D C A D D D A B B. C A B B E B C A B C B D D A D D C C D A C C A D A D B D A C B A C C D B C D B A C B A C A B D B D A. B A A B B E D B A B C D C D A A B C C B A A A D D C D D C A C C C D C B B B D C C A A B D A B A D D. A C B B C B A A D E D B C D D C C C A A C D D B A C A D C A A D B D D C B D A A C B B B B B B B D C. C B B C C B D A D C E A D B C B C C B B D D B A D A A D A C D A C A B C D C D A C A D B A D B C B C. C A B B E D D A C A B B B C B A B D B C D D A D B A A C D A B C A D C A C C D B D D C D D C A C B D. D B B B B A B C A B C B B C E A D A C D D C C C D A D D A C B B B A D A D B D A C D A D B D A C A C. D D B B B A B D D B B B B E A C B B D D C C C D D C A A C D A B C B B A A A C D C C D A C A A D C C. B B B D A C D D D B C A E A B B B D C A C C C C D D C D A C A A B A D D B A B D B C C B A D A C D D. B A D B D C E B D A C A A B D A D B B A B B C D B C C D C A A D A C C D A D A D C B B D B B C A C C. T. i s n e y u. 7 4 2 nh. m o .c.

<span class='text_page_counter'>(6)</span> SỞ GD& ĐT TP. HỒ CHÍ MINH TRƯỜNG THPT TRẦN HƯNG ĐẠO. ĐỀ THI THỬ ĐẠI HỌC LẦN 2 MÔN ANH VĂN – KHỐI 12 (BAN A1+D) Ngày thi: 14/11/2016. Thời gian làm bài: 60 phút (50 câu trắc nghiệm) Mã đề thi 143 I-Mark the letter A, B, C, or D on your answer sheet to indicate the sentence that is closest in meaning to each of the following questions. Câu 1: The soccer team knew they lost the match. They soon started to blame each other. A. Not only did the soccer team lose the match but they blamed each other as well B. No sooner had the soccer team started to blame each other than they knew they lost the match. C. As soon as they blamed each other, the soccer team knew they lost the match. D. Hardly had the soccer team known they lost the match when they started to blame each other. Câu 2: Lucy always reminds me of my youngest sister. A. My youngest sister’s name is Lucy. B. I always think of Lucy, my youngest sister. C. Whenever I see Lucy, I think of my youngest sister. D. It is Lucy who is my youngest sister. Câu 3: They had such a fierce dog that nobody would visit them. A. So fierce was their dog that nobody would visit them. B. Their dog was fierce enough for anybody to visit them. C. If their dog weren’t fierce, somebody would visit them. D. So fierce a dog did they had that nobody would visit them. Câu 4: They are my two sisters. They aren’t teachers like me. A. They are my two sisters, that are teachers like me. B. They are my two sisters, neither of whom are teachers like me. C. They are my two sisters, both of those are teachers like me. D. Unlike me, neither of my two sisters aren’t teachers. Câu 5: “Why don’t you participate in the volunteer work in summer?" said Sophie. A. Sophie suggested my participating in the volunteer work in summer. B. Sophie made me participate in the volunteer work in summer. C. Sophie asked me why not participate in the volunteer work in summer. D. Sophie suggested me to participate in the volunteer work in summer.. m o c .. 7 4 h2. n i s n e y u. T. II-Mark the letter A, B, C, or D on your answer sheet to indicate the word or phrase that is closest in meaning to the underlined part in each of the following questions Câu 6: The students are advised to concentrate on their studying. A. remember B. pay attention to C. be interested in D. resemble Câu 7: In most countries, compulsory military service does not apply to women. A. mandatory B. superior C. beneficial D. constructive III-Mark the letter A, B, C, or D on your answer sheet to indicate the correct answer to each of the following questions. Câu 8: Many people will be out of ________ if the factory is closed. A. career B. Job C. profession D. work Câu 9: Neither Tom nor his brothers______ willing to help their mother with the housework. A. is B. was C. are D. has been Câu 10: Do you know the person _________ next to you in the evening class? A. whose sitting B. whom sits C. sitting D. who sit Câu 11: I can’t _________ of a word he is saying. A. make sense B. grasp C. comprehend D. understand Trang 1/5 - Mã đề thi 143.

<span class='text_page_counter'>(7)</span> Câu 12: Tony Blair is believed __________ for Liverpool last week. A. having left B. to have left C. to leave D. leaving Câu 13: _______ is increasing, which results from economic crisis. A. Employment B. Unemployed C. Unemployment D. Employ Câu 14: Tom: “_____” Mike: “I won’t say no!” A. How are things with you, Mike? B. What about playing badminton this afternoon? C. Mike, do you know where the scissors are? D. What’s your favourite, tea or coffee? Câu 15: In the US the first stage of compulsory education _______ as elementary education. A. to be generally known B. is generally known C. generally known D. is generally knowing Câu 16: It is vital to create a good impression ______ your interviewer. A. on B. with C. at D. for Câu 17: Geometry is the branch of mathematics _______ the properties of time, curves, shapes, and surfaces. A. it is concerned with B. that concerned with C. concerned with D. its concerned are Câu 18: We should make full use _____ the Internet as it is an endless source of information. A. of B. in C. with D. from Câu 19: Cindy: "Your hairstyle is terrific, Mary!" - Mary: - " ______________” A. Never mention it. B. Thanks, Cindy. I had it done yesterday. C. Thanks, but I'm afraid. D. Yes, all right. Câu 20: He didn’t know______ or stay until the end of the festival. A. whether to go B. if that he should go C. to go D. if to go Câu 21: If_____, the Xmas tree would look more impressive. A. done carefully B. being done carefully C. it were careful done D. it were to be carefully done. m o c .. 7 4 h2. n i s n e y u. T. IV-Read the following andmark the letter A, B, C, or D on your answer sheet to indicate the correct answer to each of the questions from 43 to 50. Harvard University, today recognized as part of the top echelon of the world's universities, came from very inauspicious and humble beginning. This oldest of American universities was founded in 1636, just sixteen years after the Pilgrims landed at Plymouth. Included in the Puritan emigrants to the Massachusetts colony during this period were more than 100 graduates of England's prestigious Oxford and Cambridge universities, and these universities graduates in the New Word were determined that their sons would have the same educational opportunities that they themselves had had. Because of this support in the colony for an institution of higher learning, the General Court of Massachusetts appropriated 400 pounds for a college in October of 1636 and early the following year decided on a parcel of land for the school; this land was in an area called Newetowne, which was later renamed Cambridge after its English cousin and is the site of the present-day university. When a young minister named John Harvard, who came from the neighboring town of Charlestowne, died from tuberculosis in 1638, he willed half of his estate of 1,700 pounds to the fledgling college. In spite of the fact that only half of the bequest was actually paid, the General Court named the college after the minister in appreciation for what he had done. The amount of the bequest may not have been large, particularly by today's standard, but it was more than the General Court had found it necessary to appropriate in order to open the college. Henry Dunster was appointed the first president of Harvard in 1640, and it should be noted that in addition to serving as president, he was also the entire faculty, with an entering freshmen class of four students. Although the staff did expand somewhat, for the first century of its existence the entire teaching staff consisted of the president and three or four tutors. Câu 22: The main idea of this passage is that ______________ . A. Harvard University developed under the auspices of the General Court of Massachusetts B. What is today a great university started out small Trang 2/5 - Mã đề thi 143.

<span class='text_page_counter'>(8)</span> C. John Harvard was key to the development of a great university D. Harvard is one of the world's most prestigious universities. Câu 23: The passage indicates that Harvard is _______________ A. One of the oldest universities in the world B. the oldest university in the world C. one of the oldest universities in America D. the oldest university in America Câu 24: It can be inferred from the passage that the Puritans who traveled to the Massachusetts colony were _________________ A. rather rich B. Rather well educated C. rather supportive of the English government D. rather undemocratic Câu 25: The pronoun "they" in the second paragraph refers to _______________ A. sons B. university graduates C. Oxford and Cambridge universities D. educational opportunities Câu 26: The "pounds" in the second paragraph are probably ______________ A. units of money B. college students C. Types of books D. school campuses Câu 27: Which of the following is NOT mentioned about John Harvard? A. What he died of B. Where he came from C. Where he was buried D. How much he bequeathed to Harvard Câu 28: The passage implies that __________________ . A. Someone else really served as president of Harvard before Henry Dunster B. Henry Dunster was an ineffective president C. Henry Dunster spent much of his time as president managing the Harvard faculty D. The position of president of Harvard was not merely an administrative position in the early ears Câu 29: The word "somewhat" in the last paragraph could best be replaced by _______ A. to and pro B. Back and forth C. side by side D. more or less. m o c .. 7 4 h2. n i s n e y u. T. V-Mark the letter A, B, C, or D on your answer sheet to indicate the word that differs from the rest in the position of the main stress in each of the following questions. Câu 30: A. engineering B. economics C. recommend D. curriculum Câu 31: A. economy B. certificate C. graduate D. semester VI-Mark the letter A, B, C, or D on your answer sheet to show the underlined part that needs correction in each of the following questions. Câu 32: My mother told me to watch the milk and don’t let it boil over. A B C D Câu 33: Fertilizer, which is added to the soil to replace or increase plant nutrients, include A B C animal and green manure, fish and bone meal and compost. D Câu 34: Foreign students who are doing a decision about which school to attend may not know A B C exactly where the choices are located. D VII-Mark the letter A, B, C, or D on your answer sheet to indicate the word or phrase that is OPPOSITE in meaning to the underlined part in each of the following questions. Câu 35: A chronic lack of sleep may make us irritable and reduces our motivation to work. A. calm B. miserable C. responsive D. uncomfortable Câu 36: I must have a watch since punctuality is imperative in my new job. A. Being courteous B. Being cheerful C. Being efficient D. Being late VIII-Read the following andmark the letter A, B, C, or D on your answer sheet to indicate the correct answer to each of the questions from 36 to 42. Trang 3/5 - Mã đề thi 143.

<span class='text_page_counter'>(9)</span> In early civilization, citizens were educated informally, usually within the family unit. Education meant simply learning to live. As civilization became more complex, however, education became more formal, structured, and comprehensive. Initial efforts of the ancient Chinese and Greek societies concentrated solely on the education of males. The post-Babylonian Jews and Plato were exceptions to this pattern. Plato was apparently the first significant advocate of the equality of the sexes. Women, in his ideal state, would have the same rights and duties and the same educational opportunities as men. This aspect of Platonic philosophy, however, had little or no effect on education for many centuries, and the concept of a liberal education for men only, which had been espoused by Aristotle, prevailed. In ancient Rome, the availability of an education was gradually extended to women, but they were taught separately from men. The early Christians and medieval Europeans continued this trend, and single-sex schools for the privileged through classes prevailed through the Reformation period. Gradually, however, education for women, in a separate but equal basis to that provided for men, was becoming a clear responsibility of society. Martin Luther appealed for civil support of schools for all children. Al the Council of Trent in the 16th century, the Roman Catholic Church encouraged the establishment of free primary schools for children of all classes. The concept of universal primary education, regardless of sex, had been born, but it was still in the realm of the single-sex school. In the late 19th and early 20th centuries, co-education became a more widely applied principle of educational philosophy. In Britain, Germany, and the Soviet Union the education of boys and girls in the same classes became an accepted practice. Since World War II, Japan and the Scandinavian countries have also adopted relatively universal co-educational systems. The greatest negative reaction to co-education has been felt in the teaching systems of the Latin countries, where the sexes have usually been separated at both primary and secondary levels, according to local conditions. A number of studies have indicated that girls seem to perform better overall and in science in particular. In single-sex classes, during the adolescent years, pressure to conform to stereotypical female gender roles may disadvantage girls in traditionally male subjects, making them reluctant to volunteer for experimental work while taking part in lessons. In Britain, academic league tables point to high standards achieved in girls’ schools. Some educationalists, therefore, suggest segregation of the sexes as a good thing, particularly in certain areas, and a number of schools are experimenting with the idea. Câu 37: Ancient education generally focused its efforts on __________. A. young people only B. on male learners C. both sexes D. female learners Câu 38: Education in early times was mostly aimed at __________. A. teaching skills B. learning new lifestyles C. learning to live D. imparting survival skills Câu 39: The first to support the equality of the sexes was __________. A. the Chinese B. the Jews C. Plato D. the Greek Câu 40: The word “informally” in this context mostly refers to an education occurring __________. A. in a department B. in classrooms C. ability D. outside the school Câu 41: When education first reached women, they were __________. A. separated from men B. locked up in a place with men C. deprived of opportunities D. isolated from a normal life Câu 42: When the concept of universal primary education was introduced, education __________. A. was intended for all the sexes B. was intended to leave out female learners C. was given free to all D. focused on imparting skills Câu 43: Co-education was negatively responded to in __________. A. conservative countries B. Japan C. South American countries D. the Scandinavian countries. m o c .. 7 4 h2. n i s n e y u. T. IX-Mark the letter A, B, C, or D on your answer sheet to indicate the word whose underlined part differs from the other three in pronunciation in each of the following questions. Câu 44: A. interview B. minute C. question D. suitable Câu 45: A. expand B. vacancy C. applicant D. category Trang 4/5 - Mã đề thi 143.

<span class='text_page_counter'>(10)</span> X-Read the following passage and mark the letter A, B, C, or D on your answer sheet to indicate the correct word for each of the blanks from 31 to 35. Here are tips that help succeed in your job interview: Always arrive early. If you do not know (46) _________ the organization is located, call for exact directions (47) ____________ advance. Leave some extra time for any traffic, parking, or unexpected events. If you are running late, call right away and let someone know. The best time to arrive is approximately 5 - 10 minutes early. Give yourself the time to read your resume one more time, to catch your breath, and to be ready for the interview. Once you are at the office, treat everyone you encounter with respect. Be (48) _____________ to everyone as soon as you walk in the door. Wear a professional business suit. This point should be emphasized enough. First (49) ____________ are extremely important in the interview process. Women should (50)_________ wearing too much jewelry or make up. Men should avoid flashy suits or wearing too much perfume. It is also important that you feel comfortable. While a suit is the standard interview attire in a business environment, if you think it is an informal environment, call before and ask. Regardless, you can never be overdressed if you are wearing a tailored suit. Câu 46: A. when B. why C. where D. that Câu 47: A. with B. in C. on D. for Câu 48: A. happy B. pleasant C. disappointed D. excited Câu 49: A. attendances B. attentions C. impressions D. pressures Câu 50: A. avoid B. suggest C. enjoy D. mind. m o c .. 7 4 h2. n i s n e y u. -----------------------------------------------. ----------- HẾT ----------. Họ, tên thí sinh:..........................................................................SBD…………………... T 1 2 3 4 5 6 7 8 9 10. ĐÁP ÁN. D C A B A B A D C C. 11 12 13 14 15 16 17 18 19 20. A B C B B A C A B A. 21 22 23 24 25 26 27 28 29 30. D B D B B A C D D D. 31 32 33 34 35 36 37 38 39 40. C C C A A D B C C D. 41 42 43 44 45 46 47 48 49 50. A A C C B C B B C A. Trang 5/5 - Mã đề thi 143.

<span class='text_page_counter'>(11)</span> TRƯỜNG ĐHKHTN KỲ THI KIỂM TRA CHẤT LƯỢNG LẦN 2 TRƯỜNG THPT CHUYÊN KHTN Môn: Tiếng Anh Thời gian làm bài: 60 phút , không kể thời gian giao đề. Mã đề : 203 (165876) Read the following passage and mark the letter A, B,C or D on your answer sheet to indicate the correct word or phrase that best first each of the numbered blank from 1 to 5 American folk music originated with (1) _____ people at a time when the rural population was isolated and music was not (2)______spread by radio, records, or music video. It was (3)_____by oral traditional and is noted for its energy ,humor, and emotional impact. The major source of aerly American folk songs was music from the Bristish Isles , but songs from Africa as songs of the American Indians have significant part in its heritage. Later settler from other countries also contributed songs. In the nineteenth century, composer Steven Foster wrote some of the most enduringly popular of all American songs ,(4)______soon became part of the folk tradition. Beginning in the 1930s , Woody Guthrie gained great popularity by adapting melodies and lyrics and supplying new ones as well. In the 1950s and 1960s , singer – composers such as Peter Seeger , Bob Dylan , Joan Baez continued this tradition by „urban‟ folk music. Many of these songs deal (5)_____important social issue, such as racial intergration and the war in Vietnam. Question 1: A. typical B. ordinary C. common D. popular Question 2: A. yet B. still C. until D. even Question 3: A. transferred B. transited C. transmitted D. transformed Question 4: A. who B. which C. that D. this Question 5: A. with B. in C. by D. at (165883) Mark the letter A, B, C, or D on your answer sheet to indicate the word(s) CLOSET in meaning to the underlined word(s) in each of the following question. Question 6: In order to better understand English, the students incorporated gramar, vocabulary, and speaking in their stidies. A. Combined B. granted C. scrutinized D. skipped Question 7: Some of the rude drivers, on the road today are the ones who will not allow other cars to merge into traffic, A. blend B. concentrate C. secede D. desensitize (165887) Mark the letter A, B, C, or D on your answer sheet to indicate the word(s) OPPOSITE in meaning to the underlined word(s) in each of the following question. Question 8: Slavery was abolished in the US in the 19th century. A. eradicated B. instituded C. eliminate D. required Question 9: The distinction between schooling and education implied by this remark is important. A. odd B. implicit C. obscure D. explicit (165890) Mark the letter A, B, C, or D on your answer sheet to indicate the sentence that is closet in meaning to each of the following question. Question 10: “I would like you not to play computer games any more, Amber”said her mother. A. Amber‟s mother would rather she doesn‟t play computer games any more. B. Amber‟s mother would rather her not to play computer games any more C. Amber‟s mother would rather she didn‟t play computer games any more. Truy cập vào: để học Toán – Lý – Hóa – Sinh – Văn – Anh – Sử - Địa tốt nhất!1.

<span class='text_page_counter'>(12)</span> D. Amber‟s mother would rather she wouldn‟t play computer games any more. Question 11: It is a pity that I can’t speak English as native speaker. A. I wish I could speak English as a native speaker B. I wish I couldn‟t speak English as a native speaker C. I wish I can speak English as a native speaker D. I wish I have spoken English as a native speaker Question 12: The music was so loud that we had to shout to each other. A. We didn‟t say loud enough for us to hear each other B. The music was too loud that we had to shout to each other. C. We didn‟t say loud enough to hear each other D. Because of loud music,we had to shout to each other. (165896) Mark the letter A,B,C or D on your answer sheet to indicate the word whose underlined part differs from the other three in the pronunciation in each of the following questions. Question 13: A.appreciate B. efficient C. suspicious D. apprentice Question 14: A. assure B. press C. blessing D. classic (165899) Mark the letter A,B,C or D on your answer sheet to indicate the word that differs from the other three in the position of primary stress in each of the following questions. Question 15: A. punishment B. behavior C. influence D. regional Question 16: A. acquaintance B. symbolize C. etiquette D. socialize (165903) Mark the letter A,B,C or D on your answer sheet to indicate the underlined part needs correction in each of the following questions. Question 17: Ceramic can be harder, light, and more resistant to heat than metals. A B C D Question 18: It is now believed that some damage to tissues may result from exposing it to frequent Xray. A B C D Question 19: The museum contains sixth century sculptures, eighteenth century swords and the A B dress won by nineteenthcenturyroyal family. C D (165907) Mark the letter A,B,C or D on your answer sheet to indicate the correct answer to each of ther following questinons. Question 20 : The rhinoceros, whose numbers have dropped alarmingly recently, has been declared a/an species. A. danger B. endanger C. endangered D. dangerous Question 21: A curriculum that ignores ethnic tensions, racial anlagonisms, cultural and religiousdifferences is not relevant. A. conlacts B. barriers C. diversity D. levels Question 22: Elizabeth explained during the meeting that it was a of what should have priority. A. belief B. fact C. possibility D. question. Truy cập vào: để học Toán – Lý – Hóa – Sinh – Văn – Anh – Sử - Địa tốt nhất!2.

<span class='text_page_counter'>(13)</span> Qucstion 23: A recent survey has shown that supporters of equal partnership in marriage are in the _____ A. crowd B. particular C. obligation D. mạjority Qucstion 24: The effects of literacy often extend personal benefits. A. after B. beneath C. over D. beyond Question 25: Instructors from the same ecological and vocational background as illiterates are more successful in literacy classes than school-teachers. A. carrying out B. pulling out C. acting out D. calling out Question 26: For a public campaing to succeed, it is important to make____of existing social organizations as well as other relations. A. fun B. advantage C. benefit D. use Question 27: The British and the American share the same language, but in other aspects they are as different as_______ A. cats and dogs B. chalk and cheese C. salt and pepper D. here and there Question 28: Had it not been for your support, we couldn‟this plan. A. have completed B. complete C. be completed D. have been completed Question 29: - I have gone to the doctor's to have a check up. - You ____ .You just had your check-up last week! A. didn't need to go B. needn‟t have gone C.needn‟t go D. don‟t need to go Question 30: In the kitchen there is a table . A. beautiful large round wooden B. large beautiful wooden round C. wooden round large beautiful D. round large vvooden beautiful Question 31: Everyone knows about pollution problems, but not many people have any solution. A.come up with B. looked into C. thought over D. got round to (165920) Mark the letter A,B,C or D on your answer sheet to indicate the most suitable response to complete each of the following exchanges. Question 32: Two friends Diana and Anne are talking about Anne‟s new hairstyle. - Diana: “ That new hairstyle suits you perfectly, Anne.” - Anne “ …………..” A. Never mind B. Don't mention it C. Thank you D. You re welcome. Question 33: Mary is talking to a porter in the hotel lobby Porter: “Shall I help you with your suitcase?” Mary: “_____.” A. Not a chance. B. That's very kind of you. C. I can‟t agree more. D. What a pity! (165923) Read the following pasage and mark the letter A. B, C or D on your answer sheet to indicate the correct answer to each of the questions from 34 to 40. As computers have become powerful tools for rapid and economic of production of picture, computer graphics has emerged as one of the most rapidly growing fields in computers scienee. It such used routinely in such diverse areas as business, industry, government, research, training, and madiecine. One of of the initial uses of computer graphics and ultimately its greatest use, been as an aid to design, generally referred to as computer – aided degign (CAD). One of its greatest advantages is that designers can see how an object will lock after construction and make changes freely and much more quickly than with. Truy cập vào: để học Toán – Lý – Hóa – Sinh – Văn – Anh – Sử - Địa tốt nhất!3.

<span class='text_page_counter'>(14)</span> hands drafting. For three-dimensional rendering of machine parts, engineers rely heavily on CAD. Automobile, spacecraft, aerospace, and ship designers use CAD terluaiques to design vehicles and test their pertomance. Building designs are also created with computer graphics systems. Architect can design a building layout create a three-dimersional model, and even go for simulated “walk” through the rooms or around the outside of the building. Business graphics is another rapidly growing are of computer graphics, where it is to create graphs, charts, and cost models summarize financial, statistical, msthematieal, scientific, and economic data. As an education aid, computer also has creative and commencial art applications, where it is used in advertissing, publishing and film productions, particulurly for computer animation, which is achieved by a sequential process. Question 34: What does the passage mainly discuss? A. Routine uses of computers B. Computer graphic applications C.The rapidly grown ag field of camrater science D.Computers as the architects af the future Question 35: The word “it" in line 2 refers to........................... A computer graphics B. computer science C. field D.computer Question 36: According to the passage, engineers use CAD for .......................... A. A simulated “walk” through modal rooms B. Rendering machine parts C. Making cost models D. Advertising products Question 37: Which of the following is NOT mentioned as a use of computer graphic in business? A charts B. cost models C.graphs D hiring Question 38: The word"applications" in the third paragraph means ........................ A. jobs B. uses C.creators D. layers Question 39: It can be inferred from the last paragraph that………. relies heavily on computer graphics A. making cartoons B. growing crops C. producing drugs D. Playing sports Question 40: The author's tone in this passage is............................... A. sarcastic B neutral C. pessimistic D. doubtful (165931) Read the following pasage and mark the letter A. B, C or D on your answer sheet to indicate the correct answer to each of the questions from 41 to 48. Another critical factor that plays a part in susceptibility to colds is age. A study done by the University of Michigan School of Public Health revealed particulars that seem to hold true for the general population. Infants are the most cold-ridden group, averaging more than six colds in their first year. Boys have more colds than girls up to age three. After the age of three, girls are more susceptible than boy's , and teenege girls average three colds a year to boy‟s two. The general incidence of continues to decline into maturity. Elderly people who are in good health have as few as one or two colds annually. One exception is founds among people in the twentics, especially women, who show a rise in cold infections, because people in this age group are most likely to have young children. Adults who delay having children until thirties forties experience the same sudden increase in cold infections. The study also found that economics play an important role. As income increases, the frequency at which are reported in the family decreases. Families with the lowest income suffer about a third more colds than families at the lower end. Lower income generally forces people to live in more cramped quarters than those typically occupied by wealthier by wealthier people, and crowding increses the opportunities for the cold virus totravel from person to person. Low income may also adversely influence diet. The degree to which. Truy cập vào: để học Toán – Lý – Hóa – Sinh – Văn – Anh – Sử - Địa tốt nhất!4.

<span class='text_page_counter'>(15)</span> poor nutrition affects susceptibility to colds is not yet clearly established, but an inadequate diet is suspected of lowering resistance generally. (source: TOEFL reading) Question 41: Which of the following is closet in meaning to the word “particulars” in line 22? A. Minor errors. B. specific facts. C. small distinctions. D. individual people. Question 42: What does the author claim about the study discussed in the passage? A. It contains many inconsistencies B. It specializes in children C. It contradicts the results of earlier studies in the field D. Its results apparently are relevant for the population as a whole Question 43: It maybe inferred from the passage that which of the following groups of people is most likely to catch colds? A. infant boys. B. young girls. C. teenage boys. D. elderly women. Question 44: There is information in the second paragraph of the passage to support which of the following conclusions? A. Men are more susceptible to cold than women B. Children infect their parents with colds. C. People who live in a cold climate have more colds than those who live in a warm one. D. People who don‟t have children are more susceptible to colds than those who do. Question 45: The phrase “in this age group” refers to A. Infants B. People in their twenties C. People in their thirties and forties D. Elderly people Question 46: The author‟s main purpose in writing the last paragraph of the passage is to A. Explain how cold viruses are transmitted B. Prove that a poor diet cause colds C. Discuss the relationship between income and frequency of colds D. Discuss the distribution of income among the people in the study Question 47: The word “cramped” is closest in meaning to A. Cheap. B. Crowded. C. depressing. D. simple. Question 48: The author‟s tone in this passage could best be described as A. Neutral and objective B. Humorous C. Tentative but interested. Truy cập vào: để học Toán – Lý – Hóa – Sinh – Văn – Anh – Sử - Địa tốt nhất!5.

<span class='text_page_counter'>(16)</span> D. Highly critical. (165940) Mark the letter A,B,C, or D on your answer sheet to indicate the sentence that best combines each pair of sentences in the following questions. Question 49: She tried very hard to pass the driving test. She could hardly pass it. A. Although she didn‟t try hard to pass pass the driving test, she could pass it. B. Despite being able to pass the driving test, she didn‟t pass it. C. Hard as she tried, she could hardly hard pass the driving test D. She tried very hard, so she passed the driving test satisfactorily Question 50: We didn‟t want to spend a lot of money. We stayed in a cheap hotel A. In stead of spending a lot of money, we stayed in a cheap hotel. B. In spite of spending a lot of money, we stayed in a cheap hotel C. We stayed in a cheap hotel, but we had to spend a lot of money. D. We didn‟t want to spend a lot of money. We stayed in the cheap hotel ........................THE END........................ ĐÁP ÁN 1 2 3 4 5 6 7 8 9 10. B A C B A A A B D C. 11 12 13 14 15 16 17 18 19 20. A D D A B A A D C C. 21 22 23 24 25 26 27 28 29 30. C D D D A D B A B A. 31 32 33 34 35 36 37 38 39 40. A C B B A B D B A B. 41 42 43 44 45 46 47 48 49 50. B D A B C C B A C A. Truy cập vào: để học Toán – Lý – Hóa – Sinh – Văn – Anh – Sử - Địa tốt nhất!6.

<span class='text_page_counter'>(17)</span> SỞ GD-ĐT PHÚ YÊN TRƯỜNG THPT PHAN CHU TRINH ---------------***---------------. ĐỀ THI THỬ THPT QUỐC GIA 2017 MÔN : TIẾNG ANH ------------------****---------------. I> Mark the letter A, B, C or D on your answer sheet to indicate the word whose underlined part is pronounced differently from that of the others. Question 1: A. telephone B. interpreter C. perverted D. restaurant Question 2: A. measure B. decision C. permission D. pleasure II>Mark the letter A, B, C, or D on your answer sheet to indicate the word that differs from the other three in the position of the primary stress in each of the following questions. Question 3: A. religious B. performance C. miserable D. including Question 4: A. decoration B. temperament C. opportunity D. expectation. m o c .. 7 4 2. III>Mark the letter A, B, C, or D on your answer sheet to indicate the underlined part that needs correction in each of the following questions.. h in. Question 5: My father used to giving me some good advice whenever I had a problem. A B C D Question 6: The woman of whom the red car is parked in front of the bank is a famous pop star. A B C D Question 7. Migrantworkers live in substandard unsanitary, and dilapidated housing and often A B C are lacking medical care. D. u T. s n e y. IV>Mark the letter A, B, C, or D on your answer sheet to indicate the correct answer to each of the following questions. Question 8: _______ is the study of the Earth's physical features and the people, plants, and animals that live in different regions of the world. a. Science b. Geography c. History d. Technology Question 9: As it was getting late, the boys decided to…….…..the campfire and crept into their sleeping bags. a. put up b. put off c. put out d. put on Question 10: You shouldn't _________to your teacher like that. It was very rude. A. have talked B. talk C. have be talked D. talked Question 11: We _________for three hours and are very tired. A. are walking B. have been walking C. were walking D. had been walking Question 12: It is believed _________causes insomnia. A. too much caffeine which B. that too much caffeine C. it in too much caffeine D. too much caffeine that Question 13: _______ non-verbal language is _______ important aspect of interpersonal communication. A.  /an B. A/ the C. the/ D.The/a Question 14 This small town developed ______ I had expected. A. not fast as B. as much fast as C. slowly than D. much faster than Question 15: Many people and organizations have been making every possible effort in order to save _______species. A. endangered B. dangerous C. fearful D. threatening Question 16: The man asked me _______ in English. A. what does this word mean B. what that word means C. what did this word mean D. what that word meant Question 17 : The student took that course ______ he could improve his English..

<span class='text_page_counter'>(18)</span> A. in order to B. though C. so that D. unless Question 18: An eyewitness described how ten people_____ in the fire. A. had been killed B. had killed C. were killed D. had been being Question 19: No one died in the accident,________? A. didn’t they B. did he C. didn’t he D. did they Question 20: Janet: “Do you feel like going to the cinema this evening?” -Susan: "_______ .” A. You’re welcome B. That would be great C. I feel very bored D. I don't agree, I'm afraid Question 21: - “ Can I use your motorbike this evening?” - “_________.” A. Of course, you can B. Of course, you might C. It’s my pleasure D. Do it if you can. m o c .. V>Mark the letter A, B, C, or D on your answer sheet to indicate the word(s) CLOSEST in meaning to the underlined word(s) in each of the following questions. Question 22. The teacher gave some suggestions on what could come out for the examination. A. hints B. symptoms C. effects D. demonstrations Question 23. I’ll take the new job whose salary is fantastic. A. wonderful B. acceptable C. pretty high D. reasonable. h in. 7 4 2. VI> Mark the letter A, B, C, or D on your answer sheet to indicate the word(s) OPPOSITE in meaning to the underlined word(s) in each of the following questions. Question 24. We’d better speed up if we want to get there in time. A. slow down B. turn down C. put down D. lie down Question 25: The machine has been out of order since last month. A. under repair B. functioning well C. sold out D. refusing orders. s n e y. u T. VII>Mark the letter A, B, C, or D on your answer sheet to indicate the sentence that is closest in meaning to each of the following questions. Question 26: "Why don't you ask the teacher for help?" Peter asked me. a. Peter advised me to ask the teacher for help. b. Peter recommended me not to ask the teacher for help. c. Peter told me the reason why I did not ask the teacher for help. d. Peter suggested that he should ask the teacher for help. Question 27: The Vietnamese students have to take an entrance exam ______. A. in order that they should go to a college or university B. for going to a college and university C. so that they can go to a college or university D. so as go to a college or university Question 28 Without skilful surgery, he would not have survived the operation. A. Had it been for skilful surgery he would not have survived the operation. B. He wouldn't have survived the operation if he hadn't had skilful surgery. C. But for skilful surgery he would not have survived the operation. D. With skilful surgery he would have survived the operation. VIII> Mark the letter A, B, C, or D on your answer sheet to indicate the sentence that best combines each pair of sentences in the following questions. Question 29: She received three letters this morning. All of them were from Tony A. All of the letters from Tony were received by her this morning B. She received three letters this morning, all of which were from Tony C. Three of the letters she received this morning were from Tony D. All letters from Tony were received by her this morning Question 30: : He is very intelligent. He can solve all the problems in no time. A. So intelligent is he that he can solve all the problems in no time. B. He is very intelligent that he can solve all the problems in no time. C. An intelligent student is he that he can solve all the problems in no time..

<span class='text_page_counter'>(19)</span> D. So intelligent a student is he that he can solve all the problems in no time. IX>Read the following passage and mark the letter A, B, C, or D on your answer sheet to indicate the correct word or phrase that best fits each of the numbered blanks from 31 to 35 : All relationships go through difficult times. In the past, when married couples had problems they usually didn’t (31) _________. They had to either (32)_________ with each other or continue to live together in an unhappy relationship. Getting divorced wasn’t an option for most people due to economic and social reasons. Some people believe that this wasn’t such a bad thing. They say that relationships require hard work and (33)_________ . “If a relationship is going to last a lifetime, you have to keep working at it,” says Doreen, who is celebrating her fiftieth wedding anniversary this year. “It isn’t all roses and romance. No one perfect all of the time. These days young people give up when there’s the (34)_________ argument.” Experts agree that communication is key. The most important thing is to keep talking. How many times have you heard yourself say to somebody , “If only you’d listen!” or “ I wish you wouldn’t do that!” The truth is, the (35)______ couples talk, the better their relationship can be. Question 31 : A. grow up B. split up C. chat up D. make up Question 32 : A. get on B. go out C. get back D. fall out Question 33 : A. agreement B. argument C. achievement D.commitment Question 34: A. smallest B. slightest C. most violent D.most controversial Question 35 : A. more B. fewer C. less D. least. 7 4 2. h in. s n e y. m o c .. X>Read the following passage and mark the letter A, B, C, or D on your answer sheet to indicate the correct answer to each of the questions from 36 to 42 A recent investigation by scientists at the U.S.Geological Survey shows that strange animal behavior might help predict earthquakes. Investigators found such occurrences within a ten – kilometer radius of the epicenter of a fairly recent quake. Some birds screeched and flew about wildly; dogs yelped and ran around uncontrollably. Scientists believed that animals can perceive environmental changes several hours or even days before the mishap. Animals were noted as being restless foer several weeks before a Tashkent, Uzbekistan, earthquake. An hour before the disaster, domestic animals refused go to indoors, and dogs howled and barked furiously. In 1960, an earthquake struck Agadir in Morocco. Survivors recall that stray animals, including dogs, were seen streaming out of town before the earthquake. In a safari zoo near San Francisco, llamas would not eat the evening before a 1979 quake, and they ran around wildly all night. Unusual animal behavior preceding earthquakes has been noted for centuries. British Admiral Robert Fitzroy reported huge flocks of screaming seabirds over Concepcion, Chile, in 1835. An hour and a half later, dogs were seen fleeing, and ten minutes later the town was destroyed. Similar stories of chickens running around in apparent states of panic, horses trembling, and dogs barking incessantly were recorded throughout the eighteenth and nineteenth centuries by survivors of eathquakes destruction in India, Yugoslavia, Peru, Mexico, and the United States. In 1976, after monitoring bizarre animal behavior, the Chinese predicted a devastating earthquake. Although hundreds of thousands of people were killed, the government was able to evacuate millions of other people and thus keep the death toll at a lower level. Question 36: What prediction may be made by observing animal behaviors? A. An impending earthquake B. The number of people who will die C. The ten kilometer radius from the epicenter D. The fact that an earthquake has occurred Question 37 : “devastating” means most nearly the same as A. destructive B. voracious C. intense D. forthcoming Question 38 : The author implies that animals are aware of an impending earthquake because A. of their superior intelligence B. they have certain instinctive abilities to perceive that humans do not possess C. they are generally closer to epicenter than the human observers. u T.

<span class='text_page_counter'>(20)</span> D. they react to other animal behavior Question 39 : The word “epicenter” is nearest in meaning to A. stratosphere B. contour C. periphery D. core Question 40 : All of the following statements are true EXCEPT A. some animals may be able to sense an approaching earthquake B. by observing animal behavior scientists perhaps can predict earthquake C. the Chinese have successfully predicted an earthquake and saved many lives D. only dogs and horses seem to possess the special perception that allows them to predict earthquakes Question 41 : The passage implies that if scientists can accurately predict earthquakes, there will be A. fewer animals going crazy B. a lower death rate C. fewer people evacuated D. fewer environmental changes Question 42: The word “evacuate” is closest in meaning to A. remove B. exile C. destroy D. emaciate. 7 4 2. m o c .. XI> Read the following passage and mark the letter A, B, C, or D on your answer sheet to indicate the correct answer to each of the questions from 43 to50 Noise is unwanted sound and is among the most pervasive pollutants today. Noise from road traffic, jet planes, jet skis, garbage trucks, construction equipment, manufacturing processes, lawn mowers, leaf blowers, and boom boxes, to name a few, are among the unwanted sounds that are routinely broadcast into the air. The problem with noise is not only that it is unwanted, but also that it negatively affects human health and well-being. Problems related to noise include hearing loss, stress, high blood pressure, sleep loss, distraction and lost productivity, and a general reduction in the quality of life and opportunities for tranquillity. We experience noise in a number of ways. On some occasions, we can be both the cause and the victim of noise, such as when we are operating noisy appliances or equipment. There are also instances when we experience noise generated by others just as people experience second-hand smoke. While in both instances, noises are equally damaging, second-hand noise is more troubling because it has negative impacts on us but is put into the environment by others, without our consent. The air into which second-hand noise is emitted and on which it travels is a “commons”, a public good. It belongs to no one person or group, but to everyone. People, businesses, and organizations, therefore, do not have unlimited rights to broadcast noise as they please, as if the effects of noise were limited only to their private property. On the contrary, they have an obligation to use the commons in ways that are compatible with or do not detract from other uses. People, businesses, and organizations that disregard the obligation to not interfere with others' use and enjoyment of the commons by producing noise pollution are, in many ways, acting like a bully in a school yard. Although perhaps unknowingly, they nevertheless disregard the rights of others and claim for themselves rights that are not theirs. We have organized to raise awareness of noise pollution and help communities take back the commons from those acting like bullies. Our efforts include building a library of resources and tools concerning noise pollution, establishing links to other groups that have similar collections, establishing networks among local noise activists, assisting communities and activists who are working to reduce noise pollution, and monitoring and advocating for stronger noise controls. Question 43 : It is stated in the passage that all of the following are things the noise comes from EXCEPT ______. A. Television, radio B. Lawn mower, leaf blowers C. Road traffic, garbage trucks D. Jet planes, watercraft Question 44: It can be inferred from the passage that the effects of noise pollution on the human body are that ______. A. people get accustomed to ear and heart diseases B. people focus mainly on hearing C. people have difficulty falling asleep D. people are sensitive about everything around them Question 45: Which of the following is supposed to be way to reduce noise pollution? A. cooperation with other noise activists B. fining people who break laws in noise regulation. u T. s n e y. h in.

<span class='text_page_counter'>(21)</span> C. construction of large projects along roads D. prohibiting public transport during peak hours Question 46: The word “it” in paragraph 4 refers to ______. A. the air B. a public good C. effect of noise D. second-hand noise Question 47 : It is mentioned in the passage that when we are operating noisy appliances or we experience noise generated by others ______. A. The noise is so troublesome if we make it or not. B. We feel more comfortable if we don’t experience second-hand noise. C. The noise have a strong influence on us, but not on others. D. We just ignore the serious detriment if the noise is put into the environment by others. Question 48: The word “disregard” in paragraph 5 could be best replaced by which of the following? A. ignore B. obey C. invade D. disturb. 7 4 2. m o c .. Question 49: Which of the following is NOT true according to the passage? A. Noise that is experienced by people who did not produce it is called second-hand noise. B. People’s enjoyment of the commons by producing noise pollution is regarded as a bully. C. Noise is one of the most common contaminants nowadays. D. People intentionally make noise when they work.. h in. s n e y. Question 50 : Which of the following best describes the author’s tone in the passage? A. Incentive B. Explanatory C. Cynical D. Provocative. u T. ------------------THE END--------------------.

<span class='text_page_counter'>(22)</span> ĐẠI HỌC KHOA HỌC TỰ NHIÊN KỲ THI THỰC HÀNH THQG 2016 – 2017 TRƯỜNG THPT CHUYÊN KHTN MÔN THI: TIẾNG ANH Thời gian làm bài: 60 phút, không kể thời gian phát đề Đề thi gồm 05 trang MÃ ĐỀ: 621 Bài thi gồm 50 câu trắc nghiệm Các chỉ dẫn bằng tiếng Anh (160421) Read the following passage and mark the letter A, B, C, or D on your answer sheet to indicate the correct answer to each of the questions from 1 to 7. One of the highest honors for formalists, writers, and musical composers is the Pulitzer Prize. First awarded in 1927, the Pulitzer Prize has been won by Ernest Hemingway, Harper Lee, John F. Kennedy, and Rodgers and Hammerstein, among others. As with many famous awards, this prize was named after its founder, Joseph Pulitzer. Joseph Pulitzer‟s story, like that of many immigrants to the United States, is one of hardship, hard work and triumph. Born in Hungary, Joseph Pulitzer moved to United States in 1864. He wanted to be a reporter, but he started his American life by fighting in the American Civil War. After the war, Pulitzer worked for the German - language newspaper, the Westliche Post. His skills as a reporter were wonderful, and he soon became a partial owner of the paper. In 1978, Pulitzer was able to start a newspaper of his own. Right from the first edition, the newspaper took a controversial approach to new. Pulitzer wanted to appeal to the average reader, so he produced exciting stories of scandal and intrigue. Such an approach is commonplace today, but in Pulitzer‟s time it was new and different. The approach led to the discovery of many instances of corruption by influential people. Pulitzer „paper became very famous and is still produced today. The success of Joseph Pulitzer‟s newspaper made him a very wealthy man, so he wanted to give something back to his profession. Throughout his later years, he worked to establish university programs for the teaching of journalism, and he funded numerous scholarships to assist journalism students. Finally, he wanted to leave a legacy that would encourage writers to remember the importance of quality. On his death, he gave two million dollars to Columbia University so they could award prizes to great writers. The Pulitzer Prize recipients are a very select group. For most, winning a Pulitzer Prize is the highlight of their career. If an author, journalist, or composer you know has won a Pulitzer Prize, you can be sure they are at the top of their profession. Question 1. Why does the writer mention "John F. Kennedy" in line 3? A.He was one of the inventors of the famous awards. B. He was one of the winners of the Pulitzer Prize. C. He was one of the people who selected the Pulitzer winners. D. He was in one of the scandals reported on by Joseph Pulitzer. Question 2. According to the reading passage, why did Joseph Pulitzer invent the Pulitzer Prize? A. to encourage people to remember his name and success B. encourage writers to remember the importance of quality C. to encourage journalism students to achieve their goals D. to encourage work of the Pulitzer winners Question 3. The word “partial" in the passage is closest in meaning to…… A. in part only B. brand new C. one and only D. very important . Question 4. According the passage, who receives the Pulitzer Prize? A. Columbia University graduates B.journalism students C.noted writers and composers D.most newspaper reporters Question 5. According to the reading passage, how did Joseph Pulitzer appeal to the average reader? A. He wrote about famous writers of journalism and literature B. He wrote stories about the war C. He produced his own newspaper. y u T. m o .c. n i s en. 7 4 2 h. Truy cập để học Toán – Lý – Hóa – Sinh – Văn – Anh – Sử - Địa tốt nhất!1.

<span class='text_page_counter'>(23)</span> D. He produced exciting stories of scandal and intrigue Question 6. Which sentence about Joseph Pulitzer is true according to the reading passage? A. He received a scholarship when he was a university student. B. He was rich even when he was young C. He was a reporter during the American Civil War D. He immigrated to the United States from Hungary Question 7. Which sentence about Joseph Pulitzer is NOT true according to the reading passage? A. Joseph Pulitzer was the first writer to the win the prize in 1917. B. The prize is aimed at encouraging a writer‟s career. C. Joseph Pulitzer left money to award to the prizewinners. D. Receiving the prize is one of the highest honors for writers.. m o .c. ( 160429) Mask the letter A,B,C or D on your answer sheet to indicate the underlined part that needs correction on each of the followwing questions Question 8. United Nations iss aimed at develop friendly relations among nations based on respect for A B the principle of equal rights ans sefl-determination of peoples C D Question 9. All members shall give the United Nations every assistance in any action it does in A B C accordance with the present Charter. D Question 10. Cutural diversity is important because most countries, workplaces, and schools A increasingly consist of various culture, racial and ethnic group. B C D. y u T. n i s en. 7 4 2 h. (160437) Mark the letter A, B, C, or D on your answer sheet to indicate the word whose underlined part differs from the other three in pronunciation in each of the following questions. Question 11.A. rhinoceros B. biologist C. reserve D. digest Question 12.A. form B. shortcoming C. chore D. sector ( 160443) Mark the letter A, B, c, or D on your answer sheet to indicate the correct answer to each of the following questions Question 13. The atmosphere at the meeting was very and everyone was on first name terms A. formal B. informal C. formality D. informality Question 14. Mark enjoys having fun by causing trouble. He's very boy. A. strong willed B. mischievous C. obedient D. well behaved Question 15. Many young people nowadays are prepared to getting married to pursue their professional careers. A satisfy B. sacrifice C. prefer D. confide Question 16. My boss's planeat 10:15 tomorrow morning, but I cannot pick him up. A. arrives B. is arriving C. will be arriving D. arrived Question 17. Jack asked Jil interested in any kinds of sports. A. if she were B.if were she C. if was she D. if she was Question 18. Whenever problemswe discuss frankly and find solutions quickly. Truy cập để học Toán – Lý – Hóa – Sinh – Văn – Anh – Sử - Địa tốt nhất!2.

<span class='text_page_counter'>(24)</span> A. make up B. come up C. put up D. turn up Question 19. Children should be taught that they have to……….. everything after they use it. A. put away B. pick off C. collect up D. catch on Question 20. Vietnamese parents normally do not let their children make a decision their own future career. A. in B. of C. on D. for Question 21. The police have just round the man and his car were swept away during the heavy storm last week. A. that B. which C.whose D.when Question 22. the more terrible the terrorism will become. A. The more weapons are powerful C The more powerful weapons are B. The weapons more powerful D. Weapons are the more powerful Question 23. We are concerned with the problem of energy resourceswe must also think of our environment. A. despite B. though C. however D. but Question 24. When finding a new house, parents should………….all the conditions for their children‟s education and entertainment. A. take into account B. make all the conditions C. get a measure of D. put into effect. m o .c. n i s en. 7 4 2 h. (160489) Mark the letter A, B, C or D on your answer sheet to indicate the most suitable response to complete each of following exchanges. Question 25. Two friends Diana and Anne are talking about their upcoming exams. - Diana:” Our midterm exams will start next Tuesday, are you ready?” - Anne:” ______” A. I‟m half ready. B. God save you. C. Thank you so much D. Don‟t mention it! Question 26. Mary is talking to her professor in his office. - Mary:” Can you tell me how to find material for my science report, professor?” - Professor:”______” A. I like it that you understand. B. Try your best, Mary. C. you can borrow books from the library. D. You mean the podcasts from other students?. y u T. (160500) Mark the letter A, B, C or D on your answer sheet to indicate the word that differs from the other three in the position of primary stress in each of the following questions. Question 27. A. maximum B. vacancy C. terrorist D. investment Question 28A. vertical B. contractual C. domestic D. outstanding (160511) Mark the letter A, B, C or D on your answer sheet to indicate the word (s) OPPOSITE in meaning to the underlined word (s) in each of the following questions. Question 29. Language teaching in the United State is base on the idea that the goal of language acquisition is communicative complete. A. Not good at socializing. C. unable to understand B. Excellent in orating in front of others D. incapable of working with words Question 30. This is new washing machine is not a patch on our old one. These clothes are still dirty. A. To be expensive B. To be strange C. to be broken D. to be better Truy cập để học Toán – Lý – Hóa – Sinh – Văn – Anh – Sử - Địa tốt nhất!3.

<span class='text_page_counter'>(25)</span> (160522) Mark the letter A, B, C or D on your answer sheet to indicate the word (s) CLOSEST in meaning to the underlined word (s) in each of the following questions Question 31. Today, American English is particularly influential, due to the USA‟s dominance of cinema, television, popular music, trade and technology (including the Internet) A. complete mastery B. overwhelming superiority C. complete control D. profound effect Question 32: Students ‘motivation for learning a language increase when they see connection between what they do in classroom and what they hope to do with the language in the future. A. the reason for which someone does something B. the action that someone takes to deal with something C. the eagerness that someone has to do something D. the excitement with which one is filled when doing something.. m o .c. 7 4 2 h. ( 160533) Mark the letter A, B, C or D on your answer sheet to indicate the sentence that is closest in meaning to each of the following questions Question 33. “What are you going to do with such a long list of books, Dane?” asked Sarah A. Sarah was curious why Dane had such a long list of books. B. Sarah asked Dane what he was going to do with such a long list of books. C. Sarah could not understand why Dane was borrowing such a long list of books. D. Sarah warned Dane not to borrow such a long list of books. Question 34. “Are you going to the cinema with us tonight, Susan?” asks her friends. A. Susan‟s friend asked her whether she went to the cinema with them that night B. Susan‟s friend asked her if she was going to the cinema with them that night C. Susan‟s friend would like to invite her to go to the cinema with them that night D. Susan‟s friend would rather her went to the cinema with them that night. Question 35. The Internet has enabled most people to get contact in a matter of moment A. Most of people have been able to get in contact by the Internet in a matter of moments. B. Most of people have got in contact as enable in a matter of moments by the Internet C. On the Internet, most of people are able to get in contact a matter of moments. D. On the Internet, most of people can find their contact a matter of moments. y u T. n i s en. ( 160542) Read the following passage and mark the letter A, B, C or D on your answer sheet to indicate the correct word or phrase that best fist each of the numbered blanks from 36 to 40 Tim Samaras is a storm chaser. His job is to find tornadoes and follow them. When he gets close to a tornado, he puts a special tool (36)____a turtle probe on the ground. This tool measures things like a twister‟s temperatune, humidity, and wind speed. With this information, Samaras can lean what causes tornadoes to develop. If meteorologists understand this, they can warn people (37)____twisters sooner and save lives. How does Samaras hunt tornadoes? It‟s not easy. First, he has to find one. Tornadoes are too small to see using weather satellites. So Samaras can‟t rely on these tools to find a twister. (38)____, he waits for tornadoes to develop. Once Samaras sees a tornado, the chase begins. But a tornado is hard to follow. Some tornadoes change (39)____several time – for example, moving east and then west and then east again. When Samaras finally gets Truy cập để học Toán – Lý – Hóa – Sinh – Văn – Anh – Sử - Địa tốt nhất!4.

<span class='text_page_counter'>(26)</span> near a tornado, het puts the turtle probe on the ground. Being this close to twister is (40)____. He must get away quickly. Question 36. A. called B. know C. made D. meant Question 37. A. with B. about C. at D. for Question 38. A. Rather B. Still C. Instead D. Yet Question 39. A. progression B. movement C. dimension D. direction Question 40. A. terrifly B. terrifying C. terrified D. terrifies ( 160551) Mark the letter A, B, C, or D to indicate the sentence that best combines each pair of sentences in the following question. Question 41. The teacher has done his best to help all students. However, none of them made any effort on their part A. The teacher has done his best to help all student, then, none of them made any effort on their part B. Although the teacher has done his best to help all students, none of them made any effort on their part C. Because the teacher has done his best to help all students, , none of them made any effort on their part D. If the teacher has done his best to help all students, , none of them made any effort on their part Question 42. “ Finish your work. And then you can go home”. A. “ You can‟t go home until you finish your work” B. “ You finish your work to go home as early as you can” C. “ When you go home, finish your work then” D. “ Because you have finished your work, you can go home”.. y u T. m o .c. n i s en. 7 4 2 h. ( 160554) Read the following passage and mark the letter A, B,C or D on your answer sheet to indicate the correct answer to each of the questions from 43 to 50 Clara Barton became known as “ The Angel of the Battlefield” during the American Civil War. Born in Oxford, Massachusetts in 1821, Clara Barton‟s interest in helping soldiers on the battlefield began when she was told army stories from her father. Another event that influenced her decision to help soldiers was an accident her brother had. His injuries were cared for by Barton for 2 year. At the time, she was only 11 years old. Barton began teaching school at the age of 15. She taught for 18 years before she moved to Washington, D.C in 1854. The civil war broke out 6 years later. Immediately, Barton started war service by helping the soldiers with their needs. At the battle of Bull run, Clara Barton received permission from the government to take care of the sick and hurt. Barton did this with great empathy and kindness. She acknowledged each soldier as a person . her endurance and courage on the battlefield were admired by many. When the war ended in 1865 , she used 4 years of her life to assist the government in searching for soldiers who were missing during the war. The search for missing soldiers and years of hard work made her feeble physically. In 1869, her doctors recommended a trip to Europe for a rest. While she was on vacation , she became involved with the International Red Cross, an organization set up by the Geneva Convention in 1864. Clara Barton realized that the Red Cross would be a best help to the United States. After she returned to the United States, she worked very hard to create an American Red Cross. She talked to government leaders and let American people know about the Red Cross. In 1881, the National Society of the Red Cross was finally established with its headquarters in Washington , D.C. Clara Barton managed its activities for 23 years.. Truy cập để học Toán – Lý – Hóa – Sinh – Văn – Anh – Sử - Địa tốt nhất!5.

<span class='text_page_counter'>(27)</span> Barton never let her age stop her from helping people. At the age of 79, she helped food victims in Galveston, Texas. Barton finally resigned from the Red Cross in 1904. She was 92 years old and had truly earned her title “ The Angel of the Battle”. Question 43: According to the paragraph 1, which of the following is true of the young Barton Clara? A. She helpep her father when he was a soldier B. She suffered from an accident when she was 11 C. She helped her brother who hurt in an accident D. She made a decision to live with her brother for 2 years Question 44. The phrase broke out in paragraph 2 is closest in meaning to A. extended C. broken down C. closed D. began Question 45. The word this in paragraph 2 refers to A. recognized each soldier as a person B. cooker for soldiers C. took care of the sick and hurt D. received permission Question 46. The word acknowledged in paragraph 2 could best be replaced by A. nursed B. recognized C. pleaded D. believed Question 47. What can be inferred about the government? A. It did not always agree with Clara barton B. It did not have the money to help Clara Barton C. It showed Clara Barton great empathy and kindness D. It had respect for Clara Barton Question 48. What does the author mention about the American Red Cross? A. It was disapproved again and again by the Geneva Convention B. Barton tried to have it set up in America C. The American people were not interested in the Red Cross D. It was first established in the Unites States Question 49. What is the main idea of the passage? A. Clara Barton helped wounded soldiers and she was the founder of the Red Cross B. Clara Barton was a kind and strong woman who helped people in need C. Clara Barton becam a nurse during the American Civil War D. Clara Barton worked for disaster victims until she was old Question 50. What can be the best title of the reading passage? A. The angel of the Battlefield B. The American Red Cross C.The American Civil War D. The International Red Cross. y u T. m o .c. n i s en. 7 4 2 h. ………………………THE END………………………. Truy cập để học Toán – Lý – Hóa – Sinh – Văn – Anh – Sử - Địa tốt nhất!6.

<span class='text_page_counter'>(28)</span> ĐÁP ÁN 1 2 3 4 5 6 7 8 9 10. B B A C D D A A C C. y u T. 11 12 13 14 15 16 17 18 19 20. C D B B B A D B A C. 21 22 23 24 25 26 27 28 29 30. A C D A A C D A A D. 31 32 33 34 35 36 37 38 39 40. B A B B A A B C D B. B A C D C B D B B A. m o .c. 7 4 2 h. n i s en. 41 42 43 44 45 46 47 48 49 50. Truy cập để học Toán – Lý – Hóa – Sinh – Văn – Anh – Sử - Địa tốt nhất!7.

<span class='text_page_counter'>(29)</span> VnDoc - Tải tài liệu, văn bản pháp luật, biểu mẫu miễn phí THI THỬ THPT QUỐC GIA (21/4/2016) - SỞ GDDT HÀ NỘI MÔN ANH (90 PHUT) SECTION A (8,0 points) Mark the letter A, B, C or D on your answer sheet to indicate the word whose underlined part differs from the Other three in pronunciation in each of the following questions. 1. A. against. B. awful. C. award. D. ahead. 2. A. weather. B. healthy. C. although. D. breathe. Mark the letter A, B, C, or D on your answer sheet to indicate the word that differs from the other three in the position of primary stress in each of the following questions. 3. A. distraction. B. assignment. C. concentrate. D. economist. 4. A. globalization. B. intellectual. C. multinational. D. international. 5. A. scientific. B. imaginary. C. reviewer. D. advantage. Mark the letter A, B, C, or D on your answer sheet to indicate the correct answer to each of the following questions. 6. Before you start applying for any job, you must be absolutely sure you have the right ________ . A. qualities. B. qualifications. 7. Linda: "Let's meet outside the library.". C. qualifying. D. qualifiers. - Lucy: " ________”. A. I’d like to go to the library.. B. How about meeting again?. C. Yes, let's do it.. D. Is 6.30 all right?. 8. "Would you like to come to my house for dinner tonight?"-“ ________ ." A. Thanks. I'd love to. B. You're welcome. C. No, 1 wouldn't. D. No, I don't. 9. How about taking a shower instead of ________ to save water. A. to have a bath. B. have a bath. C. having a bath. D. had a bath. 10. ________rain or snow, there are always more than fifty thousand fans at the football games. A. Despite. B. In spite. 11.. ________ the teacher's questions.. Peter tried to avoid. A. answered. B. to answer. C. Despite of C. answering. D. Although D. answers. 12. There was no one else at the box office. Mary ________ in a queue. A. needn't wait. B. doesn't need to wait. C. needn't to wait. D. didn't need to wait. 13. Sorry, I can't join the picnic. I'm busy. ________ , I don't have any money after buying all these stuff. A. So. B. Besides. C. Although. D. However. 14. ________ hungry I am, I never seem to be able to finish off a whole pizza. A. Besides. B. However. C. Nevertheless. D. Whatever. 15. Unlike most Europeans, many Americans ________ a bowl of cereal for breakfast every day. A. use to eat. B. used to eating. C. are used to eating. 16. A new school________. They hope to finish building it next month.. D. are used to eat.

<span class='text_page_counter'>(30)</span> VnDoc - Tải tài liệu, văn bản pháp luật, biểu mẫu miễn phí A. is being built. B. has been built. C. is built. D. was built. 17. Ann is very temperamental. How do you ________ her? A. put with. B. putting up with. C. put up to. D. put up with. 18. This new model not only saves time but also ________ operating on two batteries instead of four. A. saving energy 19. It is a ________. B. save energy. C. saves energy. D. to save energy. .. A. blue sleeping polyester bag. B. polyester sleeping blue bag. C. blue polyester sleeping bag. D. sleeping blue polyester bag. 20. I didn't enjoy this book on how to succeed in business. It wasn't very ________ A. well typed. B. poorly written. C. good written. .. D. well written. 21. We shouldn't use too many plastic bags because they are very hard to ________. A. dissolve. B. melt. C. heat. D. soften. C. learnt. D. learning. 22. It's time she ________ how to look after herself. A. learns. B. learn. 23. The trouble with Frank is that he never ________ on time for a meeting. A. turns up. B. turn up. C. turning up. D. to turn up. Mark the letter A, B, C or D on your answer sheet to indicate the word(s) CLOSEST in meaning to the underlined word(s) in each of the following questions. 24 My elder brother failed his final exam, which depressed my parents A. encouraged. B. satisfied. C. disappointed. D. pleased. 25 I came to John’s party last night but I stayed there for a while before I left. A. for relaxation. B. for a whole night. C. a long period of time. D. for a short period of. time 26. She was a devoted teacher. She spent most of her time teaching and taking care of her students. A. intelligent. B. dedicated. C. polite. D. honest. 27. You can withdraw money from the account at any time without penalty. A. punishment. B. offense. C. demand. D.. loss. Mark the letter A, B, C, or D on your answer sheet to indicate the word(s) OPPOSITE in meaning to the underlined word(s) in each of the following questions. 24. Scientists proof that choosing a career for money will make you less efficient, happy and more selfish. A. effective. B. ineffective. C. capable. D. proficient. 25. A surprising percentage of the population in remote areas is illiterate. A. able to speak fluently. B. unable to speak fluently. C. unable to read and write D. able to read. and write Read the following passage and mark the letter A, B, C, or D on your answer sheet to indicate the correct word for each of the blanks..

<span class='text_page_counter'>(31)</span> VnDoc - Tải tài liệu, văn bản pháp luật, biểu mẫu miễn phí What's your dream? Touring castles in Scotland? Walking on the Great Wall of China? Working to improve the lives of women in rural Uganda? If you're thinking of studying (30) ______, there's no end to the places you can go, things you can see, and subjects you can study. At many top schools, such as Duke, Tufts, or Brown, over a third of the junior class take the opportunity to complete part of their (31) _____ outside of the United States. Even if your school doesn't have an extensive study abroad program, you can often get credit from a different school. Multiple benefits increase to those who spend significant (32)____ in another country, and a significant proportion of students see the experience as an important (33) _____ of their college years. You're likely to have fun. But if you're also thinking about study abroad as a way to gain a critical career advantage read on. You'll find that all foreign experiences are not created equal in the minds of employers. Employers are looking for graduates who can (34). ____ well with others, both in person and in writing.. They know the (35) ____ of cross-cultural understanding and an appreciation for different points of view. They gravitate toward students who demonstrate maturity, initiative, and (36)______. All of these assets can be demonstrated through study abroad, but it's going to be much (37) ____ to set yourself apart if you've taken the easy route. It's not hard to find the "easy route": that's the one where you go with your friends to another country; all the arrangements are made for you by the school- including the American-style apartment where you live with your classmates. In this scenario, it doesn't (38) ___ which country you go to because all your classes will be in English, and possibly taught even by your American professors. You'll undoubtedly have a somewhat different experience, but to do the "easy route" is to forego some of the major (39) ________ of your time away. 30. A. away. B. outdoor. C. outside. D. abroad. 31. A. educator. B. educating. C. education. D. educations. 32 A. time. B. lifetime. C. moment. D. period. 33. A. way. B. thing. C. part. D. terminal. 34. A. communicate. B. work. C. talk. D. do. 35. A. relation. B. link. C. reason. D. importance. 36. A. creating. B. creation. C. creativity. D. creative. 37. A faster. B. shorter. C. harder. D. quicker. 38.A. matter. B. show. C. problem. D. trouble. 39. A waste. B. advantage. C. experience. D. giving. Read the following passage and mark the letter A, B, C, or D on your answer sheet to indicate the correct answer to each of the following questions..

<span class='text_page_counter'>(32)</span> VnDoc - Tải tài liệu, văn bản pháp luật, biểu mẫu miễn phí Telecommuting is some form of computer communication between employees’ homes and offices. For employees whose job involve sitting at a terminal or word processor entering data or typing reports, the location of the computer is of no consequence. If the machine can communicate over telephone lines, when the work is completed, employees can dial the office computer and transmit the material to their employers. A recent survey in USA Today estimates that there are approximately 8,7 million telecommuters. But although the numbers are rising annually, the trend does not appear to be as significant as predicted when Business Week published “The Portable Executive” as its cover story a few years ago. Why hasn’t telecommuting become more popular? Clearly, change simply takes time. But in addition, there has been active resistance on the part of many managers. These executives claim that supervising the telecommuters in a large work force scattered across the country would be too difficult, or, at least, systems for managing them are not yet developed, thereby complicating the manager’s responsibilities. It is also true that employees who are given the option of telecommuting are reluctant to accept the opportunity. Most people feel that they need regular interaction with a group, and many are concerned that they will not have the same consideration for advancement if they are not more visible in the office setting. Some people feel that even when a space in their homes is set aside as a work area, they never really get away from the office. 31: With which of the following topics is the passage primarily concerned? A. The advantages of telecommuting.. B. A definition of telecommuting.. C. An overview of telecommuting.. D. The failure of telecommuting.. 32.How many Americans are involved in telecommuting? A. More than predicted in Business Week.. B. More than 8 million.. C. Fewer than estimated in USA Today.. D. Fewer than last year.. 33. The phrase “of no consequence” means ________ A. of no use. B. of no good. .. C. unimportant. D. irrelevant. 34. The author mentions all of the following as concerns of telecommuting, EXCEPT ________ A. the opportunities for advancement.. B. the different system of supervision.. C. the lack of interaction with a group.. D. The work place is in the home.. 35: The word “executives” in line 10 refers to ________ A. telecommuters. B. managers. A. systems. B. telecommuters. .. C. employees. 36: The word “them” in line 10 refers to ________. .. D. most people. .. C. executives. D. responsibilities. 37. The reason why telecommuting has not become popular is that the employees ________ A. need regular interaction with their families. B. are worried about the promotion if they are not seen at the office. C. feel that a work area in their home is away from the office.. ..

<span class='text_page_counter'>(33)</span> VnDoc - Tải tài liệu, văn bản pháp luật, biểu mẫu miễn phí D. are ignorant of telecommuting. 38: It can be inferred from the passage that the author is ________ A. a telecommuter. B. the manager. C. a statistician. . D. a reporter. 39: The word “reluctant” in line 13 can best be replaced by ________ A. opposite. B. willing. C. hesitate. . D. typical. 40: When Business Week published “The Portable Executive”, it implied that ________. .. A. systems for managing telecommuters were not effective. B. there was resistance on the part of many managers about telecommuting. C. the trend for telecommuting was optimistic. D. most telecommuters were satisfied with their work. Mark the letter A, B. C, or D on your answer sheet to indicate the underlined part following that needs correcting. 50. Many people believed that women's natural roles were as mother and wife. 51. The World Health Organization's main activities are carrying of research on medical development and improving international health care. 52. In nature, the distributive of plants is obviously related to climate. 53. Women are now better educate and can promote themselves much more easily than in the past. 54. ASEAN has actively worked to improve the socio-economic situation and solve problems among their member countries. Read the following passage and mark the letter A, B, C, or D on your answer sheet to indicate the correct answer t each of the following questions. It is commonly believed in the United States that school is where people go to get an education. Nevertheless, it has been said that today children interrupt their education to go to school. The distinction between schooling and education implied by this remark is important. Education is much more open-ended and all-inclusive than schooling. Education knows no bounds. It can take place anywhere, whether in the shower or on the job, whether in a kitchen or on a tractor. It includes both the formal learning that takes place in schools and the whole universe of informal learning. The agents of education can range from a revered grandparent to the people debating politics on the radio, from a child to a distinguished scientist. Whereas schooling has a certain predictability, education quite often produces surprises. A chance conversation with a stranger may lead a person to discover how little is known of other religions. People are engaged in education from infancy on. Education, then, is a very broad, inclusive term. It is a lifelong process, a process that starts long before the start of school, and one that should be an integral part of one's entire life. Schooling, on the other hand, is a specific, formalized process, whose general pattern varies little from one setting to the next. Throughout a country, children arrive at school at approximately the same time, take assigned seats, are taught by an adult, use similar textbooks, do homework, take exams, and so on. The.

<span class='text_page_counter'>(34)</span> VnDoc - Tải tài liệu, văn bản pháp luật, biểu mẫu miễn phí slices of reality that are to be learned, whether they are the alphabet or an understanding of the workings of government, have usually been limited by the boundaries of the subject being taught. For example, high school students know that they are not likely to find out in their classes the truth about political problems in their communities or what the newest filmmakers are experimenting with. There are definite conditions surrounding the formalized process of schooling. 55. What is the main idea of the passage? A. The best school teach a wide variety of subjects B. Education and schooling are quite different experiences C. Students benefit from schools, which require long hours and homework D. The more years students go to school, the better their education is 56. The word “distinction” in paragraph 1 is closest in meaning to ___________. A. similarity. B. difference. C. differently. D. insignificance. 57. The word “bounds” in paragraph 2 is closest in meaning to ___________. A. rules. B. experience. C. limits. D. exceptions. 58. The word “chance” in paragraph 2 is closest in meaning to_________________. A. unplanned. B. unusual. C. lengthy. D. lively. 59. The word “an integral” in paragraph 2 is closest in meaning to ____________. A. an equitable. B. a profitable. C. a pleasant. D. an essential. 60. The word “they” in paragraph 3 refers to ____________. A. boundaries. B. similar textbooks. C. slices of reality. D. seats. 61. The phrase “For example,” in paragraph 3 introduces a sentence that gives examples of _________. A. similar textbooks B. the results of schooling C. the workings of a government D. the boundaries of classroom subjects 62. The passage supports which of the following conclusions? A. Without formal education, people would remain ignorant. B. Education systems need to be radically reformed. C. Going to school is only part of how people become educated. D. Education involves many years of professional training. 63. The passage is organized by ___________________________. A. listing and discussing several educational problems. B. giving examples of different kinds of schools. C. narrating a story about excellent teacher. D. contrasting the meanings of two related. words 64. The writer seems to agree that ________. A. Education is more influential than schooling. B. Education is not as important as schooling. C. Schooling is unlimited and more informal. D. Schooling is as important than education. SECTION B: (2,0 points).

<span class='text_page_counter'>(35)</span> VnDoc - Tải tài liệu, văn bản pháp luật, biểu mẫu miễn phí Part I: Finish each of the following sentences in such a way that it means the same as the sentence printed before it. Write your answers on your answer sheet. 65. Jane asked Terry whether he would attend the meeting the following day. (Jane asked " ………………………………………………………… 66. My brother spends two hours surfing the Internet every night. (It takes 67.. …………………………………………………………... When the police caught him, he was climbing over the garden wall.. (The police caught………………………………………………………………… 68. Although Tom was a poor student, he studied very well. (Despite. ……………………………………………………………………………. 69. Many people believed that success depends on hard work. (The harder………………………………………………………………………… Part II. In about 140 words, write a paragraph about your plan for this summer vacation. The following prompts might be helpful to you. Place you will go - Person you will go with - Things you will do Đáp án 1. B 2 B 3. C 4. A 5. A 6. B 7. C 8. A 9. C 10. A. II C 12. 0 13. B 14. B 15. C 16. A 17. I) 18. C 19. C 20. D. 21 . A 22. C 23 . A 24. C 25 . D 26. B 27 . A 28 . B 29 . D 30. D. 31 . C 32. A 33 . C 34. A 35 . D 36. C 37. C 38 . A 39 . B 40. D. 41 . B 42. C 43 . D 44. B 45 . B 46. I) 47 . D 48 . C 49 . B 50. D. 51 . B 52. B 53 . B 54. D 55. C 56. B 57 . C 58 . A 59 . D 60. C. 61 . D 62. C 63 . D 64. A. SECTIO N B PAR T I: 1. Jane asked " Will yo u attend the meeting yesterday" 2. It takes my brother tw o hours to surf the Internet every night. 3. The police caught him climbin g over the garden wall. /Th e police caught him while he wa s climbing over the garden wall. 4. Despite being a po o r student, To m studied very well. 5. The harder yo u work, the mor e successful you are.

<span class='text_page_counter'>(36)</span> VnDoc - Tải tài liệu, văn bản pháp luật, biểu mẫu miễn phí SỞ GD&ĐT YÊN BÁI TRƯỜNG THPT CẢM NHÂN ĐỀ DỰ PHÒNG Đề thi gồm có 05 trang. KỲ THI THPT QUỐC GIA NĂM 2017 Môn TIẾNG ANH Thời gian làm bài: 60 phút, không kể thời gian phát đề ----------------------------------------------------------------. Mark the fetter A, B, C, or D on your answer sheet to indicate the word whose underlined part difers from the other three in pronounciation in each of the following Questions. Question 1: A. share B. rare C. are D. declare Question 2: A. cooks B. loves C. joins D. spends Mark the letter A, B, C, or D on your answer sheet to indicate the word that differs from the rest in the position of the main stress in each o f the following Questions. Question 3: A. pursuit B. leisure C. popular D. television Question 4: A. memento B. solitude C. spectacular D. athletics Mark the letter A, B, C, or D on your answer sheet to show the underlined part that needs correction in each of the following questions. Question 5: What does he usually does ? - He usually waters flowers in the garden. A B C D Question 6: Hurry up! The train is come. I don’t want to miss it. A B C D Question 7: If John is passing the GCSE examination, he would have been allowed to take A B C the entrance examination to the university. D Mark the letter A, B, C, or D on your answer sheet to indicate the correct answer to each of the following Questions. Question 8: The academic year in Viet Nam is _______ into 2 terms. A. devide B. devided C. deviding D. devides Question 9: The _______ school system is paid for by the state. A. state B. private C. home D. their Question 10: Mrs Brown was the first owner _______ dog won three prizes in the same show. A. who B. whose C. where D. which Question 11: On Sunday, _______was my birthday, we went out for a meal. A. who B. whose C. where D. which Question 12: We need English local guides _______accompany foreign visitors on trips throughout Vietnam. A. into B. in C. of D. to Question 13: By next week, they ____________ resurfacing the road. A. will have finished B. will finish C. will be finishing D. finish Question 14: According to the Red Cross 1998 had been _______ worst year for natural disasters in modern times. A. a B. an C. the D. Ø. Question 15: I __________ here for hours and I feel tired. A. stood B. have been stood C. have been standing Question 16: Jane said she _________ 20 the following week. A. will be B. would be C. was. D. am standing D. is Trang 1/6.

<span class='text_page_counter'>(37)</span> VnDoc - Tải tài liệu, văn bản pháp luật, biểu mẫu miễn phí Question 17: I can’t play _________ piano, but I can play_________ guitar. A. a – the B. the – the C. the – a D. an – a Question 18: The movie has been highly ________ by critics. A. recommend B. recommendation C. recommended D. recommending nd Question 19: The 22 SEA Games held in Vietnam with its spirit: solidarity, __________ and development. A. co-operate B. co-operation C. operate D. operation Mark your letter A, B, C or D on your answer sheet to indicate the most suitable response to complete each of the following exchanges. Question 20: Two friends, Tom and John are talking. Tom: Have you ever been active in music ? John: “________” A. Yes, I used to listen to it B. Yes, I liked listening to it C. Yes, I bought an electric gutar in the past D. Yes, I played electric gutar in the past Question 21: Ann and John are talking about weekend plan. Ann : _______ go to the theatre tonight? I hear there is a live show. John: I’d like to, but I need to visit my grandparents. A. Why don’t we B. Why don’t I C. How can we What do we Mark the letter A, B, C, or D on your answer sheet to indicate the word(s) CLOSEST in meaning to the underlined word(s) in each of the following questions. Question 22: You should find out as much as possble about the job and the vacancy. A. a part of the newspaper where jobs are advertised B. a job that is available C. a part of the TV programme where the jobs are advertised D. a seat that is available Question 23: If you have your keenness at work, everyone will except you. A. special interest B. very quick understanding C. sence of responsility D. unamiability Mark the letter A, B, C, or D on your answer sheet to indicate the word(s) OPPOSITE in meaning to the underlined word(s) in each of the following questions. Question 24: Her pictures had influence on his life. A. missed B. were fair C. lost D. discussed Question 25: What out, Lan. It is very dangerous. Don’t tought it. A. safe B. not very safe C. easy to be hurt D. unsafe Choose the best sentence that can be made from the words given. Question 26: Since/ beginning/ course/ I/ never/ be late/ class/ A. Since the beginning of the course I had never .been late for class. B. Since the beginning of the course I never have been late for class. C. Since the beginning of the course I have never been late for class. D. Since beginning the course I had never been late for class. Question 27: Be/ doctor/ she/ know/ what/ side-effects/ medicine/ have/ A. Be a doctor, she knows what side-effects the medicine can have. B. Being a doctor, she knew what side-effects the medicine could have. C. Having been a doctor, she had known what side-effects the medicine could have. Trang 2/6.

<span class='text_page_counter'>(38)</span> VnDoc - Tải tài liệu, văn bản pháp luật, biểu mẫu miễn phí D. To be a doctor, she knows what side-effects the medicine might have. Question 28: young people/ fed/ queue/ unemployment benefit/ A. The young people fed with queuing for their unemployment benefit. B. The young are fed with queuing for their unemployment benefit. C. Young people have fed up with queuing for their unemployment benefit. D. Young people are fed up with queuing for their unemployment benefit. Mark the letter A, B, C, or D on your answer sheet to indicate the sentences that best combines each pair of the sentences in the following questions. Question 29: People swimming here do so at their own risk. A. People may swim here without fear or risk. B. Swimming is so risky here that it is forbidden. C. People should realize that it is dangerous to swim here. D. Swimmers will not be blamed if anything happens to them here. Question 30: There was very little response to the charity appeal. A. Nobody responded to the charity appeal. B. Very little money was raised by the charity appeal. C. Everybody seemed to appeal to the charity. D. They raised a lot of money from the charity 'appeal. Read the following passage and mark the letter A, B, C, or D on your answer sheet to indicate the correct word or phrase that best fits each of the numbered blanks from 31 to 35. In the 1972 the Australian government (31)______ a quota system which allowed a certain number of kangaroos to be killed or 'culled' every year. Legislation was introduced because farmers claimed that the kangaroos were (32) ______ their crops. The problem is that (33) ______ two and a half million kangaroos can be killed legally each year, a futher two and a half million are killed illegally. The animals are killed for a variety of reasons. The main one, however, is that kangaroo meat is sold for human (34) ______ usually in the form of steaks - or is used as pet food. There are also thousands of dollars to be made (35) ______ the sale of their skins. Question 31: A. introduced B. produced C. turned out D. offered Question 32: A. enriching B. injuring C. ruining D. spoiling Question 33: A. while B. because C. when D. unless Question 34: A. consumption B. need C. employment D. usage Question 35: A. by B. during C. for D. from Read the following passage and mark the letter A, B, C or D on your answer sheet to indicate the correct answer to each of the following Questions from 36 to 42. The Association of Southeast Asian Nations or ASEAN was established on 8 August 1967 in Bangkok by the five original Member Countries, namely, Indonesia, Malaysia, Philippines, Singapore, and Thailand. Brunei joined on 8 January 1984, Vietnam on 28 July 1995, Lao PDR and Myanmar on 23 July 1997, and Cambodia on 30 April 1999. The ASEAN Declaration states that the aims and purposes of the Association are: to accelerate economic growth, social progress and cultural development in the region and to promote regional peace and stability through abiding respect for justice and the rule of law in the relationship among countries in the region and adherence to the principles of the United Nations Charter. In 2003, the ASEAN Leaders resolved that an ASEAN Community shall be established comprising three pillars, namely, ASEAN Security Community, ASEAN Economic Community and ASEAN SocioCultural Community. The ASEAN Vision 2020, adopted by the ASEAN Leaders on the 30th Anniversary of ASEAN, agreed on a shared vision of ASEAN as a concert of Southeast Asian nations, outward looking, living in peace,. Trang 3/6.

<span class='text_page_counter'>(39)</span> VnDoc - Tải tài liệu, văn bản pháp luật, biểu mẫu miễn phí stability and prosperity, bonded together in partnership in dynamic development and in a community of caring societies. ASEAN Member Countries have adopted the following fundamental principles in their relations with one another, as contained in the Treaty of Amity and Cooperation in Southeast Asia (TAC):  mutual respect for the independence, sovereignty, equality, territorial integrity, and national identity of all nations;  the right of every State to lead its national existence free from external interference, subversion or coercion;  non-interference in the internal affairs of one another;  settlement of differences or disputes by peaceful manner;  renunciation of the threat or use of force; and  effective cooperation among themselves. Question 36: The word “established” is the same meaning with the word _______. A. formed B. destroyed C. restored D. managed Question 37: According to the text, which nation is the last member to join The Association of Southeast Asian Nations up to now. A. Vietnam B. The Philippines C. Brunei D. Cambodia Question 38: The ASEAN Declaration is about _______. A. the cultures of all the members of the organization B. the development of all countries allover the world C. the aims and purposes of the organization D. the laws of the members of the organization Question 39: The third paragraph is about _______. A. living in peace, stability and prosperity C. The ASEAN Vision 2020 B. the 30th Anniversary of ASEAN D. the ASEAN Leaders Question 40: The Treaty of Amity and Cooperation in Southeast Asia _______. A. is not accepted by ASEAN Member Countries B. consists of fundamental principles in the relations of ASEAN Member Countries C. is disapproved by ASEAN Member Countries D. has two principles Question 41: How many fundamental principles are there? A. four B. five C. six D. seven Question 42: Which principle does not belong to the Treaty of Amity and Cooperation in Southeast Asia? A. interference in the internal affairs of member countries B. effective cooperation C. mutual respect D. peaceful manner Read the passage and mark the letter A, B, C or Don your answer sheet to indicate the correct answer to each of the questions from 43 to 50 On Monday, May 12, 2008 a violent earthquake, measuring 8.0 on the Richter scale, happened in southwestern China's Sichuan Province. Hundreds of aftershocks followed in the area. The earthquake destroyed thousands buildings, roads, schools and hospitals, as well as infrastructure like communication networks and electrical towers. Thousands of people died or are missing, and more than 45 million people were affected by the earthquake, which has been the worst natural disaster to hit China for 30 years. The American Red Cross has contributed $20 million to support the relief and recovery efforts of the Red Cross Society of China. These funds will be used to assist survivors through the purchase and distribution of relief supplies, coordination of logistics and transportation of disaster workers to the hardest hit areas. The American Red Cross has also sent relief experts to the affected area to help monitor and coordinate the response efforts. On June 6, the American Red Cross co-hosted a forum to address the U.S. response and recovery efforts following the earthquake. More than 30 representatives from humanitarian organizations, the business companies and the government participated in the event. Trang 4/6.

<span class='text_page_counter'>(40)</span> VnDoc - Tải tài liệu, văn bản pháp luật, biểu mẫu miễn phí More than 35,000 staff and volunteers with the Red Cross Society of China responded to the disaster by distributing food, water, tents and other essential items. The International Federation of Red Cross and Red Crescent Societies are providing 100,000 tents to help those in need. The Red Cross Society of China is a very strong organization with extensive experience responding to disasters. The American Red Cross has long history of working with the Red Cross Society of China, going to back to famines in 1906 and including severe storms earlier this year. There has been a close contact between the Red Cross Society of China and the International Federation of Red Cross and Red Crescent Societies, arid they have been ready to provide additional support such as disaster workers, relief supplies or financial assistance. Concerned family and friends in the United States may have difficulty contacting their loved ones because telecommunication has been out of work by this disaster. If they are trying to reach relatives living in China or those who are citizens of China, the American Red Cross suggests them keep calling or try contacting other family members who live nearby. Question 43: The word “destroyed” has opposite meaning to the words_______. A. formed B. demolished C. repainted D. provided Question 44 The earthquake on May 12, 2008 in China _______. A. left no aftershocks B. destroyed everything except communication networks C. caused no human loss D. had effects on more than 45 million people Question 45: According to the second paragraph, _______. A. the American Red Cross helped the Red Cross Society of China to recover from the earthquake B. the American Red Cross owed the Red Cross Society of China $20 million C. the victims of the earthquake were not helped to buy necessary things D. disaster workers were not sent to the hardest hit areas because of the aftershocks Question 46: Which sentence is not true? A. There are more than 30 representatives from humanitarian organizations, the business companies and the government in the US helping the Chinese earthquake victims. B. The American government did not do anything to help the earthquake victims in China. C. The American Red Cross has contributed $20 million to help the earthquake victims in China. D. The American Red Cross has also sent relief experts to help the earthquake victims in China. Question 47: The American Red Cross has long history of working with the Red Cross Society of China, going to back to famines ________ A. before 1905 B. in 1906 C. in 1960 C. after 1906 Question 48: Which is not mentioned about the Red Cross Society of China? A. It is a strong organization. B. It gets on well with the American Red Cross. C. It supplied the earthquake victims a lot of things except tents. D. It has extensive experience responding to disasters. Question 49: After the earthquakes _______. A. it has been difficult to contact with the resident in the ,attacked area B. the American Red Cross advised people not to keep contact with the victims C. contacting with the victims was not a problem D. everything has been in order soon Question 50: What does the American Red Cross suggest? A. people should try to co-oporate B. them keep calling or try contacting other family members who live nearby C. them keep working hard D. it must joined in The End SỞ GD&ĐT YÊN BÁI TRƯỜNG THPT CẢM NHÂN. KỲ THI THPT QUỐC GIA NĂM 2017 Môn TIẾNG ANH Trang 5/6.

<span class='text_page_counter'>(41)</span> VnDoc - Tải tài liệu, văn bản pháp luật, biểu mẫu miễn phí Thời gian làm bài: 60 phút, không kể thời gian phát đề ----------------------------------------------------------------. ĐỀ DỰ PHÒNG. ĐÁP ÁN Câu Đáp án Câu Đáp án Câu Đáp án Câu Đáp án Câu Đáp án. 1 C 11 D 21 A 31 A 41 C. 2 A 12 D 22 B 32 C 42 A. 3 A 13 A 23 A 33 A 43 A. 4 B 14 C 24 B 34 A 44 D. 5 B 15 C 25 A 35 D 45 A. 6 B 16 B 26 C 36 A 46 B. 7 B 17 B 27 B 37 C 47 B. 8 B 18 C 28 D 38 D 48 C. 9 A 19 B 29 C 39 C 49 A. 10 B 20 D 30 B 40 B 50 B. Trang 6/6.

<span class='text_page_counter'>(42)</span> VnDoc - Tải tài liệu, văn bản pháp luật, biểu mẫu miễn phí. SỞ GD  ĐT TỈNH THANH HÓA THPT CHUYÊN LAM SƠN (Đề thi có 50 câu / 5 trang). ĐỀ THI THỬ LẦN 1 THPT QUỐC GIA NĂM 2017 Môn: Tiếng Anh Thời gian làm bài: 60 phút, không kể thời gian phát đề Mã đề: 951. Read the following passage and mark the letter A, B, C or D on your answer sheet to indicate the correct word or phrase that best fits each of the numbered blanks from 1 to 5. The warming of the Earth is caused by exhaust gas from automobile engines, fatories and power (1)_________. Carbon dioxide goes up into the atmosphere, and it form a kind of screen that keeps or allows the sunshine in but stop the Earth heart (2) _________getting out. It works like a greenhouse, that’s why we call (3) _________the Green House effect. Because of this effect, the Earth is getting warmer all the time. This (4) _________in temperature will cause big changes to the world’s climate. The sea level will increase as the ice (5) _________the poles will melt. Question 1: A. companies B. factories C. sites D. stations Question 2: A. from B. up C. against D. away Question 3: A. is B. be C. it D. them Question 4: A. raise B. rise C. drop D. fall Question 5: A. covering B. covers C. covered D. cover Mark the letter A, B, C or D on your answer sheet to indicate the word(s) CLOSEST in meaning to the underlined word(s) in each of the following question. Question 6: It was relatively easy for him to learn baseball because he had been a cricket player. A. approximately B. comparatively C. nearly D. essentially Question 7: You have to be on your toes if you want to beat her. A. pay all your attention to what you are doing B. upset her in what she is doing C. get involved in what she is doing D. make her comply with your orders Mark the letter A, B, C or D on your answer sheet to indicate the word whose underlined part differs from the other three in pronunciation in each of the following question. Question 8: A. accurate B. account C. accept D. accuse Question 9: A. talks B. takes C. decides D. completes Mark the letter A, B, C or D on your answer sheet to indicate the word having different stress from the others in each of the following questions. Question 10: A. rapidly B. comfortably C. fluently D. necessarily Question 11: A. politician B. genetics C. artificial D. controversial Mark the letter A, B, C or D on your answer sheet to indicate the underlined part that needs correction in each of the following questions Question 12: Foreign students who are (A)doing a decision (B) about which school (C) to attend may not know exactly where (D) the choices are located. Trang 1.

<span class='text_page_counter'>(43)</span> VnDoc - Tải tài liệu, văn bản pháp luật, biểu mẫu miễn phí. Question 13: (A) It is a good idea (B) to be careful in buying (C) or purchasing magazines (D) from salespersons who may come to your door. Question 14: In (A) that age of computers, (B) it is difficult to imagine how tedious (C) work of accountants and clerks must (D) have been in the past. Mark the letter A, B, C or D on your answer sheet to indicate the correct answer to each of the following questions. Question 15: Since he failed his exam, he had to_________for it again. A. pass B. make C. take D. sit Question 16: Is there_________at all I can help? A. everything B. anything C. something D. one thing Question 17: Don’t worry. He’ll do the job as_________as possible. A. economizing B. econimic C. economical D. economically Question 18: Only when you grow up, _________the truth. A. you will know B. you know C. do you know D. will you know Question 19: My brother left his job last week because he did not have any_________to travel. A. position B. chance C. ability D. location Question 20: John paid $2 for his meal, _________he had thought it would cost. A. not as much B. not so much as C. less as D. not so many as Question 21: It is very important for a film or a company to keep_________the changes in the market. A. pace of B. track about C. touch with D. up with Question 22: I’m sure you’ll have no_________the exam. A. difficulty passing B. difficulties to pass C. difficulty to pass D. difficulties of passing Question 23: I_________this letter around for days without looking at it. A. carry B. must carry C. have been carrying D. am carrying Question 24: Vietnam’s rice export this year will decrease_________about 10%, compared with that of last year. A. with B. at C. by D. on Question 25: I won’t change my mind_________what you say. A. whether B. no matter C. because D. although Question 26: My car isn’t_________. It’s always letting me down. A. believable B. reliable C. colorable D. conceivable Mark the letter A, B, C or D on your answer sheet to indicate the most suitable response to complete each of the following exchanges. Question 27: - “ Has an annoucement been made about the eight o’clock flight to Paris?” -“ .” A. Not yet B. Yes, it was C. I don’t think that D. Sorry, I don’t Question 28: - “ I can speak English well enough to apply for that post.” -“ .” A. Me neither B. Me too C. Me either D. Me also Mark the letter A, B, C or D on your answer sheet to indicate the word(s) OPPOSITE in the meaning to the underlined word(s) in ecah of the following questions. Question 29: I’m sorry to say that it seems you’ve been shirking your responsibilities. A. going along B. standing up to C. taking on D. refraining from Question 30: She gave an impeccable reading of the sonata and had the audience on their feet. A. suspicious B. unqualified C. negative D. imperfect Trang 2.

<span class='text_page_counter'>(44)</span> VnDoc - Tải tài liệu, văn bản pháp luật, biểu mẫu miễn phí. Mark the letter A, B, C or D on your answer sheet to indicate the sentence that is closest inmeaning to each of the following question. Question 31: I got the impression that the boys were not enjoying themselves. A. The boys impressed me that they were not enjoying themselves. B. The boys didn’t seem to be enjoying themselves. C. The boys seemed not to enjoying themselves. D. It seemed to me the boys were enjoying themselves. Question 32: The children ran to the field to see the dragon, but it was no longer there. A. The dragon had gone by the time the children ran to the filed. B. The dragon went by the time the children ran to the filed. C. When the children ran to the filed, the dragon went. D. The dragon had gone after the children ran to the filed. Question 33: The boy became so confused and worried that he left home. A. The boy became too confused and worried to leave home. B. So confued and worried that the boy left home. C. So confused and worried, the boy left home. D. The boy was not confused and worried enough to leave home. Mark the letter A, B, C or D on your answer sheet to indicate the sentence that best combines each pair of sentences in the following questions. Question 34: Sue lives in a house. The house is opposite my house. A. Sue lives in a house where is opposite my house. B. Sue lives in a house which is opposite my house. C. Sue lives in a house who is opposite my house. D. Sue lives in a house and which is opposite my house. Question 35: She turned the radio on at 7.30. She was still listening to it when her mother came home at 9.00. A. She has been listening to the radio at 7.30. B. She had been listening to the radio since 7.30. C. She has been listening to the radio after 7.30. D. She had been listening to the radio by 7.30. Read the following passage and mark the letters A, B, C or D on your answer sheet to indicate the correct answer to each of the questions Scientists do not yet thoroughly understand just how the body of an individual becomes sensitive to a substance that is harmless or even wholesome for the average person. Milk, wheat, and egg, for example, rank among the most healthful and widely used foods. Yet these foods can cause persons sensitive to them to suffer greatly. At first, the body of the individual is not harmed by coming into contact with the substance. After a varying interval of time, usually longer than a few weeks, the body becomes sensitive to it, and an allergy has begun to develop. Sometimes it's hard to figure out if you have a food allergy, since it can show up so many different ways. Your symptoms could be caused by many other problems. You may have rashes, hives, joint pains mimicking arthritis, headaches, irritability, or depression. The most common food allergies are to milk, eggs, seafood, wheat, nuts, seeds, chocolate, oranges, and tomatoes. Many of these allergies will not develop if these foods are not fed to an infant until her or his intestines mature at around seven months. Breast milk also tends to be protective. Migraines can be set off by foods containing tyramine, phenathylamine, monosodium glutamate, or sodium nitrate. Common Trang 3.

<span class='text_page_counter'>(45)</span> VnDoc - Tải tài liệu, văn bản pháp luật, biểu mẫu miễn phí. foods which contain these are chocolate, aged cheeses, sour cream, red wine, pickled herring, chicken livers, avocados, ripe bananas, cured meats, many Oriental and prepared foods (read the labels!). Some people have been successful in treating their migraines with supplements of B-vitamins, particularly B6 and niacin. Children who are hyperactive may benefit from eliminating food additives, especially colorings, and foods high in salicylates from their diets. Question 36: The topic of this passage is______________. A. reactions to foods B. food and nutrition C. infants and allergies D. a good diet Question 37: According to the passage, the difficulty in diagnosing allergies to foods is due to ______________. A. the vast number of different foods we eat B. lack of a proper treatment plan C. the similarity of symptoms of the allergy to other problems D. the use of prepared formula to feed babies Question 38: The word "symptoms" is closest in meaning to______________. A. indications B. diet C. diagnosis D. prescriptions Question 39: The phrase "set off" is closest in meaning to______________. A. relieved B. identified C. avoided D. triggered Question 40: What can be inferred about babies from this passage? A. They can eat almost anything. B. They should have a carefully restricted diet as infants. C. They gain little benefit from being breast fed. D. They may become hyperactive if fed solid food too early. Question 41: The word "hyperactive" is closest in meaning to______________. A. overly active B. unusually low activity C. excited D. inquisitive Question 42: The author states that the reason that infants need to avoid certain foods related to allergies has to do with the infant's______________. A. lack of teeth B. poor metabolism C. underdeveloped intestinal tract D. inability to swallow solid foods Read the following passage and mark the letters A, B, C or D on your answer sheet to indicate the correct answer to each of the questions The rapid transcontinentat settlement and these new urban industrial circumstances of the last half of the 19th century were accompanied by the development of national literature of great abundance and variety. New themes, new forms, new subjects, new regions, new authors, new audiences all emerged in the literature of this half century. As a result, at the onset of World War I, the spirit and substance of American literature had evolved remarkably, just as its center of production had shifted from Boston to New York in the late 1880s and the sources of its energy to Chicago and the Midwest. No longer was it produced, at least in its popular forms, in the main by solemn, typically moralistic men from New England and the Old South; no longer were polite, well-dressed, grammatically correct, middle-class young people the only central characters in its narratives; no longer were these narratives to be set in exotic places and remote times; no longer, indeed, were fiction, poetry, drama, and formal history the chief acceptable forms of literary expression; no longer, finally, was literature read primarily by young, middle class women. In sum, American literature in these years fulfilled in considerable measure the condition Walt Whitman called for in 1867 in describing Leaves of Grass: it treats, he said of his own major work, each state and region as peers "and expands from them, and includes the world ... connecting an American citizen with the citizens of all nations." Trang 4.

<span class='text_page_counter'>(46)</span> VnDoc - Tải tài liệu, văn bản pháp luật, biểu mẫu miễn phí. Question 43: The main idea of this passage is________. A. that the new American literature was less provincial than the old B. that World War I caused a dramatic change in America C. that centers of culture shifted from East to West D. that most people were wary of the new literature Question 44: It can be inferred from the passage that the previous passage probably discussed________. A. the importance of tradition to writers B. new developments in industrialization and population shifts C. the fashions and values of 19th century America D. the limitations of American literature to this time Question 45: The word “evolved” in paragraph two is closest in meaning to________. A. became famous B. turned back C. diminished D. changed Question 46: The word “it” in the second paragraph refers to________. A. the population B. the energy C. American literature D. the manufacturing Question 47: The word “exotic" in paragraph two is closest in meaning to________. A. urban B. unusual C. well-known D. old-fashioned Question 48: The author uses the word “indeed” in the second paragraph for what purpose? A. To emphasize the contrast he is making. B. For variety in a lengthy paragraph. C. To wind down his argument. D. To show a favorable attitude to these forms of literature. Question 49: The phrase “these years” in the third paragraph refers to________. A. 1850-1900 B. the 1900s C. the early 1800s D. the present Question 50: It can be inferred from the passage that Walt Whitman________. A. disliked urban life B. was disapproving of the new literature C. wrote Leaves of Grass D. was an international diplomat. ĐÁP ÁN: 1.D 2.A 3.C 4.B 5.A 6.B 7.A 8.A 9.C 10.D. 11.B 12.A 13.C 14.A 15.D 16.B 17.D 18.D 19.B 20.B. 21.D 22.A 23.C 24.C 25.B 26.B 27.A 28.B 29.A 30.D. 31.B 32.A 33.C 34.B 35.B 36.A 37.C 38.A 39.D 40.B. 41.A 42.C 43.A 44.B 45.D 46.C 47.B 48.A 49.A 50.C. LỜI GIẢI CHI TIẾT: 1D Power station: trạm năng lượng Company: công ty Factory: nhà máy Trang 5.

<span class='text_page_counter'>(47)</span> VnDoc - Tải tài liệu, văn bản pháp luật, biểu mẫu miễn phí. Site: địa điểm Dịch: Sự ấm lên của trái đất là do khí thải từ động cơ ô tô, các nhà máy và trạm năng lượng 2A Stop from doing st: ngừng, ngăn chặn khỏi việc gì Dịch câu: Carbon dioxide đi lên vào bầu khí quyển, và nó tạo thành một loại màn mà giữ hoặc cho phép ánh nắng mặt trời vào nhưng không cho trung tâm Trái Đất đi ra. 3C Vế phía trước có dùng chủ ngữ it, vì thế ở đây tiếp tục dùng tân ngữ it (Vì cả câu này là nói về “it”) Dịch: Nó hoạt động như một nhà kính, đó là lý do tại sao chúng ta gọi nó là hiệu ứng nhà kính 4B Rise: (n): sự tăng lên Ở đây không dùng raise vì sau raise bắt buộc phải có một tân ngữ Dịch câu: sự tăng nhiệt độ này sẽ gây ra những thay đổi lớn đến khí hậu của thế giới. 5A Covering: (đang) bao phủ Dịch câu: Mực nước biển sẽ tăng lên khi lớp băng bao phủ các cực sẽ tan chảy 6B Relatively ~ comparatively: tương đối, khá Approximately: xấp xỉ Nearly: gần như Essentially: cần thiết, thiết yếu Dịch: Nó tương đối dễ dàng cho anh ta để học bóng chày vì anh là một cầu thủ cricket. 7A be on your toes ~ pay all your attention to what you are doing: chú ý vào việc bạn đang làm Dịch câu: Bạn phải tập trung vào nếu như muốn thắng cô ấy 8A accurate /'ækjərət/ account /ə'kaʊnt/ accept /ək'sept/ accuse /ə'kju:z/ 9C C được đọc là /z/, các trường hợp còn lại đọc là /s/ Khi thêm „s‟, nó được đọc là /s/ khi các từ có âm kết thúc là /k/, /p/, /t/ Các trường hợp còn lại được đọc là /z/ 10 D Phần D trọng âm rơi vào âm tiết thứ 3, còn lại là thứ nhất rapidly /'ræpidli/ comfortably /'kʌmfətəbli/ fluently /'flu:əntli/ Trang 6.

<span class='text_page_counter'>(48)</span> VnDoc - Tải tài liệu, văn bản pháp luật, biểu mẫu miễn phí. necessarily /,nesə'serəli/ 11 B Phần B trọng âm rơi vào âm tiết thứ 2, còn lại là thứ 3 politician /,pəli'ti∫n/ genetics /dʒi'netiks/ artificial /,ɑ:ti'fi∫l/ controversial /,kɒntrə'vɜ:∫əl/ 12 A “doing” => “making” Cụm: make a decision = decide: quyết định Dịch câu: Những sinh viên nước ngoài, người đang quyết định nhập học trường nào, có thể không biết chính xác những lựa chọn (địa điểm) đó ở đâu 13 C Bỏ C Buy và purchase đều có nghĩa là mua, vì thế ở đây dùng cả 2 từ là không cần thiết Dịch: Nó là một ý tưởng hay trong việc thận trọng khi mua tạp chí từ nhân viên bán hàng – người có thể đến trước nhà bạn 14 A “that age” => “the age” Trong này không nhắc đến age (thời đại) nào, vì thế không thể dùng that age (thời đại đó) được Dịch: Trong thời đại của máy tính, rất khó để tưởng tượng công việc của kế toán viên và thư ký trong quá khứ tẻ nhạt như thế nào 15 D Sit và take đều có thể đi với exam nhưng điểm khác biệt là sit có thể dùng với for, trong khi take thì không thể Như vậy, mấu chốt của câu này nằm ở chỗ giới từ for, chính chữ for này đã loại take 16 B Anything (at all) được dùng trong câu hỏi Trong câu hỏi đôi khi có thể dùng something, tuy nhiên phía sau có “at all” nên không thể dùng something ở đây Everything và one thing ở đây không phù hợp vê nghĩa 17 D Ở đây ta cần một trạng từ để bổ sung ý nghĩa cho động từ do Chỉ có đáp án D là trạng từ Economically: (một cách) kinh tế (hợp lý về kinh tế) 18 D Đây là câu đảo ngữ với only when Cấu trúc: only when + mệnh đề bình thường + mệnh đề đảo ngữ Ở đây, vế sau (vế đảo ngữ) rõ ràng là thì tương lai (vì vế trước nói: only when you grow up: chỉ khi bạn lớn lên) => chúng ta phải sử dụng will you know (đảo will lên trước chủ ngữ) Trang 7.

<span class='text_page_counter'>(49)</span> VnDoc - Tải tài liệu, văn bản pháp luật, biểu mẫu miễn phí. 19 B Have (stand) a chance to do st: có cơ hội làm gì Dịch câu: Anh trai tôi đã bỏ công việc của mình vào tuần trước vì anh không có bất kỳ cơ hội để đi du lịch. 20 B $2 là danh từ không đếm được => không thể dùng many, D loại C không phù hợp về nghĩa A không chính xác bởi thiếu “as”, đáp án A sẽ đúng nếu như là “not as much as” Đáp án là B: not so much as: không nhiều như… 21 D Keep up with st: bắt kịp, theo kịp với cái gì Dịch: Nó là rất quan trọng đối với một bộ phim hay một công ty để theo kịp với những thay đổi trên thị trường. 22 A Cấu trúc have (no) difficulty (in) doing st: có (không có) khó khăn (trong việc) làm gì Dịch câu: Tớ chắc rằng cậu sẽ không có khó khăn gì trong việc vượt qua kì thi 23 C Ở đây có cụm trạng ngữ chỉ thời gian: for days => dùng thì hoàn thành hoặc thì hoàn thành tiếp diễn Đáp án là C (thì hiện tại hoàn thành tiếp diễn) Dịch: Tôi đã mang lá thư này nhiều ngày mà không nhìn tới nó 24 C Cụm decrease/increase by + …%: giảm/tăng …% (so với trước) Dịch: xuất khẩu gạo của Việt Nam trong năm nay sẽ giảm khoảng 10%, so với cùng kỳ năm ngoái. 25 B No matter st, who, what…: bất kỳ cái gì, ai, cái gì… Dịch: tôi sẽ không thay đổi quyết định dù bạn nói gì đi nữa 26 B đáng tin cậy Believable: có thể tin được Conceivable: có thể quan niệm được, có thể tưởng tượng được Không có từ colorable Dịch: Xe của tôi không đáng tin cậy chút nào. Nó luôn luôn làm tôi thất vọng 27 A Dịch câu: - Đã có thông báo về chuyến bay 8h đến Paris chưa? - Chưa (chưa có thông báo) 28 B Dịch câu: Trang 8.

<span class='text_page_counter'>(50)</span> VnDoc - Tải tài liệu, văn bản pháp luật, biểu mẫu miễn phí. - Tớ nói tiếng anh đủ tốt để ứng cử cho vị trí đó - Tớ cũng vậy Neither và either được dùng cho câu phủ định => loại Me also không đúng ngữ pháp 29 A Shirk: co lại, lẩn tránh, trốn tránh Trái nghĩa là go along: tiếp tục, tiến triển Tôi rất tiếc phải nói rằng có vẻ như bạn đã trốn tránh trách nhiệm của mình. 30 D Impeccable: hoàn hảo, không có lỗi Trái nghĩa là imperfect: không hoàn hảo Dịch: Cô ấy có một bài đọc hoàn hảo về Sonata và khiến khán giả đứng dậy (để khen ngợi) 31 B I got the impression that the boys were not enjoying themselves. Tôi có ấn tượng rằng lũ nhóc đang không hề vui thích = The boys didn‟t seem to be enjoying themselves. Lũ nhóc dường như đang không vui thích 32 A The children ran to the field to see the dragon, but it was no longer there. Những đứa trẻ chạy đến hiện trường để xem con rồng, nhưng nó đã không còn ở đó. = The dragon had gone by the time the children ran to the filed. Con rồng đã đi mất trước khi lũ trẻ đến hiện trường Ở đây dùng kết hợp thì quá khứ đơn và quá khứ hoàn thành (diễn tả một hành động xảy ra trước một hành động khác trong quá khứ) 33 C The boy became so confused and worried that he left home. Cậu bé bối rối và lo lắng đến nỗi rời khỏi nhà = So confused and worried, the boy left home. Quá bối rối và lo lắng, cậu bé rời khỏi nhà 34 B Sue lives in a house. The house is opposite my house. Sue ở trong một ngôi nhà. Ngôi nhà đối diện nhà tôi. = Sue lives in a house which is opposite my house. Sue ở trong một ngôi nhà, cái (nhà) đối diện nhà tôi Ở đây ta dùng mệnh đề quan hệ which để thay thế cho a house 35 B Ở đây đang dùng thì quá khứ => thì hiện tại ở đáp án A và C loại 7.30 là một thời điểm trong quá khứ, một mốc thời gian trong quá khứ => sử dụng since Đáp án là B: Cô ấy đã nghe đài suốt từ 7.30 36 A. Trang 9.

<span class='text_page_counter'>(51)</span> VnDoc - Tải tài liệu, văn bản pháp luật, biểu mẫu miễn phí. Chủ đề của bài viết là: những phản ứng với thức ăn. Đọc bài,sẽ dễ dàng thấy những supporting idea cho topic này. - Milk, wheat, and egg, for example, rank among the most healthful and widely used foods. Yet these foods can cause persons sensitive to them to suffer greatly. - Your symptoms could be caused by many other problems. You may have rashes, hives, joint pains mimicking arthritis, headaches, irritability, or depression. - Migraines can be set off by foods containing tyramine, phenathylamine, monosodium glutamate, or sodium nitrate. 37 C Thông tin ở 2 câu trong bài: Sometimes it's hard to figure out if you have a food allergy, since it can show up so many different ways. Your symptoms could be caused by many other problems. Đôi khi thật khó để tìm ra nếu bạn bị dị ứng thực phẩm, vì nó có thể biểu hiện lên rất nhiều cách khác nhau. triệu chứng của bạn có thể bị gây ra bởi nhiều vấn đề khác. 38 A symptoms = indications : dấu hiệu, biểu hiện, triệu chứng Các từ còn lại : diet : chế độ ăn kiêng ; diagnosis : sự chuẩn đoán ; prescription : đơn thuốc Your symptoms could be caused by many other problems. triệu chứng của bạn có thể bị gây ra bởi nhiều vấn đề khác. 39 D triggered = set off: được gây ra Các động từ còn lại: relieved: được làm yên lòng; identified: được xác định; avoided: được tránh Migraines can be set off by foods containing tyramine, phenathylamine, monosodium glutamate, or sodium nitrate. Đau nửa đầu có thể bị gây ra bởi các loại thực phẩm có chứa tyramine, phenathylamine, bột ngọt, hoặc natri nitrat. 40 B Thông tin ở câu cuối: Children who are hyperactive may benefit from eliminating food additives, especially colorings, and foods high in salicylates from their diets. Những trẻ quá hiếu động có thể được hưởng lợi từ việc loại bỏ các chất phụ gia thực phẩm, đặc biệt là chất tạo màu, và các thực phẩm giàu salicylate từ chế độ ăn uống của chúng. => They should have a carefully restricted diet as infants. Chúng nên có một chế độ ăn uống hạn chế một cách cẩn thận như trẻ sơ sinh. 41 A Hyperactive = overly active : hiếu động thái quá Children who are hyperactive may benefit from eliminating food additives, especially colorings, and foods high in salicylates from their diets. Những trẻ quá hiếu động có thể được hưởng lợi từ việc loại bỏ các chất phụ gia thực phẩm, đặc biệt là chất tạo màu, và các thực phẩm giàu salicylate từ chế độ ăn uống của chúng. 42 C Thông tin trong bài: Many of these allergies will not develop if these foods are not fed to an infant until her or his intestines mature at around seven months Nhiều loại dị ứng sẽ không phát triển nếu những thực phẩm này không được làm thức ăn cho trẻ sơ sinh cho đến khi đường ruột của bé trưởng thành vào khoảng bảy tháng Trang 10.

<span class='text_page_counter'>(52)</span> VnDoc - Tải tài liệu, văn bản pháp luật, biểu mẫu miễn phí. 43 A Ý chính của bài là : the new American literature was less provincial than the old Văn học Mỹ mới có tính lẻ tẻ hơn văn học cũ Văn chương, ít ra là các thể loại chính, không còn chủ yếu được viết bởi những người nghiêm trang, đạo đức tiêu biểu từ Tân Anh Cát Lợi và miền Nam cũ; không còn việc những người trẻ lịch sự, ăn mặc đẹp, chú trọng ngữ pháp, thuộc tầng lớp trung lưu là những nhân vật trung tâm trong tác phẩm; không còn việc các tác phẩm được đặt ở những địa điểm và thời gian xa xăm; không còn việc tiểu thuyết, thơ, kịch, lịch sử chính quy là các thể loại được chấp nhận khi thể hiện văn chương; cuối cùng, không còn việc văn chương chủ yếu chỉ được những phụ nữ trẻ trung lưu đọc. => Văn chương mới của Mỹ có tính lẻ tẻ hơn văn học cũ 44 B Câu đầu tiên của bài đọc là: The rapid transcontinentat settlement and these new urban industrial circumstances of the last half of the 19th century were accompanied by the development of national literature of great abundance and variety. Việc định cư xuyên lục địa nhanh chóng và những trường hợp công nghiệp đô thị mới của nửa cuối của thế kỷ 19 đã được đi kèm với sự phát triển của văn học dân tộc một cách phong phú và đa dạng. Vì thế đoạn văn ở phía trước bài đọc này có thể nói về: new developments in industrialization and population shifts (phát triển mới trong công nghiệp và chuyển dịch dân cư) 45 D “evolved” ~ changed: thay đổi, tiến triển As a result, at the onset of World War I, the spirit and substance of American literature had evolved remarkably: Kết quả là, vào lúc bắt đầu của Thế chiến I, tinh thần và vật chất của văn học Mỹ đã phát triển đáng kể 46 C “it” được thay thế cho American literature trong câu trước: As a result, at the onset of World War I, the spirit and substance of American literature had evolved remarkably… 47 B “exotic" ~ unusual: kỳ lạ, khác thường no longer were these narratives to be set in exotic places and remote times: những câu chuyện không còn bị đặt ở những nơi kỳ lạ và thời gian xa xăm 48 A Tác giả sử dụng “indeed” để To emphasize the contrast he is making. (Để nhấn mạnh sự tương phản ông đang làm.) No longer was it produced, at least in its popular forms;…; no longer, indeed, were fiction, poetry, drama, and formal history the chief acceptable forms of literary expression Nó không còn được sản xuất, ít nhất là trong các hình thức phổ biến của nó;…; thực sự, tiểu thuyết, thơ, kịch, và lịch sử chính thống không còn được chấp nhận là các hình thức chính của biểu đạt văn chương 49 A The rapid transcontinentat settlement and these new urban industrial circumstances of the last half of the 19th century were accompanied by the development of national literature of great abundance and variety. New themes, new forms, new subjects, new regions, new authors, new audiences all emerged in the literature of this half century. Trang 11.

<span class='text_page_counter'>(53)</span> VnDoc - Tải tài liệu, văn bản pháp luật, biểu mẫu miễn phí. “these years” ở đây chính là đề cập đến the last half of the 19th century (nửa cuối thế kỉ 19) => từ năm 1850-1900 50 C Thông tin ở câu đầu đoạn cuối: In sum, American literature in these years fulfilled in considerable measure the condition Walt Whitman called for in 1867 in describing Leaves of Grass Tóm lại, văn học Mỹ trong những năm này đã đáp ứng, xét theo một tiêu chuẩn thích hợp, được điều kiện mà Walt Whitman yêu cầu vào năm 1867 khi mô tả tác phẩm Leaves of Grass => Walt Whitman đã viết Leaves of Grass. Trang 12.

<span class='text_page_counter'>(54)</span> Luy n thi ĐH ti ng Anh cùng cô Hoa. SỞ GD-ĐT THỪA THIÊN HUẾ TRƯỜNG THPT CHUYÊN QUỐC HỌC ĐỀ CHÍNH THỨC (Đề thi có 06 trang). ĐỀ THI THỬ KỲ THI THPT QUỐC GIA LẦN I NĂM HỌC: 2016-2017 Môn: TIẾNG ANH Thời gian làm bài: 60 phút, không kể thời gian giao đề. Mã đề thi 405 (166572) Read the following passage and mark the letter A,B,C or D on your answer sheet to indicate the correct word or phrase that best fits each of the numbered blanks. A trend that has emerged recently is the sharing of childcare (1) __________ between husband and wife. Young couples will try to arrange their work schedules so that they work opposite hours or shifts in order that one parent is always home with the children. Since childcare is expensive, this saves money for the young couple trying to establish themselves and provide a secure environment for the family. Husband and wife may also share household chores. Some fathers are just as capable as mothers at cooking dinner, changing and bathing the baby, and doing the laundry. In some cases, the woman’s salary is for family (2) __________ and the father becomes the “househusband." These cases are still fairly rare. One positive trend, however, is that fathers seem to be spending more time with their children. In a recent survey, 41% of the children sampled said they spend equal time with their mothers and fathers. “This is one of our most significant cultural changes,” says Dr. Leon Hoffman, who co-directs the Parent Child Center at the New York Psychoanalytic Society. In practice, for over 30 years, Hoffman has found "a very dramatic difference in the involvement of the father in everything from care-taking to general decision (3) __________ around kids' lives.” Another factor has recently been added to the childcare formula. The number of people who work from home nearly full-time rose 23% from the last decade. The (4) _________ of technology - computers, faxes, teleconferencing - has made it easier for at-home workers to be constantly in touch. Will this new flexibility in the workforce bring a positive change for the (5) __________ of children? Only time will tell. Question 1: A. abilities. B. possibilities. C. techniques. D. responsibilities. Question 2: A. payment. B. expenses. C. fares. D. fees. Question 3: A. making. B. creating. C. holding. D. giving. Question 4: A. accessible. B. accessibly. C. access. D. accessibility. Question 5: A. well-being. B. security. C. comfort. D. interests. (166578) Mark the letter A,B,C or D on your answer sheet to indicate the word(s) OPPOSITE in meaning to the underlined word(s) in each of the following questions. Question 6: My first impression of her was her impassive face..

<span class='text_page_counter'>(55)</span> Luy n thi ĐH ti ng Anh cùng cô Hoa A. respectful. B. emotional. C. solid. D. fractious. Question 7: We left New York when I was six, so my recollections of it are rather faint. A. clear. B. unintelligible. C. explicable. D. ambiguous. (166581) Mark the letter A,B,C or D on your answer sheet to indicate the sentence that is closest in meaning to each of the following questions. Question 8: Much to my astonishment, I found his lecture on wildlife conservation extremely interesting. A. Contrary to my expectations, his lecture on wildlife conservation was the most fascinating of all. B. I was fascinated by what he said in his lecture on wildlife conservation though I hadn’t expected to be. C. I hadn’t expected him to lecture on wildlife conservation, but he spoke well. D. It was at his lecture on wildlife conservation that I realized I needed to study it. Question 9: Although he was able to do the job, he wasn’t given the position. A. The position wasn’t given to him in spite of his ability to do the job. B. He was given neither the job nor the position. C. Because he couldn’t do the job, he wasn’t given the position. D. He got the position despite being unable to do the job. Question 10: It is certain that the new cuts will worry the staff. A. The new cuts certainly worry the staff. B. The new cuts will be certainly worry the staff. C. The new cuts are bound to worry the staff. D. The new cuts will be bound to worry the staff. (166585) Mark the letter A,B,C or D on your answer sheet to indicate the word that differs from the other three in the preposition of primary stress in each of the following questions. Question 11: A. involve. B. control. C. tonight. D. purpose. Question 12: A. hurricane. B. photograph. C. recommend. D. separate. (166588) Mark the letter A,B,C or D on your answer sheet to indicate the underlined part that needs correction in each of the following questions. Question 13: Digital clocks, however precise, they cannot be perfectly accurate because the earth’s rotation changes slightly over years..

<span class='text_page_counter'>(56)</span> Luy n thi ĐH ti ng Anh cùng cô Hoa Question 14: On the floor of the Pacific Ocean is hundreds of flat-topped mountains more than a mile beneath sea level. Question 15: My sister told me that she had met my teacher at the supermarket yesterday. (166592) Read the following passage and mark the letters A, B, C or D on your answer sheet to indicate the correct answer to each of the questions. Scientists do not yet thoroughly understand just how the body of an individual becomes sensitive to a substance that is harmless or even wholesome for the average person. Milk, wheat, and egg, for example, rank among the most healthful and widely used foods. Yet these foods can cause persons sensitive to them to suffer greatly. At first, the body of the individual is not harmed by coming into contact with the substance. After a varying interval of time, usually longer than a few weeks, the body becomes sensitive to it, and an allergy has begun to develop. Sometimes it's hard to figure out if you have a food allergy, since it can show up so many different ways. Your symptoms could be caused by many other problems. You may have rashes, hives, joint pains mimicking arthritis, headaches, irritability, or depression. The most common food allergies are to milk, eggs, seafood, wheat, nuts, seeds, chocolate, oranges, and tomatoes. Many of these allergies will not develop if these foods are not fed to an infant until her or his intestines mature at around seven months. Breast milk also tends to be protective. Migraines can be set off by foods containing tyramine, phenathylamine, monosodium glutamate, or sodium nitrate. Common foods which contain these are chocolate, aged cheeses, sour cream, red wine, pickled herring, chicken livers, avocados, ripe bananas, cured meats, many Oriental and prepared foods (read the labels!). Some people have been successful in treating their migraines with supplements of B-vitamins, particularly B6 and niacin. Children who are hyperactive may benefit from eliminating food additives, especially colorings, and foods high in salicylates from their diets. A few of these are almonds, green peppers, peaches, tea, grapes. This is the diet made popular by Benjamin Feingold, who has written the book “Why your Child is Hyperactive”. Other researchers have had mixed results when testing whether the diet is effective. Question 16: The topic of this passage is _______________. A. infants and allergies. B. food and nutrition. C. reactions to foods. D. a good diet. Question 17: According to the passage, the difficulty in diagnosing allergies to foods is due to ___________. A. the vast number of different foods we eat B. lack of a proper treatment plan C. the similarity of symptoms of the allergy to other problems D. the use of prepared formula to feed babies Question 18: The word "symptoms" is closest in meaning to ___________. A. prescriptions. B. diet. C. diagnosis. Question 19: What can be inferred about babies from this passage? A. They can eat almost anything. B. They should have a carefully restricted diet as infants. C. They gain little benefit from being breast fed.. D. indications.

<span class='text_page_counter'>(57)</span> Luy n thi ĐH ti ng Anh cùng cô Hoa D. They may become hyperactive if fed solid food too early. Question 20: The author states that the reason that infants need to avoid certain foods related to allergies has to do with the infant's ___________. A. lack of teeth. B. poor metabolism. C. inability to swallow solid foods. D. underdeveloped intestinal tract. Question 21: The word "these" refers to ___________. A. food additives. B. food colorings. C. innutritious foods. D. foods high in salicylates. Question 22: Which of the following was a suggested treatment for migraines in the passage? A. Using Vitamin B in addition to a good diet. B. Avoiding all Oriental foods. C. Getting plenty of sodium nitrate. D. Eating more ripe bananas. Question 23: According to the article the Feingold diet is NOT ___________. A. available in book form B. verified by researchers as being consistently effective C. beneficial for hyperactive children D. designed to eliminate foods containing certain food additives (166601) Mark the letter A,B,C or D on your answer sheet to indicate the word(s) CLOSEST in meaning to the underlined word(s) in each of the following questions. Question 24: I knew he was only flattering me because he wanted to borrow some money. A. teasing. B. threatening. C. praising. D. elevating. Question 25: The kidnapper gave himself up to the authorities. A. confided himself. B. surrendered. C. accommodated himself. D. went up. (166604) Mark the letter A,B,C or D on your answer sheet to indicate the sentence that best combines each pair of sentences in the following questions. Question 26: The demand was so great. They had to reprint the book immediately. A. They demanded that the book be reprinted immediately. B. So great was the demand that they had to reprint the book immediately. C. The book would be reprinted immediately since the demand was great. D. They demanded to reprint the book immediately..

<span class='text_page_counter'>(58)</span> Luy n thi ĐH ti ng Anh cùng cô Hoa Question 27: He did not work hard. He failed the exam. A. Even though he failed the exam, he didn’t work hard. B. Unless he had worked hard, he would have failed the exam. C. If he had worked hard, he would have passed the exam. D. However hard he worked, he failed the exam. (166607) Read the following passage and mark the letter A,B,C or D on your answer sheet to indicate the correct answer to each of the questions Smart cards and mobile phones are becoming an increasingly popular way to make all sorts of payments. Even now, in Japan thousands of transactions, from paying rail tickets to picking up the groceries, take place every day with customers passing their handsets across a small flat-screen device. And predictions in the world of finance reckon that payments using mobile phones will have risen to more than $50 billion in the very near future. What's the appeal of e-cash? Compared to cheques or credit cards, it offers the speed of cash, but more so. It takes just one tenth of a second to complete most transactions and as no change is required, errors in counting are eliminated. Fraud and theft are also reduced and for the retailer, it reduces the cost of handling money. Sony's vision of having a chip embedded in computers. TVs and games consoles means that films, music and games can be paid for easily and without having to input credit card details. And what about the future of the banks? Within their grip on the market, banks and credit-card firms want to be in a position to collect most of the fees from the users of mobile and contactless-payment systems. But the new system could prove to be a "disruptive technology" as far as the banks are concerned. If payments for a few coffees, a train ticket and a newspaper are made every day by a commuter with a mobile, this will not appear on their monthly credit card statements but on their mobile phone statements. And having spent fortunes on branding, credit-card companies and banks do not want to see other payment systems gaining popularity. It's too early to say whether banks will miss out and if so, by how much. However, quite a few American bankers are optimistic They feel there is reason to he suspicious of those who predict that high-street banks may be a thing of the past. They point out that Internet banking did not result in the closure of their high-street branches as was predicted. On the contrary, more Americans than ever are using local branches. So, whether we'll become a totally cash-free society remains open to contention. Question 28: What is the main idea of the first paragraph? A. The absence of traditional payment methods B. Predictions of future payment methods C. Japan's advanced forms of payment D. The increasing popularity of new payment methods Question 29: Why does the author mention "a small flat-screen device" in the first paragraph?.

<span class='text_page_counter'>(59)</span> Luy n thi ĐH ti ng Anh cùng cô Hoa A. to exemplify the e-cash system B. to criticize the e-cash system C. to praise the e-cash system D. to inform the e-cash system Question 30: Which of the following is NOT true about the strong point of e-cash? A. faster speed. B. no fraud. C. fewer mistakes. D. reduced cost. Question 31: The word "embedded” in the second paragraph is closest in meaning to ____________. A. manufactured. B. isolated. C. integrated. D generated. Question 32: The author mentions the case of commuters in the third paragraph to illustrate __________. A. the modern technology of the e-cash system B. a possible drawback of the system C. the banks' cooperation with credit-card companies D. the transferability of the system Question 33: What does the author think may happen in the future? A. Daily expenses on drinks and tickets will appear on phone statements. B. Banks will collect their fees through credit-card companies. C. Americans will no longer go to their local bank branches. D. Credit-card companies and banks will want to promote cash. Question 34: How does the writer seem to feel about the future of banks? A. neutral. B. uncertain. C. optimistic. D. pessimistic. (166615) Mark the letter A,B,C or D on your answer sheet to indicate the most suitable response to complete each of the following exchanges. Question 35: “Can I use your computer?” – “_______.” A. Well done.. B. No, thanks.. C. Of course. D. Yes, I can. Question 36: “I’m sorry I’m late.” – “________.” A. Never mind. Go ahead. B. No worries. Come in, please.. C. Good to hear that.. D. Don’t say so again..

<span class='text_page_counter'>(60)</span> Luy n thi ĐH ti ng Anh cùng cô Hoa (166618) Mark the letter A,B,C or D on your answer sheet to indicate the word whose underlined part different from the other three in pronunciation in each of the following questions. Question 37: A. shrine. B. potential. C. sure. D. question. Question 38: A. compete. B. intend. C. medal. D. defend. (166621) Mark the letter A,B,C or D on your answer sheet to indicate the correct answer to each of the following questions. Question 39: Bob has a bad lung cancer and his doctor advised him to ______ smoking. A. turn up. B. give up. C. take up. D. put up. Question 40: They ______ have seen the play last night as they went to a football match instead. A. could. B. must. C. can’t. D. might. Question 41: We ______ for this opportunity for more than three years. A. were waiting. B. have been waiting C. waited. D. are waiting. Question 42: I’m sorry, but I’ve got ______ much work to do to come to the beach today. A. too. B. such. C. enough. D. so. Question 43: During the week of the national tourism festival, we had visitors ______ from all over the country. A. come. B. came. C. coming. D. to have come. Question 44: Going on this diet has really ______ me good. I’ve lost weight and I feel fantastic! A. done. B. taken. C. made. D. had. Question 45: I have lived near the airport for so long now that I’ve grown ______ to the noise of the airplanes. A. accustomed. B. unconscious. C. familiar. D. aware. Question 46: “Your kitchen is fantastic! Did you do it all by yourself?” – “No, I ______ by a professional.” A. had it designed. B. have it to be designed. C. had designed it. D. designed it. Question 47: I think there’s a picture of the hotel ______the first page. A. in. B. at. C. on. D. to. Question 48: All applicants must ______ their university transcript and two reference letters to be considered for this job. A. permit. B. omit. C. submit. Question 49: We will leave for the airport ______ he is ready.. D. admit.

<span class='text_page_counter'>(61)</span> Luy n thi ĐH ti ng Anh cùng cô Hoa A. while. B. during. C. until. D. as soon as. Question 50: It is time every student ______ harder for the coming exam. A. works. B. should work. C. work. ---THE END---. D. worked.

<span class='text_page_counter'>(62)</span> Luy n thi ĐH ti ng Anh cùng cô Hoa. 1 2 3 4 5 6 7 8 9 10. D B A D A B A B A C. 11 12 13 14 15 16 17 18 19 20. D C A B D C C D B D. 21 22 23 24 25 26 27 28 29 30. D A B C B B C D A B. 31 32 33 34 35 36 37 38 39 40. C B A B C B D A B C. 41 42 43 44 45 46 47 48 49 50. B A C A A A D C D D.

<span class='text_page_counter'>(63)</span> VnDoc - Tải tài liệu, văn bản pháp luật, biểu mẫu miễn phí BỘ GIÁO DỤC VÀ ĐÀ TẠO. KỲ THI TRUNG HỌC PHỔ THÔNG QUỐC GIA NĂM 2017 Môn: TIẾNG ANH. ĐỀ CHÍNH THỨC. Thời gian: 60 phút, không kể thời gian giao đề. (Đề thi gồm 06 trang). Mark the letter A, B, C, or D on your answer sheet to indicate the word that differs from the rest in the position of the main stress in each of the following questions. Question 1. A. included B. wanted C. decided D. noticed Question 2. A. allow B. tomorrow C. slowly D. below Mark A,B,C or D to indicate the word whose main stress differs from the rest. Question 3. A. struggle B. survive C. enlarge D. occur Question 4. A. reliable B. conventional C. preservative D. intellectual Identify the underlined word or phrase that must be changed in order for the sentence to be corrected Question 5: These exercises look easy, but they are very relatively difficult for us. A. These exercises B. easy C. relatively D. for Question 6: As the old one, this new copier can perform its functions in half the time A. As B. can perform C. its functions D. in half the time Question 7: The assumption that smoking has bad effects on our health have been proved. A. that B. effects C. on D. have been proved Mark the letter A, B, C, or D on your answer sheet to indicate the most suitable response to complete each of the following questions Question 8: Kay: “I wouldn’t do that if I were you.” John: “___________” A. Wouldn’t you? Why? B. Would you, really? C. I’d rather you didn’t. D. It’s out of the question. Question 9: Lucy: “You look really nice in that red sweater!” Sue: “___________” A. Don’t mention it. B. How dare you? C. I’m afraid so. D. Thank you. Mark the letter A, B, C or D on your answer sheet to indicate the word(s) CLOSEST in meaning to the underlined word(s) in each of the following questions. Question 10: I hope to have the privilege of working with them again. A. honor B. advantage C. favor D. right Question 11: Everything was in a thorough mess. A. utter B. full C. complete D. appalling Mark the letter A, B, C, or D on your answer sheet to indicate the word or phrase that is OPPOSITE meaning to the underlined part in each of the following questions. Question 12: His extravagant ideas were never brought to fruition. A. impressive B. exaggerated C. unacceptable D. practical Question 13: This shouldn’t be too taxing for you. A. comfortable B. demanding C. easy D. relaxing Mark the letter A, B, C, or D to indicate the correct answer in each of the following questions. Question 14: Deborah is going to take extra lessons to______ what she missed when she was away. A. catch up on B. put up with C. cut down on D. take up with Question 15: I am sorry I have no time at present to_______ detail of our plan. A. bring in B. come in C. take into D. go into Question 16: In spite of her abilities, Laura has been _____ overlooked for promotion. Trang 1/6.

<span class='text_page_counter'>(64)</span> VnDoc - Tải tài liệu, văn bản pháp luật, biểu mẫu miễn phí A. repetitive B. repeatedly C. repetition D. repeat Question 17: The criminal knows the________ of successful robberies. A. trash and treasure B. part and parcel C. ins and outs D. close all Question 18: Don’t________ the kettle; it’s still hot. A. touch B. feel C. look D. taste Question 19: Policemen are sometimes on________ at night. A. force B. alert C. cover D. patrol Question 20: George won five medals at the competition. His parents ___ very proud of him. A. can’t be B. can’t have C. must have been D. could have been Question 21: We bought some________. A. German lovely old glasses C. German old lovely glasses B. lovely old German glasses D. old lovely German glasses Question 22: This is the third time James________ the volunteer program to the village. A. joins B. joined C. has joined D. has been joining Question 23: The higher the content of carbon dioxide in the air is, _________. A. the more heat it retains B. the heat it retains more C. it retains the more heat D. more heat it retains Question 24: The pool should not be made so deep_____ small children can be safe there. A. so as to B. though C. if D. so that Question 25: Standing on the tip of the cape, _________. A. people have seen a lighthouse far away B. a lighthouse can see from the distance C. we can see the lighthouse in the distance D. lies a lighthouse in the middle of the sea Read the following passage and mark the letter A, B, C, or D on your answer sheet to indicate the correct answer to each of the questions. New surveys suggest that the technological tools we use to make our lives easier are killing our leisure time. We are working longer hours, taking fewer and shorter vacations (and when we do go away, we take our cell phones, PDAs, and laptops along). And, we are more stressed than ever as increased use of e-mail, voice mail, cell phones, and the Internet is destroying any idea of privacy and leisure. Since the Industrial Revolution, people have assumed that new labor-saving devices would free them from the burdens of the workplace and give them more time to grow intellectually, creatively, and socially – exploring the arts, keeping up with current events, spending more time with friends and family, and even just ‘goofing off’. But here we are at the start of the 21st century, enjoying one of the greatest technological boom times in human history, and nothing could be further from the truth. The very tools that were supposed to liberate us have bound us to our work and study in ways that were inconceivable just a few years ago. It would seem that technology almost never does what we expect. In ‘the old days’, the lines between work and leisure time were markedly clearer. People left their offices at a predictable time, were often completely disconnected from and out of touch with their jobs as they traveled to and from work, and were off-duty once they were home. That is no longer true. In today’s highly competitive job market, employers demand increased productivity, expecting workers to put in longer hours and to keep in touch almost constantly via fax, cell phones, e-mail, or other communications devices. As a result, employees feel the need to check in on what is going on at the office, even on days off. They feel pressured to work after hours just to catch up on everything they have to do. Workers work harder and longer, change their work tasks more frequently, and have more and more reasons to worry about job security. Bosses, colleagues, family members, lovers, and friends expect instant responses to voice mail and e-mail messages. Even college students have become bound to their desks by an environment in which faculty, friends, and other members of the college community increasingly do their work online. Studies of time spent on instant messaging services would probably show staggering use. Trang 2/6.

<span class='text_page_counter'>(65)</span> VnDoc - Tải tài liệu, văn bản pháp luật, biểu mẫu miễn phí This is not what technology was supposed to be doing for us. New technologies, from genetic research to the Internet, offer all sorts of benefits and opportunities. But, when new tools make life more difficult and stressful rather than easier and more meaningful – and we are, as a society, barely conscious of it – then something has gone seriously awry, both with our expectations for technology and our understanding of how it should benefit us. From “Summit 1” by Joan Saslow & Allen Ascher Question 26: According to the first three paragraphs, technological tools that were designed to make our lives easier__________. A. have brought us complete happiness B. have fully met our expectations C. have not interfered with our privacy D. have turned out to do us more harm than good Question 27: Which of the following is NOT true about technological tools, according to new surveys? A. They make our life more stressful.B. They bring more leisure to our life. C. They are used even during vacations. D. They are being increasingly used. Question 28: The word “inconceivable” in the passage is closest in meaning to”_________”. A. unforgettable B. unimaginable C. predictable D. foreseeable Question 29: It can be inferred from the fourth paragraph that_________. A. it is compulsory that employees go to the office, even on days off B. employees have more freedom to decide what time they start and finish work C. employers are more demanding and have efficient means to monitor employees D. life is more relaxing with cell phones and other technological devices Question 30: The word “They” in the fourth paragraph refers to________. A. employers B. employees C. workers D. tasks Question 31: This passage has probably been taken from_________. A. a science review B. a political journal C. an advertisement D. a fashion magazine Question 32: Which of the following could best serve as the title of the passage? A. Expectations and Plain Reality B. Benefits of Technology C. Research on the Roles of Computers D. Changes at the Workplace Read the following passage and mark the letter A, B, C, or D on your answer sheet to choose the word or phrase that best fits each of the numbered blanks. In “Cerealizing America”, Scott Bruce and Bill Crawford remark that the cereal industry uses 816 million pounds of sugar per year. Americans buy 2.7 billion packages of breakfast cereal each year. If (33)_________ end to end, the empty cereal boxes from one year’s consumption would stretch to the moon and back. One point three (1.3) million advertisements for cereal are broadcast on American television every year at a(n) (34)_________ of $762 million for airtime. Only automobile manufacturers spend more money on television advertising than the makers of breakfast cereal. (35)_________ of the boxed cereals found in supermarkets contain large amounts of sugar and some contain more than 50% sugar. Cereal manufacturers are very clever in their marketing, making many cereals appear much healthier than they really are by “fortifying” them with vitamins and minerals. Oh, lovely - you now have vitamin-fortified sugar! Before you eat any cereal, read the ingredient list and see how (36)_________ sugar appears on the ingredient list. Then check the “Nutrition facts” panel. There are actually only a small handful of national commercially-branded cereals that are made (37)_________ whole grains and are sugar-free. From “Foods That Burn Fat, Foods That Turn to Fat” by Tom Ventulo Question 33:A. to lay Question 34: A. charge Question 35: A. Most Question 36: A. tall. B. laying B. average B. Mostly B. large. C. lay C. cost C. Almost C. high Trang 3/6. D. laid D. expense D. Furthermost D. many.

<span class='text_page_counter'>(66)</span> VnDoc - Tải tài liệu, văn bản pháp luật, biểu mẫu miễn phí Question 37: A. by. B. from. C. at. D. in. Read the following passage and mark the letter A, B, C, or D on your answer sheet to indicate the correct answer to each of the questions. Very few people, groups, or governments oppose globalization in its entirety. Instead, critics of globalization believe aspects of the way globalization operates should be changed. The debate over globalization is about what the best rules are for governing the global economy so that its advantages can grow while its problems can be solved. On one side of this debate are those who stress the benefits of removing barriers to international trade and investment, allowing capital to be allocated more efficiently and giving consumers greater freedom of choice. With free-market globalization, investment funds can move unimpeded from the rich countries to the developing countries. Consumers can benefit from cheaper products because reduced taxes make goods produced at low cost from faraway places cheaper to buy. Producers of goods gain by selling to a wider market. More competition keepssellers on their toes and allows ideas and new technology to spread and benefit others. On the other side of the debate are critics who see neo-liberal policies as producing greater poverty, inequality, social conflict, cultural destruction, and environmental damage. They say that the most developed nations – the United States, Germany, and Japan – succeeded not because of free trade but because of protectionism and subsidies. They argue that the more recently successful economies of South Korea, Taiwan, and China all had strong state-led development strategies that did not follow neoliberalism. These critics think that government encouragement of “infant industries” – that is, industries that are just beginning to develop – enables a country to become internationally competitive. Furthermore, those who criticize the Washington Consensus suggest that the inflow and outflow of money from speculative investors must be limited to prevent bubbles. These bubbles are characterized by the rapid inflow of foreign funds that bid up domestic stock markets and property values. When the economy cannot sustain such expectation, the bubbles burst as investors panic and pull their money out of the country. Protests by what is called the anti-globalization movement are seldom directed against globalization itself but rather against abuses that harm the rights of workers and the environment. The question raised by nongovernmental organizations and protesters at WTO and IMF gatherings is whether globalization will result in a rise of living standards or a race to the bottom as competition takes the form of lowering living standards and undermining environmental regulations. One of the key problems of the 21st century will be determining to what extent markets should be regulated to promote fair competition, honest dealing, and fair distribution of public goods on a global scale.From “Globalization” by Tabb, William K., Microsoft ® Student 2009 [DVD] Question 38:It is stated in the passage that__________. A. the protests of globalization are directed against globalization itself B. the United States, Germany, and Japan succeeded in helping infant industries C. supporters of globalization stress the benefits of removing trade barriers D. critics of globalization say that the successful economies are all in Asia Question 39:Supporters of free-market globalization point out that__________. A. consumers can benefit from cheaper products B.there will be less competition among producers C. taxes that are paid on goods will be increased D. investment will be allocated only to rich countries Question 40: The word “allocated” in the passage mostly means “_________”. A. removed B. solved C. offered D. distributed Question 41: The phrase “keeps sellers on their toes” in the passage mostly means “__________”. A. makes sellers responsive to any changes B. allows sellers to stand on their own feet C. forces sellers to go bare-footed Trang 4/6.

<span class='text_page_counter'>(67)</span> VnDoc - Tải tài liệu, văn bản pháp luật, biểu mẫu miễn phí D. prevents sellers from selling new products Question 42: According to critics of globalization, several developed countries have become rich because of__________. A. their neo-liberal policies B. their help to developing countries C. their prevention of bubbles D. their protectionism and subsidies Question 43: Infant industries mentioned in the passage are__________. A. successful economies B. young companies C. development strategies D. young industries Question 44: Which of the following is NOT mentioned in the passage? A. Critics believe the way globalization operates should be changed. B. The anti-globalization movement was set up to end globalization. C. Some Asian countries had strong state-led economic strategies. D. Hardly anyone disapproves of globalization in its entirety. Question 45: The debate over globalization is about how__________. A. to use neo-liberal policies for the benefit of the rich countries C. to spread ideas and strategies for globalization B. to govern the global economy for the benefit of the community D. to terminate globalization in its entirely Mark the letter A, B, C, or D on your answer sheet to indicate the sentence that is closest in meaning to each of the following questions Question 46: Sam speaks Chinese well and his Japanese is good, too. A. Sam is good at either Chinese or Japanese. B. Not only Chinese but also Japanese Sam is good at. C. Not only does Sam speak Chinese but also Japanese. D. Sam not only speaks Chinese well but also is good at Japanese. Question 47: It’s a bad line. Do you want me to give you a ring later? A. Can I call you later? B. I would like to give you a ring as a present. C. Would you like to become my wife? D. Can I give the ring back to you later? Question 48: Had she read the reference books, she would have been able to finish the test. A. If she had read the reference books, she could finish the test. B. Not having read the reference books, she couldn’t finish the test. C. Although she didn’t read the reference books, she was able to finish the test. D. Because she read the reference books, she was able to finish the test. Mark the letter A, B, C, or D on your answer sheet to indicate the sentence that best combines each pair of sentences in the following questions Question 49: We’d better leave them a note. It’s possible they’ll arrive later. A. If they arrive late, we’d better leave them a note. B. We’d better leave them a note as they possibly arrive later. C. They’ll probably arrive later so that we’d better leave them a note. D. We’d better leave them a note in case they arrive later. Question 50: Women still cover their heads in some countries. They did so in the past. A. In the past, women cover their heads but they do so today in some countries. B. Women still cover their heads in some countries as they did in th past. C. Women still cover their heads in some countries similar to what they did so in the past. D. Women still cover their heads in some countries as they did so in the past. ________THE END_______. Trang 5/6.

<span class='text_page_counter'>(68)</span> VnDoc - Tải tài liệu, văn bản pháp luật, biểu mẫu miễn phí. BỘ GIÁO DỤC VÀ ĐÀ TẠO. ĐÁP ÁN KỲ THI TRUNG HỌC PHỔ THÔNG QUỐC GIA NĂM 2017. ĐỀ CHÍNH THỨC. Môn: TIẾNG ANH. 1D. 2A. 3A. 4D. 5C. 6D. 10A 11C. 12D. 13C. 14A. 19D 20C. 21B. 22C. 28B. 29C. 30B. 37B. 38C. 46D 47D. 8A. 9D. 15D 16B. 17C. 18A. 23A. 24D 25C. 26D. 27B. 31A. 32A. 33D 34C. 35A. 36C. 39A. 40D. 41A. 42D. 44B. 45B. 48B. 49D. 50B. Trang 6/6. 7D. 43D.

<span class='text_page_counter'>(69)</span> VnDoc - Tải tài liệu, văn bản pháp luật, biểu mẫu miễn phí SỞ GIÁO DỤC VÀ ĐÀO TẠO. KỲ THI TRUNG HỌC PHỔ THÔNG NĂM 2017. TỈNH YÊN BÁI. Môn: TIẾNG ANH. ĐỀ CHÍNH THỨC. Thời gian: 60 phút (không kể giao đề). (Đề thi có 04 trang, gồm 50 câu) MÃ ĐỀ: 02 Họ, Tên Giáo Viên: Nguyễn Đức Cương trường THPT Hoàng Văn Thụ- Lục Yên – Yên Bái. Mark the letter A, B, C, or D on your answer sheet to indicate the correct answer to each of the following questions. Question 1. We have just been told some ____________news. A. astonishing B. surprised C. astonished D. surprise Question 2. When he realized the police had spotted him, the man _______ the exit as quickly as possible. A. made out B. made for C. made up D. made off Question 3. Not until late 1960s ____________on the moon. A. when Americans walked B. when did Americans walked C. Americans walked D. did Americans walk Question 4. She couldn't decide ____________to dive ____________ jump into water. A. neither/ or B. not only/ but also C. both/ or D. whether/ or Question 5. ___________ of the financial crisis, all they could do was hold on and hope that things would improve. A. At the height B. At the bottom C. On the top D. In the end Question 6. Something began to go ____________ with the experiment when the two scientists were forced to retire. A. wrong B. stale C. faulty D. ill Question 7. I agree with most of what you said, but I can't __ your idea of letting children leave school at the age of 14. A. go along with B. put up with C. keep up with D. come up with Question 8. "Would you like to order now?" -"____________." A. Yes, a table for five B. It's excellent C. Yes, I like beef salad D. Yes, not now Question 9. ____________ he was kidnapped by the Iraqi guerrillas yesterday has been confirmed. A. That B. What C. If D. Unless Question 10. Congress has decreed that the gasoline tax ____________. A. should abolished B. is abolished C. be abolished D. should be abolished Question 11. My wallet ____________ at the station while I ____________ for the train. A. will be stolen/ am waiting B. had to steal/ would be waiting C. must have been stolen/ was waiting D. should have stolen/ had been waiting Question 12. He tends to forget things very quickly and behaves more and more like the typical ______ professor. A. cool-minded B. clear-minded C. well-minded D. absent-minded Question 13. Under the UK opportunity laws, an employee _______against on the grounds of race, religion or gender. A. cannot be discriminated B. hasn't been discriminating C. didn't use to discriminate D. should not discriminate. 1.

<span class='text_page_counter'>(70)</span> VnDoc - Tải tài liệu, văn bản pháp luật, biểu mẫu miễn phí Question 14. The new manager laid down very strict rules as soon as he had ____________ the position. A. taken over B. come over C. taken up D. taken off Question 15. ____________ your help, I wouldn't have got the scholarship. A. Had not it been for B. Unless I had C. If I had had D. But for Question 16. The more she practices, ____________ she becomes. A. the more confident B. the most confident C. the greater confidence D. the more confidently Question 17. Modern skyscrapers have a steel skeleton of beams and columns ______ a three-dimensional grid. A. and forming B. that forms C. forms D. from which forming Question 18. Sometimes life must be very unpleasant for ____________ near the airport. A. those living B. someone to live C. they who live D. people live Question 19. Jump in the car. There's enough ____________ for you. A. space B. place C. chair D. room Question 20. The climate of China is similar in many ways to ____________. A. that of the United states B. which of the United States C. the United States D. this of the United States Question 21. "Wow! What a nice coat you are wearing!" -"____________." A. Thanks. My mother bought it for me. B. I like you to say that C. Certainly. Do you like it, too? D. Yes, of course. It's expensive. Question 22. What milk shake ____________ do you want - strawberry, chocolate or orange? A. type B. taste C. kind D. flavor Question 23. She has just bought ____________. A. a French old interesting painting B. an interesting old French painting C. a French interesting old painting D. an old interesting painting French Question 24. Her outgoing character contrasts _______with that of her sister's. A. fully B. sharply C. thoroughly D. coolly Question 25. Patient: "Can I make an appointment to see the doctor, please?" Receptionist: "_________." A. OK, let me just check the diary. B. Not at the moment. He can't be disturbed. C. OK, you will need to check my diary. D. Have a seat and I'll be with you in an hour. Mark the letter A, B, C, or D on your answer sheet to indicate the word whose underlined part differs from the other three in pronunciation in each of the following questions. B. raise C. distinct D. reserve Question 26. A. husband Question 27. A. ivory B. crisis C. determine D. digest Mark the letter A,B,C, or D on your answer sheet to show the underline part that needs correction in each of the following questions. Question 28. A large amount of popular expressions in our language have interesting backgrounds. A. backgrounds B. A large amount C. expressions D. have Question 29. The incidence of which is now referred to as cryovolcanism, or ice volcanoes, is quite high on the surface of Triton, one of the moons of Neptune. A. referred to B. is C. of which D. one of the Question 30. Having served lunch, the committee members discussed the problem among themselves. A. Having served B. among themselves C. discussed D. the committee members Question 31. Bacteria are one of the most abundant life forms on earth, growing on and inside another living A. forms B. environment C. another D. abundant things, in every type of environment. Question 32. In general, novels are thought of extended works of prose fiction depicting the inner and outer lives of their characters. A. characters B. depicting C. lives D. of extended Mark the letter A, B, C, or D on your answer sheet to indicate the word(s) CLOSEST in meaning to the underlined word(s) in each of the following questions. Question 33. During the earthquake, a lot of buildings collapsed, which killed thousands of people.. 2.

<span class='text_page_counter'>(71)</span> VnDoc - Tải tài liệu, văn bản pháp luật, biểu mẫu miễn phí A. fell down unexpectedly B. went off accidentally C. erupted violently D. exploded suddenly Question 34. From an airplane, the grasslands of the western prairie appear almost as uniform as a placid sea. A. noisy B. calm C. seedy D. fake Question 35. Many plant and animal species will be in danger if we don't take any actions to protect them. A. in advance B. at stake C. on purpose D. at ease Mark the letter A, B, C, or D on your answer sheet to indicate the sentence that is closest in meaning to the sentence given in each of the following questions. Question 36. It was such a sunny day that none of us wanted to do any work. A. We wouldn't do any work if it was such a sunny day. B. None of us refused to do our work although it was a sunny day. C. We didn't work when it was sunny. D. None of us felt like doing any work because it was such a sunny day. Question 37. Lydia's reason for going to London was that she wanted to brush up her English. A. Lydia could hardly see any point in going to London to brush up her English. B. Lydia went to London so that she could improve her English. C. Not brushing up her English, Lydia had difficulty communicating when she went to London. D. Lydia went to London with a view to brush up her English. Question 38. "You didn't lock the door this morning as I found the keys on the table when I got home!" the woman told her son. A. The woman criticized her son for not locking the door that morning, adding that she saw the keys on the table. B. The woman reproached her son of not locking the door that morning, emphasizing that she saw the keys on the table. C. The woman scolded her son with unlocking the door that morning as she found the key on the table. D. The woman blamed her son for not unlocking the door that morning as she found the key on the table. Question 39. I travel by bus only when I have no alternative. A. I resort to travel by bus only when I had no alternative. B. It's my only alternative to travel by bus. C. Traveling by bus is my only alternative. D. I travel by bus only as a last resort. Question 40. Joe doesn't like it when people treat him like a child. A. Joe resents his treating like a child. B. Joe resents to be treated like a child. C. Joe resents being treated like a child. D. Joe resents of being treated like a child. Read the following passage and mark the letter A, B, C, or D on your answer sheet to indicate the correct answer to each of the questions. Very few people in the modern world obtain their food supply by hunting and gathering in the natural environment surrounding their homes. This method of harvesting from nature's provision is the oldest known subsistence strategy and has been practised for at least the last two million years. It was, indeed, the only way to obtain food until rudimentary farming and the domestication of wild animals were introduced about 10,000 years ago. Because hunter-gatherers have fared poorly in comparison with their agricultural cousins, their numbers have dwindled, and they have been forced to live in marginal environments, such as deserts and arctic wastelands. In higher latitudes, the shorter growing seasons have restricted the availability of plant life. Such conditions have caused a greater dependence on hunting, and on fishing along the coasts and waterways. The abundance of vegetation in the lower latitudes of the tropics, on the other hand, has provided a greater opportunity for gathering a variety of plants. In short, the environmental differences have restricted the diet and have limited possibilities for the development of subsistence societies. Contemporary hunter-gatherers may help us understand our prehistoric ancestors. We know from the observation of modern hunter-gatherers in both Africa and Alaska that a society based on hunting and gathering must be very mobile. While the entire community camps in a central location, a smaller party harvests the food within a reasonable distance from the camp. When the food in the area has become. 3.

<span class='text_page_counter'>(72)</span> VnDoc - Tải tài liệu, văn bản pháp luật, biểu mẫu miễn phí exhausted, the community moves on to exploit another site. We also notice seasonal migration patterns evolving for most hunter-gatherers, along with a strict division of labor between the sexes. These patterns of behavior may be similar to those practised by mankind during the Paleolithic Period. Question 41. A typical feature of both modern and prehistoric hunter-gatherers is that ____________. A. they don't have a healthy and balanced diet B. they live in the forests for all their life C. they don't have a strong sense of community D. they often change their living places Question 42. The word "marginal" in the second paragraph is closest in meaning to ____________. A. suburban B. disadvantaged C. forgotten D. abandoned Question 43. According to the passage, studies of contemporary subsistence societies can provide a _________. A. deeper insight into the dry-land farming B. further understanding of modern subsistence societies C. broader vision of prehistoric natural environments D. further understanding of prehistoric times Question 44. According to the passage, subsistence societies depend mainly on ____________. A. agricultural products B. nature's provision C. farming methods D. hunter-gatherers' tools Question 45. According to the author, most contemporary and prehistoric hunter-gatherers share ___________. A. only the way of duty division B. some restricted daily rules C. some methods of production D. some patterns of behavior Question 46. Which of the following would serve as the best title of the passage? A. Hunter-gatherers: Always on the Move B. Evolution of Humans' Farming Methods C. Hunter-gatherers and Subsistence Societies D. A Brief History of Subsistence Farming Question 47. In the lower latitudes of the tropics, hunter- gatherers ____________. A. have better food gathering from nature B. live along the coasts and waterways for fishing C. harvest shorter seasonal crops D. can free themselves from hunting Question 48. According to the passage, which of the following is NOT mentioned? A. The environmental differences produce no effect on subsistence societies. B. Hunting or fishing develops where there are no or short growing seasons. C. The number of hunter-gatherers decreases where farming is convenient. D. Harvesting from the natural environment had existed long before farming was taken up. Question 49. The word "conditions" in the second paragraph refers to ____________. A. the situations in which hunter-gatherers hardly find anything to eat B. the places where plenty of animals and fish can be found C. the environments where it is not favorable for vegetation to grow D. the situations in which hunter-gatherers can grow some crops Question 50. The word "domestication" in the first paragraph mostly means ____________. A. making wild animals used to living with and working for humans B. adapting animals to suit a new working environment C. hatching and raising new species of wild animals in the home D. teaching animals to do a particular job or activity in the home ………….HẾT…………. ĐÁP ÁN ĐỀ THI TỐT NGHIỆP THPT QUỐC GIA NĂM HỌC 2016 - 2017 Môn: TIẾNG ANH ─ Chương trình chuẩn Thời gian làm bài: 60 phút. Họ, Tên Giáo Viên: Nguyễn Đức Cương trường THPT Hoàng Văn Thụ- Lục Yên 4.

<span class='text_page_counter'>(73)</span> VnDoc - Tải tài liệu, văn bản pháp luật, biểu mẫu miễn phí Câu hỏi 1 2 3 4 5 6 7 8 9 10 11 12 13 14 15 16 17 18 19 20 21 22 23 24 25 26 27 28 29 30 31 32 33 34 35 36. Mã đề thi 01 D B C C C D C B C A C A A C A B A A C D C A B A B B C D B C D C A D C A 5. 02 A B D D A A A C A C C D A A D A B A D A A B B B A C C B C A C D A B B D.

<span class='text_page_counter'>(74)</span> VnDoc - Tải tài liệu, văn bản pháp luật, biểu mẫu miễn phí 37 38 39 40 41 42 43 44 45 46 47 48 49 50. B A D C B C A A D B D B B B. 6. B A D C A B D B D C A A C A.

<span class='text_page_counter'>(75)</span> VnDoc - Tải tài liệu, văn bản pháp luật, biểu mẫu miễn phí SỞ GD ĐT YÊN BÁI TRƯỜNG THPT HOÀNG VĂN THỤ. ĐỀ THI THPT QUỐC GIA NĂM HỌC 2016-2017 MÔN: TIẾNG ANH Thời gian làm bài: 60 phút Mã đề thi:03. Mark the letter A, B, C, or D on your answer sheet to indicate the word whose underlined part differs from the other three in pronunciation in each of the following questions Question 1. A. punished B. markedly C. laughed D. cooked Question 2. A. predator B. restore C. recollect D. preface Mark the letter on your answer sheet to indicate the word that differs from the rest in the position of the main stress Question 3. A. furnish B. reason C. promise D. tonight Question 4. A. enterprise B. prevention C. fertilize D. implement Mark the letter A, B, C, or D on your answer sheet to indicate the correct answer to each of the following questions. Question 5: The………….. customer thanked the salesperson for the good service. A. satisfy B.satisfactory C.satisfied D.satisfying Question 6: The workers have gone on strike……………, all production has creased. A. So that B. Therefore C.Because D. Now that Question 7: When exactly did the war ……………..between the two countries? A. go off B.set in C. break out D.call off Question 8: I’ll never ……………the King of pop music for the first time. A. forget meet B. forget met C. forget to meet D.forget meeting Question 9: Many women find it hard to……………… both a job and a family. A. catch up with B.deal with C.cope with D.keep up with Question 10: I have never seen……………….festival in my life A. so spectacular a B. a so spectacular C. a such spectacular D. such spectacular Question 11: They’ve just taken on 200 people……………...the number that was taken on last year. A. twice bigger as B. twice as many as C.as twice as D.bigger two times than Question 12: …………….it with my own eyes, I would have never believed it. A. Unless I had not seen B.Provided I had seen C.If I had seen D.Had I not seen Question 13. She should _____ in the garage when we come around, which would explain why she didn't hear the bell. A. work B. be working C. have worked D. have been working Question 14. Dr. Parker gave my mum a lovely ________ for spaghetti carbonara. A. recipe B. prescription C. receipt D. paper Question 15. You should __________ a professional to check your house for earthquake damage. A. have B. make C. take D. get Question 16. We _________ today and I got into trouble because I hadn't done it. A. had checked our homew B. had our homework checked C. were checked our homework D. have our homework checking Mark the letter A, B, C, or D on your answer sheet to show the underlined part that needs correction. Question 17: Globally and internationally, the 1990's stood out as the warmest decade in the history of weather records A B C D Question 18: When precipitation occurs, some of it evaporates, some runs off the surface it strikes, and some A B C sinking into the ground. D Question 19: A warning printed on a makeshift lifebuoy says: "This is not a life-saving device. Children should be accompany by their parents." A B C D Mark the letter A, B, C, or D on your answer sheet to indicate the sentence that is closest in meaning to each of the following questions. Question 20: We chose to find a place for the night. We found the bad weather very inconvenient. A. Bad weather was approaching, so we started to look for a place to stay. B. The bad weather prevented us from driving any further. C. Seeing that the bad weather had set in, we decided to find somewhere to spend the night..

<span class='text_page_counter'>(76)</span> VnDoc - Tải tài liệu, văn bản pháp luật, biểu mẫu miễn phí D. Because the climate was so severe, we were worried about what we'd do at night. Question 21: It isn't just that the level of education of this school is high. It's that it's also been consistent for years. A. The level of education in this school, which is usually quite high, shows only slight variations from year to year. B. The standard of education is not high in this school, but at least all the students are at the same level. C. Not only are the standards of education good in this school, but it has maintained those standards over the years. D. It isn't fair to deny that this school is successful, as it has had the same high standards for many years now. Mark the letter A, B, C, or D on your answer sheet to indicate the most suitable response to complete each of the following exchanges. Question 22. – “How lovely your pets are!” “ _________” A. Thank you, it's nice of you to say so B. Really? They are C. can you say that again Question 23. Stephanie: "Oh, no! I left my book at home. Can I share yours?" Scotts: "_____" A. Yes, I do too B. No, thanks C. Yes, sure! D. No, not at all!. D. I love them, too. Choose the sentence that is similar in meaning to the given one.. Question 24. Bali has far better beaches than Java, which make it more attractive to tourists. A. Bali is more popular with tourists because it has more beaches than Java. B. Bali attracted more tourists than Java because its beaches are much farther. C. Tourists prefer the beaches in Bali to those in Java because they are far better. D. Bali is more popular with tourists because its beaches are much better than those of Java Question 25. “Send this urgent document immediately!” the officer told the soldier. A. The officer requested that the soldier rush out due to the document’s urgency. B. The officer ordered the soldier to deliver the urgent document instantly. C. The officer recommended the soldier leave right away because of the urgent document. D. The officer advised the soldier to send the urgent document right away. Question 26. It is widely believed that hard work makes success. A. People think that success is when you work hard. B. Believers of success think that we should work hard. C. Many people think that success at work is hard. D. Many people believe that if you want to succeed, you should work hard. Mark the letter A, B, C, or D to indicate the word or phrase that is CLOSEST in meaning to the underlined part in each of the following questions Question 27. This tapestry has a very complicated pattern. D. appropriate A. obsolete B. intricate C. ultimate Question 28. The drought was finally over as the fall brought in some welcome rain. A. heatware B. harvest C. summer D. aridity Mark the letter A, B, C or D on your answer sheet to indicate the word or phrase that is OPPOSITE in meaning to the underlined part in each of the following questions. Question 29: During the five- decade history the Asian Games have been advancing in all aspects. D. holding by A. holding at B. holding back C. holding to Question 30: The distinction between schooling and education implied by this remark is important. A. explicit B. implicit C. obscure D. odd Read the following passage and mark the letter A, B, C, or D on your answer sheet to indicate the correct answer to each of the sentences The principle of use and disuse states that those parts of organisms' bodies that are used grown larger. Those parts that are not tend to wither away. It is an observed fact that when you exercise particular muscles, they grow. Those that are never used dimish. By examining a man's body, we can tell which muscles he uses and which he doesn't. we may even be able to guess his profession or his reaction. Enthusiasts of the "body- building" cult make use of the principle of use and disuse to "build" their bodies, almost like a piece of sculpture, into whatever unnatural shape is demanded by fashion in this peculiar minority culture. Muscles are not the only parts of the body that respond to use in this kind of way. Walk barefoot and you acquire harder skin on your soles. It is easy to tell a farmer from a bank teller by looking at their hands alone. The farmer's hands are horny, hardened by long exposure to rough work. The teller's hands are relatively soft. The principle of use and disuse enables animals to become better at the job of surviving in their world, progressively better during their lifetime as a result of living in that world. Humans, through direct exposure to sunlight, or lack of it, develop a skin color which equips them better to survive in the particular local conditions. Too much sunlight is dangerous. Enthusiastic sunbathers with very fair skins are susceptible to skin cancer. Too little sunlight, on the other hand, leads to vitamin-D deficiency and rickets. The brown pigment melanin which is.

<span class='text_page_counter'>(77)</span> VnDoc - Tải tài liệu, văn bản pháp luật, biểu mẫu miễn phí synthesized under the influence of sunlight, makes a screen to protect the underlying tissues from the harmful effects of further sunlight. If a suntanned person moves to a less sunny climate, the melanin disappears, and the body is able to benefit from what little sun there is. This can be represented as an instance of the principle of use and disuse: skin goes brown when it is "used", and fades to white when it is not. Question 31. What does the passage mainly discuss? A. How the principles of use and disuse change people's concepts of themselves. B. The way in which people change themselves to conform to fashion. C. The changes that occur according to the principle of use and disuse. D. The effects of the sun on the principle of use and disuse. Question 32. The word "Those" in line 3 refers to___. A. organisms B. bodies C. parts D. muscles Question 33. According to the passage, men who body build____. A. appear like sculptures B. change their appearance C. belong to strange cults D. are very fashionable Question 34. The word "horny" in line 9 is closest in meaning to____. A. firm B. strong C. tough D. dense Question 35. It can be inferred from the passage that the principle of use and disuse enables organisms to____. A. change their existence B. automatically benefit C. survive in any condition D. improve their lifetime Question 36. The author suggests that melanin_____. A. is necessary for the production of vitamin-D B. is beneficial in sunless climates C. helps protect fair-skinned people D. is a synthetic product Question 37. The word " susceptible" could be best replaced by____. A. condemned B. vulnerable C. allergic D. suggestible Read the following passage and mark the letter A, B, C or D on your answer sheet to indicate the correct word for each Environmental Concerns Earth is the only place we know of in the universe that can support human life. Yet human activities are making the planet less fit to live on. As the western world carries on consuming two-thirds of the world's resources while half of the world's population do so just to stay alive we are rapidly destroying the very resource we have by which all people can survive and prosper. Everywhere fertile soil is (38) _____ built on or washed into the sea. Renewable resources are exploited so much that they will never be able to recover (39) _____. We discharge pollutants into the atmosphere without any thought of the consequences. As a result the planet's ability to support people is being reduced at the very time when rising human numbers and consumption are (41) _____ increasingly heavy demands on it. The Earth's (42) _____ resources are there for us to use. We need food, water, air, energy, medicines, warmth, shelter and minerals to (43) _____ us fed, comfortable, healthy and active. If we are sensible in how we use the resources, they will go indefinitely. But if we use them wastefully and excessively, they will soon run out and everyone will suffer. Question 38: A. neither. B. sooner. C. rather. D. either. Question 39: A. utterly. B. completely. C. quite. D. greatly. Question 40: A. making. B. doing. C. having. D. taking. Question 41: A. living. B. real. C. natural. D. genuine. Question 42: A. maintain. B. stay. C. hold. D. keep. Read the following passage and mark the letter A, B, C or D on your answer sheet to indicate the correct word for each of the questions You can usually tell when your friends are happy or angry by the looks on their faces or by their actions. This is useful because reading their emotional expressions helps you to know how to respond to them. Emotions have evolved to help us respond to important situations and to convey our intentions to others. But does raising the eyebrows and rounding the mouth say the same thing in Minneapolis as it does in Madagascar? Much research on emotional expressions has centered on such questions. According to Paul Ekman, the leading researcher in this area, people speak and understand substantially the same “facial language”. Studies by Ekman’s group have demonstrated that humans share a set of universal emotional expressions that testify to the common biological heritage of the human species. Smiles, for example, signal happiness and frowns indicate sadness on the faces of people in such far- flung places as Argentina, Japan, Spain, Hungary, Poland , Sumatra ,the United States, Vietnam, the jungles of New Guinea , and the Eskimo villages north of Artic Circle. Ekman and his colleagues claim that people everywhere can recognize at least seven basic emotions: sadness, fear, anger, disgust, contempt, happiness, and surprise. There are, however, huge differences across cultures in both the.

<span class='text_page_counter'>(78)</span> VnDoc - Tải tài liệu, văn bản pháp luật, biểu mẫu miễn phí context and intensity of emotional displays – the so called display rules. In many Asian cultures, for example, children are taught to control emotional responses – especially negative ones- while many American children are encouraged to express their feelings more openly. Regardless of culture, however, emotions usually show themselves, to some degree , in people’s behavior. From their first days of life, babies produce facial expressions that communicate their feelings. The ability to read facial expressions develops early, too. Very young children pay close attention to facial expressions, and by age five, they nearly equal adults in their skill at reading emotions on people’s faces. This evidence all points to a biological underpinning for our abilities to express and interpret a basic set of human emotions. Moreover, as Charles Darwin pointed out over a century ago, some emotional expressions seem to appear across species boundaries. Cross - cultural psychologists tell us that certain emotional responses carry different meanings in different cultures. For example, what emotion do you suppose might be conveyed by sticking out your tongue? For Americans, this might indicate disgust, while in China it can signify surprise. Likewise, a grin on an American face may indicate joy, while on a Japanese face it may just as easily mean embarrassment. Clearly, culture influences emotional expressions. Question 43. According to the passage, we respond to others by _________. A. observing their looks B. observing their emotional expressions C. watching their actions D. looking at their faces Question 44. Many studies on emotional expressions try to answer the question whether __________. A. different cultures have similar emotional expressions. B. eyebrow raising means the same in Minneapolis and Madagascar. C. raising the eyebrows has similar meaning to rounding the mouth. D. rounding the mouth has the same meaning in Minneapolis and Madagascar. Question 45. The word “ evolved” in line 3 is closest in meaning to __________. A. reduced B. increased C. simplified D. developed Question 46. Paul Ekman is mentioned in the passage as an example of ____________. A. lacked many main ingredients B. researchers on universal language C.researchers who can speak and understand many languages D.investigators on universal emotional expressions Question 47. Unlike American children, Asian children are encouraged to _______. A. control their emotions B. conceal their positive emotions C. display their emotions openly D. change their behaviour Question 48. Young children _______. A. spend a long time learning to read others’ emotions B. are sensitive towards others’ emotions C. make amazing progress in controlling their emotions D. take time to control their facial expressions Question 49. The phrase “ this evidence” in line 24 refers to _________. A. the fact that children are good at recognizing others’ emotions B. human facial expressions C. a biological underpinning for humans to express emotions D. the fact that children can control their feelings Question 50. The best title for the passage is ________________. A.Cultural universals in emotional expressions B. Ways to control emotional expressions C. review of research on emotional expressions D. Human habit of displaying emotions. --------------THE END------------.

<span class='text_page_counter'>(79)</span> VnDoc - Tải tài liệu, văn bản pháp luật, biểu mẫu miễn phí. SỞ GD ĐT YÊN BÁI TRƯỜNG THPT HOÀNG VĂN THỤ. ANSWERS: 10pts/ 50. 0,2pts/01 1- B 11- B 2- B 12- D 3- D 13- D 4- B 14- A 5- C 15-D 6- B 16- B 7-C 17- A 8- D 18- D 9- C 19- C 10- A 20- C. ĐÁP ÁN ĐỀ THI THPT QUỐC GIA NĂM HỌC 2016-2017 MÔN: TIẾNG ANH Thời gian làm bài: 60 phút Mã đề thi:03. 21- C 22- A 23- C 24- D 25- B 26- D 27- B 28- D 29- B 30- A. The end. 31- C 32- D 33- B 34-C 35-C 36-C 37-B 38-D 39-B 40-A. 41-C 42-B 43-B 44-A 45-D 46-D 47-A 48-B 49-B 50-A.

<span class='text_page_counter'>(80)</span> VnDoc - Tải tài liệu, văn bản pháp luật, biểu mẫu miễn phí. SỞ GIÁO DỤC VÀ ĐÀO TẠO YÊN BÁI KỲ THI TRUNG HỌC PHỔ THÔNG QUỐC GIA NĂM 2017 TRƯỜNG THPT VĂN CHẤN Môn: TIẾNG ANH Thời gian làm bài: 60 phút, không kể thời gian phát đề (Đề thi có 05 trang). Mark the letter A, B, C, or D on your answer sheet to indicate the word whose underlined part differs from the other three in pronunciation in each of the following questions. Question 1: A. processed B. infested C. balanced D. reached Question 2: A. advocate B. manage C. rival D. canal Mark the letter A, B, C, or D on your answer sheet to indicate the word that differs from the other three in the position of primary stress in each of the following questions. Question 3: A. different B. deployment C. evaluate D. appearance Question 4: A. congratulate B. regulate C. communicate D. coordinate Mark the letter A, B, C, or D on your answer sheet to indicate the underlined part that needs correction in each of the following questions. Question 5: My daily expenses are just about equal with my income. A B C D Question 6: That’s the young man whom we met him at the bus-stop last week, isn’t it? A B C D Question 7: There were several large holes in the road, five of them had to be repaired urgently. A B C D Mark the letter A, B, C, or D on your answer sheet to indicate the correct answer to each of the following questions. Question 8: _______ you study for these exams, _______ you will do. A. The harder / the better B. The more / the much C. The hardest / the best D. The more hard / the more good Question 9: By December, Tim _______ enough money to buy a mountain bike. A. saves B. will save C. has saved D. will have saved Question 10: Who was the last one ________ the classroom yesterday? A. had left B. to leave C. left D. leaving Question 11: Mary: “ Do you think it will rain ? “ Jenny: “Oh ! ________” A. I don’t hope. B. I hope not. C. I don’t hope so. D. It’s hopeless Question 12: Final year students ____________ attend lectures. It’s optional. A. don’t have to B. mustn’t C. shouldn’t D. ought to Question 13: Nobody is ready to go, ________ ? A. are they B. isn’t he C. is he D. aren’t they Question 14: ________ had they recovered from the first earthquake when they felt the second tremor. A. Never B. Scarcely C. No sooner D. Just Question 15: If we do not take steps to protect the world’s wildlife, many species of birds and animals are likely to ________ completely. A. die out B. die down C. die away D. die from Question 16: Take this umbrella with you because it _______ rain this evening. A. must B. may C. can D. shall Question 17: I would rather you _______ anything about it for the time being. A. don’t do B. not do C. not to do D. didn’t do Trang 1/1.

<span class='text_page_counter'>(81)</span> VnDoc - Tải tài liệu, văn bản pháp luật, biểu mẫu miễn phí. Question 18: They have just visited the town _______ location was little known. A. where B. whose C. which D. that Question 19: ___________one day by a passing car, the dog never walked properly again. A. Having injure B. Injuring C. Injured D. To be injured. Mark the letter A, B, C, or D on your answer sheet to indicate the most suitable response to complete each of the following exchanges. Question 20: Peter said to Jane carrying a heavy suitcase: “Need a hand with your suitcase, Jane?” Jane: “_______.” A. Well done B. That’s very kind of you C. Not a chance D. I don’t believe it Question 21: A has just turned up at the meeting and said to the group-leader: “I’m sorry. I was stuck in a traffic jam.” The group-leader: “______.” A. Never mind. Thanks B. Don’t apologize. It’s all my fault. C. My pleasure D. It’s Ok. We’ve just started Mark the letter A, B, C, or D on your answer sheet to indicate the word(s) CLOSEST in meaning to the underlined word(s) in each of the following questions. Question 22: Mayo Hospital in New Orleans was so named in recognition of Dr. Mayo’s outstanding humanitarianism. A. exhaustive B. charitable C. remarkable D. widespread Question 23: Let's wait here for her; I'm sure she'll turn up before long. A. enter B. visit C. arrive D. return Mark the letter A, B, C, or D on your answer sheet to indicate the word(s) OPPOSITE in meaning to the underlined word(s) in each of the following questions. Question 24: That is a well-behaved boy whose behavior has nothing to complain about. A. behaving cleverly B. behaving nice C. behaving improperly D. good behavior Question 25: I must have a watch since punctuality is imperative in my new job. A. being courteous B. being cheerful C. being efficient D. being late Mark the letter A, B, C, or D on your answer sheet to indicate the sentence that is closest in meaning to each of the following questions. Question 26: “Do not note down everything I say. Just listen to me.” said the teacher to the boys. A. The teacher told the boys to listen to her and not to note down everything she said. B. The teacher told the boys do not note down everything she said and listen to her. C. The teacher said to the boys not note down everything she said and listen to her. D. The teacher said the boys not to note down everything she said. Question 27: You shouldn’t have behaved improperly with your teacher like that. A. You behaved with your teacher properly. B. You didn’t behave properly because of your teacher. C. You should behave with your teacher more properly. D. It was essential to behave with your teacher properly but you didn’t. Question 28: When Jim talked about the problems arising in his work, they were quite familiar to us. A. We were quite familiar to his talk on the problems arising in his work. B. The problems were familiar to the problems arising in his work he talked. C. The problems arising in his work that Jim talked about were familiar to what we knew. D. We didn’t understand the problems arising in his work, which he talked about. Trang 1/2.

<span class='text_page_counter'>(82)</span> VnDoc - Tải tài liệu, văn bản pháp luật, biểu mẫu miễn phí. Read the following passage and mark the letter A, B, C, or D on your answer sheet to indicate the sentence that best combines each pair of sentences in the following questions. Question 29: He gave me the information. I wrote it down at once. A. He gave me the information that I wrote it down at once. B. He gave me the information, that I wrote down at once. C. He gave me the information which I wrote it down at once. D. He gave me the information which I wrote down at once. Question 30: He comes from a large family. All of them now live in Canada. A. He comes from a large family all of who now live in Canad.a B. He comes from a large family all of whose now live in Canada. C. He comes from a large family, all of whom now live in Canada. D. He comes from a large family, all of that now live in Canada. Read the following passage and mark the letter A, B, C, or D on your answer sheet to indicate the correct word or phrase that best fits each of the numbered blanks Here are the tips that help succeed in your job interview: Always arrive early. If you do not know ______(31)_____the organization is located, call for exact directions _____(32)_____advance. Leave some extra time for any traffic, parking, or ______(33)____events. If you are running late, call right away and let someone know. The best time to arrive is approximately 5-10 minutes early. Give ____(34)____ the time to read your résumé on more time, to catch your breath, and to be ready for the interview. Once you are at the office, treat everyone you encounter with respect. Be ______(35)____to everyone as soon as you walk into the door. Question 31: A. why B. when C. where D. that Question 32: A. with B. in C. on D. for Question 33: A. expected B. unexpected C. unexpectedly D. expectedly Question 34: A. you B. your C. yours D. yourself Question 35: A. pleasant B. happy C. disappointed D. excited Read the following passage and mark the letter A, B, C, or D on your answer sheet to indicate the correct answer to each of the questions. While many nineteenth-century reformers hoped to bring about reform through education or by eliminating specific social evils, some thinkers wanted to start over and remake society by founding ideal, cooperative communities. The United States seemed to them a spacious and unencumbered country where models of a perfect society could succeed. These communitarian thinkers hoped their success would lead to imitation, until communities free of crime, poverty, and other social ills would cover the land. A number of religious groups, notably the Shakers, practiced communal living, but the main impetus to found model communities came from nonreligious, rationalistic thinkers. Among the communitarian philosophers, three of the most influential were Robert Owen, Charles Fourier and John Humphrey Noyes Owen, famous for his humanitarian policies as owner of several thriving textile mills in Scotland, believed that faulty environment was to blame for human problems and that these problems could be eliminated in a rationally planned society. In 1825 he put his principles into practice at New Harmony, Indiana. The community failed economically after a few years but not before achieving a number of social successes. Fourier, a commercial employee in France, never visited the United States. However, his theories of cooperative living influenced many Americans through the writings of Albert Brisbane, whose Social Destiny of Man explained Fourier-ism and its self-sufficient associations or “phalanxes”. One or more of these phalanxes was organized in every Northern state. The most famous were Red Bank, New Jersey, and Brook Farm, Massachusetts. An early member of the latter was the author Nathaniel Hawthorne. Noyes founded the most enduring and probably the oddest of the utopian communities, the Oneida Community of upstate New York. Needless to say, none of these experiments had any lasting effects on the patterns of Trang 1/3.

<span class='text_page_counter'>(83)</span> VnDoc - Tải tài liệu, văn bản pháp luật, biểu mẫu miễn phí. American society. Question 36: The main topic of the passage is _______. A. nineteenth-century schools B. model communities in the nineteenth century C. the philosophy of Fourierism D. American reforme Question 37: The Shakers are mentioned in line 6 as an example of_______. A.rationalistic thinkers B. radical reformers C. an influential group of writers D. a communal religious group Question 38: The “phalanxes” described in the second paragraph were an idea originally conceived by_______. A. Robert Owen B. Charles Fourier C. John Humphrey Noyes D. Albert Brisbane Question 39: Why does the author mention Nathaniel Hawthorne in line 18? A. He was a critic of Charles Fourier B. He wrote a book that led to the establishment of model communities C. He founded Brook Farm in Massachu-setts D. He was at one time a member of the Brook Farm community Question 40: Which of the following communities lasted longest? A. Brook Farm B. Red Ban C. The Oneida Community D. New Harmony Question 41: The word oddest in line 19 is closest in meaning to which of the following? A. Largest B. Earliest C. Most unusual D. Most independent Question 42: The author implies that, for readers, the conclusion of the passage is __________ . A. practical B. Absurd C. obvious D. surprising Read the following passage and mark the letter A, B, C, or D on your answer sheet to indicate the correct answer to each of the questions Although noise, commonly defined as unwanted sound, is a widely recognized form of pollution, it is very difficult to measure because the discomfort experienced by different individuals is highly subjective and, therefore, variable. Exposure to lower levels of noise may be slightly irritating, whereas exposure to higher levels may actually cause hearing loss. Particularly in congested urban areas, the noise produced as a byproduct of our advancing technology causes physical and psychological harm, and detracts from the quality of life for those who are exposed to it. Unlike the eyes, which can be covered by the eyelids against strong light, the ear has no lid, and is, therefore, always open and vulnerable; noise penetrates without protection. Noise causes effects that the hearer cannot control and to which the body never becomes accustomed. Loud noises instinctively signal danger to any organism with a hearing mechanism, including human beings. In response, heartbeat and respiration accelerate, blood vessels constrict, the skin pales, and muscles tense. In fact, there is a general increase in functioning brought about by the flow of adrenaline released in response to fear, and some of these responses persist even longer than the noise, occasionally as long as thirty minutes after the sound has ceased. Because noise is unavoidable in a complex, industrial society, we are constantly responding in the same way that we would respond to danger. Recently, researchers have concluded that noise and our response may be much more than an annoyance. It may be a serious threat to physical and psychological health and well-being, causing damage not only to the ear and brain but also to the heart and stomach. We have long known that hearing loss is America’s number one nonfatal health problem, but now we are learning that some of us with heart disease and ulcers may be victims of noise as well. Fetuses exposed to noise tend to be overactive, they cry easily, and they are more sensitive to gastrointestinal problems after birth. In addition, the psychic effect of noise is very important. Nervousness, irritability, tension, and anxiety increase affecting the quality Trang 1/4.

<span class='text_page_counter'>(84)</span> VnDoc - Tải tài liệu, văn bản pháp luật, biểu mẫu miễn phí. of rest during sleep, and the efficiency of activities during waking hours, as well as the way that we interact with each other. Question 43: Which of the following is the author’s main point? A. Noise may pose a serious threat to our physical and psychological health. B. Loud noises signal danger. C. Hearing loss is America’s number one nonfatal health problem. D. The ear is not like the eye. Question 44: According to the passage, what is noise? A. Unwanted sound. B. A byproduct of technology. C. Physical and psychological harm. D. Congestion. Question 45: Why is noise difficult to measure? A. All people do not respond to it in the same way. B. It causes hearing loss. C. It is unwanted. D. People become accustomed to it. Question 46: The word congested in paragraph 1 could be best replaced by _____. A. hazardous B. polluted C. crowded D. rushed Question 47: It can be inferred from the passage that the eye _____. A. responds to fear B. enjoys greater protection than the ear C. increases functions D. is damaged by noise Question 48: According to the passage, people respond to loud noises in the same way that they respond to _____. A. annoyance B. disease C. damage D. dangers Question 49: The word accelerate in paragraph 3 is closest in meaning to _____. A. decrease B. alter C. increase D. release Question 50: The word it in paragraph 3 is closest in meaning to _____. A. the noise B. the quality of life C. advancing technology D. a by product. __________THE END___________. Trang 1/5.

<span class='text_page_counter'>(85)</span> VnDoc - Tải tài liệu, văn bản pháp luật, biểu mẫu miễn phí ĐÁP ÁN ĐỀ THI THPTQG 2017 - Tổng số câu: 50. 1B. 11 B. 21 D. 31 C. 41 C. 2C. 12 A. 22 C. 32 B. 42 C. 3A. 13 A. 23 C. 33 B. 43 A. 4B. 14 B. 24 C. 34 D. 44 A. 5D. 15 A. 25 D. 35 A. 45 A. 6B. 16 B. 26 A. 36 B. 46 C. 7C. 17 D. 27 D. 37 D. 47 B. 8A. 18 B. 28 C. 38 B. 48 D. 9D. 19 C. 29 A. 39 D. 49 C. 10 B. 20 B. 30 D. 40 C. 50 A. Trang 1/6.

<span class='text_page_counter'>(86)</span> VnDoc - Tải tài liệu, văn bản pháp luật, biểu mẫu miễn phí TRƯỜNG THPT SƠN THỊNH ___________________ ĐỀ CHÍNH THỨC (Đề thi gồm: 04 trang). KÌ THI TRUNG HỌC PHỔ THÔNG QUỐC GIA NĂM 2017 Môn: TIẾNG ANH Thời gian làm bài: 60 phút, không kể thời gian phát đề ______________________________________________. 㔠 R Ro x R …………………………………………………………. SBD ………………………… Mark Re le er A, B, C, or D o your a wer Ree o x dxca e Re word wRo e u derlx ed par dxffer from Re o Rer Rree x pro u cxa xo x eacR of Re followx g que xo . Question 1:A. passed B. wished C. realized D. touched Question 2:A. knees B. papers C. trees D. bats Mark Re le er A, B, C, or D o your a wer Ree o x dxca e Re word Ra dxffer from Re o Rer Rree x Re po x xo of prxmary re x eacR of Re followx g que xo . Question 3:A. equality B. available C. manufacture D. biography Question 4:A. adventure B. attendance C. opponent D. penalty Question 5: A. represent B. permanent C. continent D. sentiment Mark Re le er A, B, C, or D o your a wer Ree o x dxca e Re word( ) CLOSEST x mea x g o Re u derlx ed word( ) x eacR of Re followx g que xo . Question 6: We have lived there for years and grow fo d of Re urrou dx g . That is why we don’t want to leave. A. haunted by the surroundings B. possessed by the surroundings C. loved the surroundings D. planted many trees in the surroundings Question 7: His new work has enjoyed a very good re xew from critics and readers. A. regard B. viewing C. look D. . opinion Question 8: When the protestor entered the meeting clad only in a beach tower, the audience was dumbfou ded. A. speechless B. excited C. content D. applauding Mark Re le er A, B, C, or D o your a wer Ree o x dxca e Re word( ) OPPOSITE x mea x g o Re u derlx ed word( ) x eacR of Re followx g que xo . Question 9: On November 25 1972, something dreadful happened on board of the brigantine Mary Celeste, causing all crew members to hastily aba do the ship. A. hold on B. stay on C. take care of D. save for Question 10: The motorist felt that the ticket for infraction was u warra ed. A. conscientious B. inadvertent C. inevitable D. justified Mark Re le er A, B, C, or D o your a wer Ree o x dxca e Re u derlx ed par Ra eed correc xo x eacR of Re followx g que xo . Question 11: Hardly did he enter the room when all the lightswent out. A. did he enter B. when C. the lights D. went Question 12: Publishing in the UK, the book has won a number of awards in recent regional book fairs. A. Publishing in the UK B. has won C. in D. book fairs Question 13: I would be to see him, but I didn’t have time because of the coming exam. A. would be B. to see C. because of D. coming Question 14: Although he was happily married, he preferred spending lately hours at work to spending evenings lazily at home. A. happily B. lately C. to spending D. lazily Mark Re le er A, B, C, or D o your a wer Ree o x dxca e Re correc a wer Question 15: Labor unions and the company______ in a confrontation over plans to curb benefits. Trang 1/4.

<span class='text_page_counter'>(87)</span> VnDoc - Tải tài liệu, văn bản pháp luật, biểu mẫu miễn phí A. carried away B. faced off C. caught up D. showed up Question 16: What university will you take an_________ examination into? A. entrance B. entry C. admission D. attendance Question 17: At first the children enjoyed the game but quite soon _______ novelty. A. died out B. wore off C. went off D. died out Question 18: “Are you coming for a drink?” – “______________. I must get this finished tonight.” A. Thanks, but I mustn’t B. As a matter of fact, I do C. Can I take a rain check D. Well, that’s very surprising Question 19: Although he claimed to have left his job voluntarily, he was actually__________ for misconduct.A. released B. dismissed C. resigned D. dispelled Question 20: ______ the fifth largest among the night planets that make up our solar system. A. The Earth is B. The Earth being C. That the Earth is D. Being the Earth Question 21: Although the twins look identical, they have widely________ opinion on almost every topic under the sun. A. distinguished B. distinct C. dissimilar D. divergent Question 22: ________ but he also proves himself a good athlete. A. Not only did he show himself a good student B. Not only he showed himself a good student C. He did not show himself only a good student D. A good student not only showed himself Question 23: It is recommended that he__________- this course. A. took B. take C. takes D. taking Read Re followx g pa age a d mark Re le er A, B, C, or D o your a wer Ree o x dxca e Re correc word or pRra e Ra be fx eacR of Re umbered bla k . Ask most people for their list of top ten of fears, and you’ll be sure to find (24)________ burgled is fairly high on the list. An informal survey I carried among friends at a party last week revealed that eight of them had had their homes (25)________ into more than twice, and two had been burgled five times. To put the record straight, (26)________ of my friend owns valuable paintings or a sideboard full of family silverware. Three of them are students, in fact. The most typical burglar, it seems, involved the theft of easily transportable items – the television, the video, even food from freezer. This may have something to do with the (27)________ that the average burglar is in his (or her) late teens, and probably wouldn’t know what to do with a Picasso, whereas selling a Walkman or a vacuum cleaner is a much easier matter. They are perhaps not so (28)________ professional criminals, as hard-up young people who need a few pounds and some excitement. (29)________ that this makes your house turned upside down and your favorite things stolen any easier to (30)________. In most case, the police have no luck recovering any of the stolen goods. Unless there is any definite (31)________, they are probably unable to do anything at all. And alarms or special locks don’t seem to help either. The only advice my friends could (32)________ was “Never live on the ground floor” and “Keep two or three fierce dogs”, which reminded me of a case, I read about, where the burglars’ (33)________ included the family’s pet poodle. Question 24: A. being Question 25: A. robbed Question 26: A. some Question 27: A. information Question 28: A. many Question 29: A. given Question 30: A. believe Question 31: A. case Question 32: A. put in with Question 33: A. loot. B. been B. taken B. all B. fact B. rarely B. not B. do B. burglary B. bring up with B. takings. C. having C. broken C. few C. idea C. that C. so C. accept C. evidence C. get by with C. profits. D. out D. entered D. none D. knowledge D. much D. despite D. attempt D. investigation D. come up with D. receipt Trang 2/4.

<span class='text_page_counter'>(88)</span> VnDoc - Tải tài liệu, văn bản pháp luật, biểu mẫu miễn phí Read Re followx g pa age a d mark Re le er A, B, C, or D o your a wer Ree o x dxca e Recorrec a wer o eacR of Re que xo . The food we eat seems to have profound effects on our health. Although science has made enormous steps in making food more fx to eat, it has, at the same time, made many foods unfit to eat. Some research has shown that perhaps eighty percent of all human illnesses are related to diet and forty percent of cancer is related to the diet as well, especially cancer of the colon. People of different cultures are more pro e to contact certain illnesses because of the characteristic foods they consume. That food is related to illness is not a new discovery. In 1945, government researchers realized that nitrates nitrites (commonly used to preserve color in meat) as well as other food additives caused cancer. Yet, Re e carcx oge xc additives remain in our food, and it becomes more difficult all the time to know which ingredients on the packaging label of processed food are helpful or harmful. The additives that we eat are not all so direct. Farmers often give penicillin to cattle and poultry, and because of this, penicillin has been found in the milk of treated cows. Sometimes similar drugs are administered to animals not for medical purposes, but for financial reasons. The farmers are simply trying to fatten the animals in order to obtain a higher price on the market. Although the Food and Drug Administration (FDA) has tried repeatedly to control these procedures, the practices continue. A healthy diet is directly related to good health. Often we are unaware of detrimental substances we ingest. Sometimes well-meaning farmers or others who do not realize the consequences add these substances to food without our knowledge. Question 34: How has science done to disservice to people? A. As a result of scientific intervention, some potentially harmful substances have been added to our food B. The scientists have preserved the color of meats, but not of vegetables C. It caused a lack of information concerning the value of food D. Because of science, disease caused by contaminated food has been virtually eradicated. Question 35: The word “pro e” is nearest meaning to_________. A. healthy B. unlikely C. supine D. predisposed Question 36: The word “carcx oge xc” is closest in meaning to_________. A. trouble-making B. money-making C. cancer-causing D. colorretaining Question 37: What are nitrates used for? A. They preserve the color of meat B. They preserve flavor in package food C. There are objects of research D. They cause the animals become fatter Question 38: FDA means____________. A. Federal Dairy Additives B. Food and Drug Administration C. Final Difficult Analysis D. Food Direct Additives Question 39: All of the following statements are TRUE except A. Drug are always given to animals for medical reasons B. Food may cause forty percent of the cancer in the world C. Researchers have known about the potential hazard of food additives for more than 45 years D. Some of the additives in our food are added to the food itself and some are given to the living animals Question 40: What is best title for this passage? A. The food you eat can affect your health B. Harmful and Harmless substances in food C. Avoiding injurious substances in food D. Improving health through a Natural Diet Question 41: The word “fx ” could be best replaced by which of the following? A. suitable B. tasty C. athletic D. adaptable Trang 3/4.

<span class='text_page_counter'>(89)</span> VnDoc - Tải tài liệu, văn bản pháp luật, biểu mẫu miễn phí Question 42: The word “ Re e” refers to___________? A. researchers B. nitrates and nitrites C. meats D. colors Question 43: The word “addx x e ” is closest meaning to__________. A. begin substance B. natural substance C. dangerous substance substance. D. added. Mark Re le er A, B, C, or D o your a wer Ree o x dxca e Re e e ce Ra x clo e x mea x g o eacR of Re followx g que xo . Question 44: “You shouldn’t have leaked our confidential report to the press, Frank!” said Jane. A. Jane suspected that Frank had leaked their confidential report to the press. B. Jane criticized Frank for having disclosed their confidential report to the press. C. Jane accused Frank of having cheated the press with their confidential report. D. Jane blamed Frank for having flattered the press with their confidential report. Question 45: “Don’t forget to tidy up the final draft before submission,” the team leader told us. A. The team leader ordered us to tidy up the final draft before submission. B. The team leader reminded us to tidy up the final draft before submission. C. The team leader asked us to tidy up the final draft before submission. D. The team leader simply wanted us to tidy up the final draft before submission. Question 46: “My company makes a large profit every year. Why don’t you invest more money in it?” my friend said to me. A. My friend instructed me how to put more money into his company. B. My friend persuaded me to invest more money in his company. C. I was asked to invest more money in my friend’s company. D. My friend suggested his investing more money in his company. Question 47: “Mum, please don’t tell dad about my mistake,” the boy said. A. The boy begged his mother not to tell his father about his mistake. B. The mother was forced to keep her son’s mistake as a secret when he insisted. C. The boy earnestly insisted that his mother tell his father about his mistake. D. The boy requested his mother not to talk about his mistake any more. Question 48: “If you don’t pay the ransom, we’ll kill your boy,” the kidnappers told us. A. The kidnappers ordered to kill our boy if we did not pay the ransom. B. The kidnappers pledged to kill our boy if we did not pay the ransom. C. The kidnappers threatened to kill our boy if we refused to pay the ransom. D. The kidnappers promised to kill our boy if we refused to pay the ransom. Mark Re le er A, B, C, or D o your a wer Ree o x dxca e Re correc a wer o eacR of Re followx g que xo . Question 49: Our boss would rather_________ during the working hours. A. us not chat B. we didn’t chat C. we don’t chat D. us not chatting Question 50: The sky was cloudy and foggy. We went to the beach, _________. A. so B. yet C. however D. even though ________THE END_________. Trang 4/4.

<span class='text_page_counter'>(90)</span> VnDoc - Tải tài liệu, văn bản pháp luật, biểu mẫu miễn phí. ĐÁP ÁN ĐỀ THI THPT QUỐC GIA 2017 MÔN: TIẾNG ANH. 1C 11A 21D 31C 41A. 2D 12A 22A 32D 42B. 3C 13A 23B 33A 43D. 4D 14B 24A 34A 44B. 5A 15B 25C 35D 45B. 6C 16A 26D 36C 46B. 7D 17B 27B 37A 47A. 8A 18C 28D 38B 48C. 9B 19B 29B 39A 49B. 10D 20A 30C 40A 50C. Trang 5/4.

<span class='text_page_counter'>(91)</span> SỞ GD-ĐT ĐỒNG NAI TRƯỜNG THPT CHUYÊN LƯƠNG THẾ VINH MÃ ĐỀ THI 121. KỲ THI THỬ THPT QUỐC GIA LẦN 2 NĂM 2017 MÔN THI: TIẾNG ANH Ngày thi: 14/5/2017 Thời gian làm bài: 60 phút (không kể thời gian phát đề) (Đề thi gồm 4 trang và 50 câu trắc nghiệm). Họ và tên thí sinh: _________________________________________________ Số báo danh: ______________ Read the following passage and mark the letter A, B, C, or D on your answer sheet to indicate the correct word or phrase that best fits each of the numbered blanks from 1 to 5. OXFORD UNIVERSITY Oxford University is a(n) 1 _____ of over 35 colleges, varying in date of foundation from medieval to more recent times. The colleges also vary in wealth, in character and in architecture. Some 2 _____ imposing buildings and grounds, others are almost modest in their scale. Most colleges are wellknown for their former students - Oxford is the place 3 _____ characters as diverse as Margaret Thatcher, Mrs. Gandhi, Michael Palin and Evelyn Waugh were educated. Most visitors will want to see a college and one or more of the University buildings, such as the Bodleian Library or the Ashmolean Museum. The University has supported the 4 _____ of The Oxford Story Exhibition, in Broad Street -now recognized as the best short 5 _____ to Oxford University. Here, during an entertaining ride through recreated scenes and events, Magnus Magnusson offers an informed view of Oxford's past and present. Special materials for children and foreign language books are also available. Câu 1: A. whole B. web C. hostel D. association Câu 2: A. exist B. stay C. live D. occupy Câu 3: A. where B. when C. which D. that Câu 4: A. discovery B. expectation C. investment D. creation Câu 5: A. beginning B. introduction C. history D. past Mark the letter A, B, C, or D on your answer sheet to indicate the word whose underlined part differs from the other three in pronunciation in each of the following questions. Câu 6: A. civilization B. scuba-diving C. dedication D. incredible Câu 7: A. rhinoceros B. wilderness C. digestion D. diversity Mark the letter A, B, C, or D on your answer sheet to indicate the correct answer to each of the following questions. Câu 8: Rosa suggested ___ a suit and tie when he went for the interview. A. he wear B. him to wear C. he must wear D. that he wears Câu 9: It was only later that I comprehended ___. A. what was his story’s meaning B. what his story meant C. what did his story mean D. his story meant what Câu 10: You promised her a letter. You ought to ___ days ago. A. have written it B. write it C. be writing it D. have it written Câu 11: Farmers make their soil more productive by using ___. A. fertilizers B. pesticides C. herbicides D. explosive Câu 12: Contamination and global warming have driven many species of animals and plants to the ___ of extinction. A. bank B. shore C. edge D. verge Câu 13: The fishermen and their boats ___ were off the coast three days ago have just been reported to be missing. A. who B. which C. that D. whom Câu 14: ___ schooling is compulsory in Australia between ___ ages of six and seventeen. A. No article/ the B. The/ no article C. A/ no article D. A/ the Câu 15: More than ten victims ___ missing in the storm last week. A. are reported to have been B. are reported to be C. have been reported being D. have reported to be Câu 16: You are not allowed to use the club’s facilities ___ you are a member. A. but for B. providing C. unless D. if only Câu 17: Lucia was upset because her boyfriend did not ___ at her birthday party. A. turn up B. come up C. show off D. come round Trang 1/4 - Mã đề thi 121.

<span class='text_page_counter'>(92)</span> Câu 18: You should show the interviewer that you are really keen ___ the job that you have applied for. A. over B. for C. in D. on Câu 19: All the ideas were good, but Michael ___ the best plan of all. A. came up with B. put up with C. got on with D. caught up on Read the following passage and mark the letter A, B, C, or D on your answer sheet to indicate the correct answer to each of the questions from 20 to 27. Fossils are the remains and traces (such as footprints or other marks) of ancient plant and animal life that are more than 10,000 years old. They range in size from microscopic structures to dinosaur skeletons and complete bodies of enormous animals. Skeletons of extinct species of human are also considered fossils. An environment favorable to the growth and later preservation of organisms is required for the occurrence of fossils. Two conditions are almost always present: (1) The possession of hard parts, either internal or external, such as bones, teeth, scales, shells, and wood; these parts remain after the rest of the organism has decayed. Organisms that lack hard parts, such as worms and jelly fish, have left a meager geologic record. (2) Quick burial of the dead organism, so that protection is afforded against weathering, bacterial action, and scavengers. Nature provides many situations in which the remains of animals and plants are protected against destruction. Of these, marine sediment is by far the most important environment for the preservation of fossils, owing to the incredible richness of marine life. The beds of former lakes are also prolific sources of fossils. The rapidly accumulating sediments in the channels, floodplains, and deltas of streams bury fresh-water organisms, along with land plants and animals that fall into the water. The beautifully preserved fossil fish from the Green River soil shale of Wyoming in the western United States lived in a vast shallow lake. The frigid ground in the far north acts as a remarkable preservative for animal fossils. Woolly mammoths, a long-haired rhinoceros, and other mammals have been periodically exposed in the icy land of Siberia, the hair and red flesh still frozen in cold storage. Volcanoes often provide environments favorable to fossil preservation. Extensive falls of volcanic ash and coarser particles overwhelm and bury all forms of life, from flying insects to great trees. Caves have preserved the bones of many animals that died in them and were subsequently buried under a blanket of clay or a cover of dripstone. Predatory animals and early humans alike sought shelter in caves and brought food to them to eat, leaving bones that paleontologists have discovered. Câu 20: The passage primarily discusses which of the following? A. How fossils are discovered B. What is learned from studying fossils C. Types of fossils found in different climates D. Conditions favorable to the preservation of fossils Câu 21: The word “traces” in bold is closest in meaning to A. structures B. importance C. skeletons D. imprints Câu 22: All of the following facts about fossils are referred to by the author (paragraph 1) EXCEPT the fact that they can be A. skeletons of human ancestors B. complete animal bodies C. microscopically small D. fragile Câu 23: The fossil fish from the Green River (paragraph 3) were probably preserved because they were A. buried slowly B. covered by sediment C. in a deep lake D. protected by oil Câu 24: The word “exposed” in bold is closest in meaning to A. photographed B. located C. preserved D. uncovered Câu 25: Which of the following is LEAST likely to be found as a fossil, assuming that all are buried rapidly? A. a human ancestor B. a worm C. a woolly mammoth D. a dinosaur Câu 26: The word “them” in bold refers to A. early humans B. predatory animals C. caves D. bones Câu 27: Which of the following is true of the environments in which fossils are found? A. Environments that favor fossilization have similar climates. B. Very different environments can favor fossilization. C. Environments that favor fossilization support large populations of animals. D. There are few environments in which fossils are protected.. Trang 2/4 - Mã đề thi 121.

<span class='text_page_counter'>(93)</span> Mark the letter A, B, C, or D on your answer sheet to indicate the word(s) OPPOSITE in meaning to the underlined word(s) in each of the following questions. Câu 28: My friend who taught me how to dance is an accomplished theatre performer. A. ill-educated B. unqualified C. unskilled D. well-known Câu 29: The goalkeeper can also be ejected for twenty seconds if a major foul is committed. A. prevented B. included C. eliminated D. sprinted Mark the letter A, B, C, or D on your answer sheet to indicate the most suitable response to complete each of the following exchanges Câu 30: Mrs Brown: “You’re late again, Joe.” Joe: “ __________.” A. Never mind. I stayed up too late last night. B. It’s OK. Thanks for telling me. C. I’m so sorry. The traffic was terrible. D. That’s alright. It’s my fault anyway. Câu 31: A: “This dish is really nice, Jolly.” B: “___. It’s called yakitori, and it’s made with chicken livers.” A. I’m glad you like it. B. It’s my pleasure. C. I couldn’t agree less. D. I guess you’re right. Mark the letter A, B, C, or D on your answer sheet to indicate the sentence that is closest in meaning to each of the following questions Câu 32: The house had burnt down before the fire brigade arrived. A. The house would have burnt down unless the fire brigade had arrived on time. B. The fire brigade didn’t arrive until the house had burnt down. C. The fire brigade were quite late, but they still tried to stop the house from burning down. D. The fire brigade arrived, but they failed to put the fire out. Câu 33: City centers are more dangerous than they were 20 years ago. A. In the last 20 years, city centers have become the most dangerous parts of cities. B. It has been 20 years since I was last in a city center as dangerous as this one. C. City centers aren’t as safe as they used to be 20 years ago. D. 20 years ago, I wasn’t aware that city centers were dangerous places. Câu 34: The girls were last seen just before the summer holiday. A. After the summer holiday was over, the girls disappeared. B. The girls have not been seen since they went on holiday last summer. C. Nobody has seen the girls since the summer holiday started. D. The last time we saw the girls was when we were on holiday in summer. Mark the letter A, B, C, or D on your answer sheet to indicate the word(s) CLOSEST in meaning to the underlined word(s) in each of the following questions. Câu 35: The increasing accumulation of garbage in oceans is due to a number of factors. A. dumping B. collecting C. building up D. throwing Câu 36: Most deserts are enormous sandy areas. A. narrow B. shallow C. immense D. mysterious Mark the letter A, B, C, or D on your answer sheet to indicate the word that differs from the other three in the position of primary stress in each of the following questions. Câu 37: A. compulsory B. compliment C. methodical D. certificate Câu 38: A. endanger B. catastrophe C. opponent D. vulnerable Mark the letter A, B, C, or D on your answer sheet to indicate the underlined part that needs correction in each of the following questions. Câu 39: A severely (A) punishment is (B) necessary to prevent (C) poaching and cutting trees illegally (D). Câu 40: Some (A) of the most famous Middle Eastern newspapers (B), the Pyramids, was established (C) in Egypt in 1875 (D). Câu 41: Public health experts say that the money (A) you spend to avoid (B) illness is less (C) than the cost of being sick (D). Trang 3/4 - Mã đề thi 121.

<span class='text_page_counter'>(94)</span> Mark the letter A, B, C, or D on your answer sheet to indicate the sentence that best combines each pair of sentences in the following questions. Câu 42: The roads were very slippery. They managed to complete the race. A. Slippery as the roads were, they managed to complete the race. B. The roads were so slippery that they could hardly complete the race. C. Slippery as the roads were, they failed to complete the race. D. The roads were so slippery; therefore, they could hardly complete the race. Câu 43: Peter drove carelessly. He had an accident yesterday. A. Had Peter driven more carefully, he wouldn’t have had an accident yesterday. B. If Peter drove more carefully, he would never have an accident. C. Unless Peter had driven more carefully, he would not have had an accident yesterday. D. If I were Peter, I would drive more carefully. Read the following passage and mark the letter A, B, C, or D on your answer sheet to indicate the correct answer to each of the questions from 44 to 50. As computers have become powerful tools for the rapid and economic production of pictures, computer graphics has emerged as one of the most rapidly growing fields in computer science. It is used routinely in such diverse areas as business, industry, art, government, education, research, training, and medicine. One of the initial uses of computer graphics, and ultimately its greatest use, has been as an aid to design, generally referred to as computer-aided design (CAD). One of its greatest advantages is that designers can see how an object will look after construction and make changes freely and much more quickly than with hand drafting. For three-dimensional rendering of machine parts, engineers now rely heavily on CAD. Automobile, spacecraft, aerospace, and ship designers use CAD techniques to design vehicles and test their performance. Building designs are also created with computer graphics systems. Architects can design a building layout, create a threedimensional model, and even go for a simulated "walk" through the rooms or around the outside of the building. Business graphics is another rapidly growing area of computer graphics, where it is used to create graphs, charts, and cost models to summarize financial, statistical, mathematical, scientific, and economic data. As an educational aid, computer graphics can be used to create weather maps and cartographic materials. Computer art also has creative and commercial art applications, where it is used in advertising, publishing, and film productions, particularly for computer animation, which is achieved by a sequential process. Câu 44: What does the passage mainly discuss? A. The rapidly growing field of computer science B. Routine use of computers C. Computers as the architects of the future D. Computer graphics applications Câu 45: The word “It” in bold refers to A. computer science B. computers C. fields D. computer graphics Câu 46: According to the passage, architects use CAD to A. inspect buildings B. create three-dimensional models C. make cartographic materials D. create graphs Câu 47: According to the passage, engineers use CAD for A. making cost models B. a simulated “walk” through model rooms. C. rendering machine parts D. advertising Câu 48: The word “applications” in bold means A. creativity B. layers C. uses D. jobs Câu 49: Which of the following is NOT mentioned as a use of computer graphics in business? A. Graphs B. Cost models C. Hiring D. Charts Câu 50: The paragraph following this passage would most likely be about A. flight training B. computer animation C. cost models D. applications of CAD in medicine -----------------------------------------------. ----------- THE END ----------. Trang 4/4 - Mã đề thi 121.

<span class='text_page_counter'>(95)</span> TRƯỜNG THPT CHUYÊN LƯƠNG THẾ VINH. ĐÁP ÁN THI THỬ ĐỢT 2 NĂM 2017 Câu-Mã đề 1 2 3 4 5 6 7 8 9 10 11 12 13 14 15 16 17 18 19 20 21 22 23 24 25 26 27 28 29 30 31 32 33 34 35 36 37 38 39 40 41. 121 D D A D B A B A B A A D C A A C A D A D D D B D B C B C B C A B C C C C B D A A B. 122 A D D B B C D B A B D C A D B A A D A D D A C B C A B C C C B D D D D B B C B A C. 123 C D B A C D C C B A B D D D D B C B B B D B B D A D B C D C A A D C B A A A B A C. 124 D A D B C B C C A D D D D B C B B B D B B C D D B A A C C B D D B A A C D D D C A.

<span class='text_page_counter'>(96)</span> 42 43 44 45 46 47 48 49 50. A A D D B C C C B. C A A C A C A A B. A C C C B D D A A. C B A A C A A C A.

<span class='text_page_counter'>(97)</span>

×